You are on page 1of 41

VISIONIAS

www.visionias.in
ANSWERS & EXPLANATIONS
GENERAL STUDIES (P) TEST – 3470 (2022)

Q 1.C
 एक भारतीय नागररक जो पूणण अयु एवं क्षमता का है, ऄपनी भारतीय नागररकता के पररत्याग की घोषणा कर सकता है। ऐसी
घोषणा के पंजीकरण के ईपरांत, वह व्यक्ति भारत का नागररक नहीं रहता है। आसके ऄक्ततररि जब कोइ व्यक्ति ऄपनी भारतीय
नागररकता का पररत्याग करता है, तो ईस व्यक्ति का प्रत्येक नाबाक्तिग बच्चा भी भारतीय नागररक नहीं रहता है। यद्यक्तप, ऐसे
बच्चे 18 वषण की अयु होने पर भारतीय नागररकता पुन: प्राप्त कर सकते हैं।
 आस मामिे में, बच्चे के नाबाक्तिग होने के कारण ईसकी भी नागररकता समाप्त हो जाती है। यदद वह वतणमान में (14 वषण) का है
तब भी वह नाबाक्तिग ही होगा। आस प्रकार, ईसे भारतीय नागररक नहीं माना जाएगा। आसक्तिए वह के वि ईन मूि ऄक्तधकारों
को ही प्राप्त कर सकता है जो नागररकों और क्तवदेक्तियों दोनों के क्तिए ईपिब्ध हैं:
o क्तवक्तध के समक्ष समता और क्तवक्तधयों का समान संरक्षण (ऄनुच्छेद 14)
o ऄपराधों के क्तिए दोषक्तसक्ति के संबंध में संरक्षण (ऄनुच्छेद 20)
o प्राण और दैक्तहक स्वतंत्रता का संरक्षण (ऄनुच्छेद 21)। ईच्चतम न्यायािय द्वारा “क्तनजता के ऄक्तधकार” को ऄनुच्छेद 21 के
तहत मूि ऄक्तधकार घोक्तषत दकया गया है। आसक्तिए क्तवकल्प 1 सही है।
o क्तिक्षा का ऄक्तधकार (ऄनुच्छेद 21A)
o कु छ दिाओं में क्तगरफ्तारी और क्तनरोध से संरक्षण (ऄनुच्छेद 22)। आसक्तिए क्तवकल्प 2 सही है।
o मानव के दुव्याणपार और बिाश्रम का प्रक्ततषेध (ऄनुच्छेद 23)।
o कारखानों अदद में बािकों के क्तनयोजन का प्रक्ततषेध (ऄनुच्छेद 24)।
o ऄंत:करण की और धमण के ऄबाध रूप से मानने, अचरण और प्रचार करने की स्वतंत्रता (ऄनुच्छेद 25)।
o धार्ममक कायों के प्रबंधन की स्वतंत्रता (ऄनुच्छेद 26)।
o दकसी क्तवक्तिष्ट धमण की ऄक्तभवृक्ति के क्तिए करों के संदाय के बारे में स्वतंत्रता (ऄनुच्छेद 27)।
o कु छ क्तिक्षा संस्थाओं में धार्ममक क्तिक्षा या धार्ममक ईपासना में ईपक्तस्थत होने के बारे में स्वतंत्रता (ऄनुच्छेद 28)।
 क्तनम्नक्तिक्तखत ऄक्तधकार के वि भारतीय नागररकों के क्तिए ईपिब्ध हैं:
o धमण, मूिवंि, जाक्तत, लिग या जन्मस्थान के अधार पर क्तवभेद का प्रक्ततषेध (ऄनुच्छेद 15)
o िोकक्तनयोजन के क्तवषय में ऄवसर की समता (ऄनुच्छेद 16)
o छह बुक्तनयादी स्वतंत्रताएं युक्तियुि क्तनबंधनों के ऄधीन हैं (ऄनुच्छेद 19)। आसक्तिए क्तवकल्प 3 सही नहीं है।
o ऄल्पसंख्यक-वगों की भाषा, क्तिक्तप और संस्कृ क्तत का संरक्षण (ऄनुच्छेद 29)
o क्तिक्षा संस्थाओं की स्थापना और प्रिासन करने का ऄल्पसंख्यक-वगों का ऄक्तधकार (ऄनुच्छेद 30)।

Q 2.A
 भारतीय संक्तवधान की प्रस्तावना-
 हम, भारत के िोग, भारत को एक सम्पूणण प्रभुत्व- संपन्न समाजवादी पंथक्तनरपेक्ष िोकतंत्रात्मक गणराज्य बनाने के क्तिए, ईसके
समस्त नागररकों को:
o सामाक्तजक, अर्मथक और राजनीक्ततक, न्याय; आसक्तिए कथन 1 सही है।
o क्तवचार, ऄक्तभव्यक्ति, क्तवश्वास, धमण और ईपासना की स्वतंत्रता; आसक्तिए कथन 2 सही नहीं है।
1 www.visionias.in ©Vision IAS
o प्रक्ततष्ठा और ऄवसर की समता प्राप्त कराने के क्तिए, तथा ईन सब में; आसक्तिए कथन 3 सही है।
o व्यक्ति की गररमा और राष्ट्र की एकता और ऄखंडता सुक्तनक्तित करने वािी बंधत
ु ा बढ़ाने के क्तिए दृढ संकल्प होकर; आसक्तिए
कथन 4 सही है।
 ऄपनी आस संक्तवधान सभा में अज तारीख 26 नवंबर 1949 इ० (क्तमक्तत मागणिीषण िुक्ि सप्तमी, संवत् दो हजार छह क्तवक्रमी) को
एतद्द्वारा आस संक्तवधान को ऄंगीकृ त, ऄक्तधक्तनयक्तमत और अत्मार्मपत करते हैं।
 प्रस्तावना से संक्तवधान के ऄक्तधकार के स्रोत का पता चिता है जो दक "भारत के िोगों से ऄक्तधकार" है।
 यह भारतीय राज्य की प्रकृ क्तत को संप्रभु, समाजवादी, पंथक्तनरपेक्ष, िोकतंत्रात्मक गणराज्य राजव्यवस्था के रूप में स्पष्ट करती
है।
 आसके तहत न्याय, स्वतंत्रता, समता और बंधत
ु ा को संक्तवधान के ईद्देश्यों के रूप में क्तनर्ददष्ट दकया गया है।
 आसमें संक्तवधान को ऄंगीकृ त करने की क्ततक्तथ ऄथाणत 26 नवंबर, 1949 है, का भी ईल्िेख है।

Q 3.B
 राष्ट्रपक्तत िासन के प्रभाव की घोषणा जारी होने के दो माह के भीतर आसका संसद के दोनों सदनों द्वारा ऄनुमोदन हो जाना
चाक्तहए।
 हािांदक, यदद राष्ट्रपक्तत िासन का घोषणा पत्र िोकसभा के क्तवघरित होने के समय जारी होता है या िोकसभा दो माह की
ऄवक्तध के भीतर ईद्घोषणा को मंजूरी ददए क्तबना क्तवघरित हो जाती है, तो घोषणा पत्र िोकसभा की पहिी बैठक से 30 ददनों
तक बना रहता है, बिते राज्यसभा ने आसे क्तनक्तित समय में स्वीकृ त कर ददया हो। यदद संसद के दोनों सदनों द्वारा आसे स्वीकृ त
दकया जाता है, तो राष्ट्रपक्तत िासन छह माह तक जारी रहता है।आसे संसद की मंजरू ी के साथ प्रत्येक छह माह में, ऄक्तधकतम तीन
वषण के क्तिए बढ़ाया जा सकता है। आसक्तिए कथन 1 सही नहीं है।
 राष्ट्रपक्तत ईद्घोषणा द्वारा:
o ईस राज्य की सरकार के सभी या कोइ कृ त्य और राज्यपाि में या राज्य के क्तवधान-मंडि से क्तभन्न राज्य के दकसी क्तनकाय या
प्राक्तधकारी में क्तनक्तहत या ईसके द्वारा प्रयोिव्य सभी या कोइ िक्तियां ऄपने हाथ में िे सके गा; आसक्तिए कथन 2 सही है।
o यह घोषणा कर सके गा दक राज्य के क्तवधान-मंडि की िक्तियां संसद द्वारा या ईसके प्राक्तधकार के ऄधीन प्रयोिव्य होंगी;
o ऐसे अनुषंक्तगक और पररणाक्तमक ईपबंध कर सके गा जो ईद्घोषणा के ईद्देश्यों को प्रभावी करने के क्तिए राष्ट्रपक्तत को
अवश्यक या वांछनीय प्रतीत हों;

Q 4.B
 इ9(E9) नौ देिों यथा बांग्िादेि, ब्राजीि, चीन, क्तमस्र, भारत, आं डोनेक्तिया, मैक्तक्सको, नाआजीररया और पादकस्तान का एक
ऐसा मंच है, क्तजसका ईद्देश्य यूनस्े को की “सभी के क्तिए क्तिक्षा” (Education For All: EFA) पहि के िक्ष्यों को प्राप्त करना है।
 इ9 साझेदारी सवणप्रथम वषण 1993 में स्थाक्तपत की गइ थी, क्तजसका गठन यूनेस्को की ‘सभी के क्तिए क्तिक्षा (EFA)’ पहि के
िक्ष्यों को प्राप्त करने हेतु दकया गया था। इ9 साझेदारी सतत क्तवकास िक्ष्य (SDG) 4 - क्तिक्षा 2030 को प्राप्त करने के क्तिए
कायण कर रही है।
 हाि ही में, क्तिक्षा राज्य मंत्री ने इ9 देिों के क्तिक्षा मंक्तत्रयों की परामिण बैठक में भाग क्तिया, क्तजसका क्तवषय इ9 पहि: SDG4
की ददिा में प्रगक्तत को गक्तत देने के क्तिए क्तडक्तजिि क्तिक्षा को बढ़ावा देना' था।
 आसक्तिए क्तवकल्प (b) सही ईत्तर है।

Q 5.A
 दूसरी ऄनुसूची में क्तनम्नक्तिक्तखत की पररिक्तब्धयां, भत्ते, क्तविेषाक्तधकार अदद से संबंक्तधत प्रावधान ईपबंक्तधत हैं:
o भारत के राष्ट्रपक्तत।
o राज्यों के राज्यपािों।
o िोकसभा के ऄध्यक्ष और ईपाध्यक्ष।
o राज्यसभा के सभापक्तत और ईपसभापक्तत।
o राज्यों की क्तवधान सभा के ऄध्यक्ष और ईपाध्यक्ष।
o राज्यों की क्तवधान पररषद के सभापक्तत और ईपसभापक्तत।
2 www.visionias.in ©Vision IAS
o ईच्चतम न्यायािय के न्यायाधीिों।
o ईच्च न्यायाियों के न्यायाधीिों।
o भारत के क्तनयंत्रक- महािेखापरीक्षक।
 दूसरी ऄनुसूची में प्रावधानों को ऄनुच्छेद 368 के दायरे से बाहर संसद के दोनों सदनों के साधारण बहुमत से संिोक्तधत दकया
जा सकता है।
 आसक्तिए क्तवकल्प (a) सही ईत्तर है।

Q 6.C
 कथन 1 सही नहीं है: ऄनुच्छेद 48(क) पयाणवरण का संरक्षण तथा संवधणन और वन तथा वन्यजीवों की रक्षा का प्रावधान करता
है। जबदक ऄनुच्छेद 48 में कृ क्तष और पिुपािन को अधुक्तनक और वैज्ञाक्तनक प्रणाक्तियों से संगरठत करने का प्रयास करने तथा
क्तवक्तिष्टतया गायों, बछडों और ऄन्य दुधारू एवं वाहक पिुओं की नस्िों के परररक्षण और सुधार के क्तिए ईनके वध का प्रक्ततषेध
करने के क्तिए कदम ईठाने का प्रावधान दकया गया है।
o आसक्तिए वन्यजीव (संरक्षण) ऄक्तधक्तनयम, 1972 और वन (संरक्षण) ऄक्तधक्तनयम, 1980 ऄनुच्छेद 48(क) को प्रभावी िागू
करने के क्तिए ऄक्तधक्तनयक्तमत दकये गए हैं, न दक ऄनुच्छेद 50 को।
 कथन 2 सही है: ऄनुच्छेद 43, राज्य के ग्रामीण क्षेत्रों में कु िीर ईद्योगों को वैयक्तिक या सहकारी अधार पर बढ़ाने का प्रयास
करने का प्रावधान करता है।
o आसक्तिए ऄनुच्छेद 43 को प्रभावी करने के क्तिए खादी और ग्रामोद्योग बोडण, खादी और ग्रामोद्योग अयोग, िघु ईद्योग बोडण
अदद की स्थापना की गइ है।
 कथन 3 सही है: ऄनुच्छेद 40, राज्यों के द्वारा ग्राम पंचायतों को संगरठत करने और ईन्हें स्वायत्त िासन की आकाआयों के रूप में
कायण करने योग्य बनाने हेतु अवश्यक ईपबंध करने का प्रावधान करता है।
o आसक्तिए ऄनुच्छेद 40 को प्रभावी करने के क्तिए 73वां संवैधाक्तनक संिोधन ऄक्तधक्तनयम पाररत दकया गया था।
Q 7.D
 हाि ही में, 144 वषों में पहिी बार, जम्मू-कश्मीर (J&K) प्रिासन द्वारा वतणमान कोक्तवड-19 संकि के मद्देनज़र संघ राज्य क्षेत्र
की राजधानी के वषण में दो बार होने वािे स्थानांतरण, क्तजसे ‘दरबार स्थानांतरण’ (Darbar Move) कहा जाता है, को रोक
ददया गया है।
 दरबार स्थानांतरण एक सदी-पुरानी प्रथा है क्तजसमें सरकार संघ राज्य क्षेत्र (पूवणवती राज्य) की दोनों राजधाक्तनयों ऄथाणत
श्रीनगर और जम्मू में प्रत्येक छह माह के क्तिए कायण करती है, ऄथाणत राजधानी का वषण में दो बार या छमाही स्थानांतरण होता
है।
 संघ राज्य क्षेत्र की ग्रीष्मकािीन राजधानी श्रीनगर है जहां ऄक्िू बर के ऄंत तक कायण दकया जाता है। ईसके बाद नवंबर के प्रथम
सप्ताह में राजधानी स्थानांतररत हो कर जम्मू बन जाती है जो िीतकािीन राजधानी है तथा आस दौरान सरकार यहां कायण
करती है।
 आसक्तिए क्तवकल्प (d) सही है।
Q 8.D
 भारत में सरकार के संसदीय स्वरूप की कु छ क्तविेषताएं क्तनम्नक्तिक्तखत हैं:
o नाममात्र की एवं वास्तक्तवक कायणपाक्तिका: राष्ट्रपक्तत नाममात्र की कायणपाक्तिका (de jure executive or titular
executive) होता है, जबदक प्रधानमंत्री वास्तक्तवक कायणपाक्तिका (de facto executive) होता है। आस प्रकार, राष्ट्रपक्तत
राज्य का मुक्तखया होता है, जबदक प्रधानमंत्री सरकार का मुक्तखया होता है। ऄनुच्छेद 74, के तहत राष्ट्रपक्तत को ऄपने कायों
के क्तनष्पादन हेतु सहायता एवं सिाह प्रदान करने हेतु एक मंक्तत्रपररषद का प्रावधान दकया गया है क्तजसका प्रधान
प्रधानमंत्री होता है। ईसके परामिण को मानने के क्तिए राष्ट्रपक्तत बाध्य होता है।
o बहुमत प्राप्त दि का िासन: क्तजस राजनीक्ततक दि को िोकसभा में बहुमत ऄथाणत( 50% + 1 सीिें) प्राप्त होता हैं, वह
सरकार का गठन करती है। ईस दि के नेता को राष्ट्रपक्तत द्वारा प्रधानमंत्री क्तनयुि दकया जाता है। ऄन्य मंक्तत्रयों को
प्रधानमंत्री के परामिण पर राष्ट्रपक्तत द्वारा क्तनयुि दकया जाता है। यद्यक्तप, जब दकसी एक दि को बहुमत प्राप्त नहीं होता है
तब राष्ट्रपक्तत द्वारा दिों के गठबंधन को सरकार बनाने के क्तिए अमंक्तत्रत दकया जा सकता है।
o सामूक्तहक ईत्तरदाक्तयत्व: यह संसदीय सरकार का मूि क्तसिांत है। मंक्तत्र-पररषद सामान्य रूप में संसद के प्रक्तत और क्तवक्तिष्ट
रूप में िोकसभा के प्रक्तत सामूक्तहक रूप से ईत्तरदायी होती है (ऄनुच्छेद 75)। सभी मंत्री एक िीम के रूप में कायण करते हैं

3 www.visionias.in ©Vision IAS


और साथ-साथ संसद सदस्य बने रहते हैं या ऄपनी सदस्यता खो देते हैं। सामूक्तहक ईत्तरदाक्तयत्व का ऄथण है दक िोकसभा
ऄक्तवश्वास प्रस्ताव पाररत कर मंक्तत्रपररषद (ऄथाणत प्रधानमंत्री के नेतृत्व वािी मंक्तत्रपररषद) को हिा सकती है।
o राजनीक्ततक एकरूपता: सामान्यतः मंक्तत्रपररषद के सदस्य एक ही राजनीक्ततक दि से संबंक्तधत होते हैं और आसक्तिए ईनकी
समान राजनीक्ततक क्तवचारधारा होती है। गठबंधन सरकार के मामिे में मंत्री सवणसम्मक्तत के प्रक्तत बाध्य होते हैं।
o दोहरी सदस्यता: मंत्री, क्तवधाक्तयका और कायणपाक्तिका दोनों के सदस्य होते हैं। आसका ऄथण है दक कोइ भी व्यक्ति क्तबना संसद
का सदस्य बने मंत्री नहीं बन सकता है। संक्तवधान यह ईपबंक्तधत करता है दक यदद कोइ व्यक्ति संसद का सदस्य नहीं है और
मंत्री बनता है तो ईसे 6 माह के भीतर संसद का सदस्य बनना होगा। आसक्तिए कथन 2 सही है।
o प्रधानमंत्री का नेतत्ृ व: सरकार की आस व्यवस्था में प्रधानमंत्री नेतृत्वकताण की भूक्तमका क्तनभाता है। वह मंक्तत्रपररषद का,
संसद का और सत्तारूढ़ दि का नेता होता है। आन क्षमताओं में वह सरकार के संचािन में महत्वपूणण और ऄत्यंत क्तनणाणयक
भूक्तमका क्तनभाता है। आसक्तिए कथन 3 सही है।
o क्तनचिे सदन का क्तवघिन: संसद के क्तनचिे सदन (िोकसभा) को प्रधानमंत्री की क्तसफाररि पर राष्ट्रपक्तत द्वारा क्तवघरित दकया
जा सकता है। दूसरे िब्दों में, प्रधानमंत्री मंक्तत्रपररषद का कायणकाि समाप्त होने से पूवण िोकसभा को क्तवघरित करने और नए
चुनाव अयोक्तजत करवाने के क्तिए राष्ट्रपक्तत को क्तसफाररि कर सकता है। आसका ऄथण है दक संसदीय प्रणािी में कायणपाक्तिका
को क्तवधाक्तयका को क्तवघरित करने का ऄक्तधकार प्राप्त होता है।
o गोपनीयता: मंत्री गोपनीयता के क्तसिांत पर कायण करते हैं और ऄपनी कायणवाक्तहयों, नीक्ततयों एवं क्तनणणयों के संबंध में
जानकारी नहीं दे सकते हैं। वे ऄपना पद ग्रहण करने से पूवण गोपनीयता की िपथ िेते हैं। मंक्तत्रयों को गोपनीयता की िपथ
राष्ट्रपक्तत द्वारा ददिाइ जाती है। आसक्तिए कथन 1 सही है।

Q 9.B
 ऄनुच्छेद 358 और 359 मूि ऄक्तधकारों पर राष्ट्रीय अपातकाि के प्रभाव का वणणन करते हैं। ऄनुच्छेद 358, ऄनुच्छेद 19 द्वारा
प्रत्याभूत मूि ऄक्तधकारों के क्तनिंबन से संबंक्तधत है, जबदक ऄनुच्छेद 359 ऄन्य मूि ऄक्तधकारों के क्तनिंबन (ऄनुच्छेद 20 और 21
द्वारा प्रत्याभूत ऄक्तधकारों को छोडकर) से संबंक्तधत है।
 ऄनुच्छेद 19 में सक्तम्मक्तित छः ऄक्तधकारों को के वि तभी क्तनिंक्तबत दकया जा सकता है जब राष्ट्रीय अपातकाि की ईद्घोषणा युि
या बाह्य अक्रमण के अधार पर की जाती है न दक सिस्त्र क्तवद्रोह के अधार पर।
 ऄनुच्छेद 19 स्वतः क्तनिंक्तबत तभी होता है जब अपातकाि की ईद्घोषणा का अधार या तो युि या बाह्य अक्रमण हो। प्रारं भ में
यह प्रावधान संक्तवधान में ईपबंक्तधत नहीं था। आसे 44वें संक्तवधान संिोधन द्वारा ऄंत:स्थक्तपत दकया गया है। आसक्तिए कथन 1
सही नहीं है।
 ऄनुच्छेद 359 राष्ट्रपक्तत को राष्ट्रीय अपातकाि के दौरान मूि ऄक्तधकारों के प्रवतणन के क्तिए न्यायािय में जाने के ऄक्तधकार को
क्तनिंक्तबत करने के क्तिए ऄक्तधकृ त करता है। आसका ऄथण है दक ऄनुच्छेद 359 के ऄंतगणत मूि ऄक्तधकार नहीं बक्तल्क के वि ईनका
प्रवतणन क्तनिंक्तबत होता है।
 यह क्तनिंबन के वि ईन्ही मूि ऄक्तधकारों के प्रवतणन से संबंक्तधत है जो राष्ट्रपक्तत के अदेि में वर्मणत होते हैं।
 आसके ऄक्ततररि, यह क्तनिंबन अपातकाि की प्रवतणन ऄवक्तध ऄथवा अदेि में वर्मणत ऄल्पावक्तध के क्तिए िागू होता हैं और
क्तनिंबन का अदेि भारत के संपण
ू ण राज्यक्षेत्र ऄथवा ईसके दकसी भाग पर िागू दकया जा सकता है। आस प्रकार के राष्ट्रपक्तत के
अदेि को मंजरू ी के क्तिए संसद के प्रत्येक सदन के समक्ष प्रस्तुत करना होता है। आसक्तिए कथन 2 सही है।
 44वां संक्तवधान संिोधन ऄक्तधक्तनयम 1978, ऄनुच्छेद 359 के क्तवषय क्षेत्र पर दो प्रक्ततबंध िगाता है- प्रथम, राष्ट्रपक्तत ऄनुच्छेद
20 तथा 21 के ऄंतगणत ददए गए ऄक्तधकारों को प्रवर्मतत कराने के क्तिए न्यायािय से ईपचार प्राप्त करने के ऄक्तधकार को क्तनिंक्तबत
नहीं कर सकता है। दूसरे िब्दों में, ऄपराधों के क्तिए दोषक्तसक्ति के संबंध में संरक्षण का ऄक्तधकार (ऄनुच्छेद 20) तथा प्राण और
दैक्तहक स्वतंत्रता का ऄक्तधकार (ऄनुच्छेद 21) अपातकाि के दौरान भी िागू रहते है।
 क्तद्वतीय, के वि ईन्हीं कानूनों को चुनौती से संरक्षण प्राप्त हैं जो अपातकाि से संबंक्तधत हैं न दक दूसरे कानून और के वि ईन्हीं
कायणकारी कायों को संरक्षण प्राप्त हैं जो के वि ऐसे कानून के तहत क्तिए गए हैं।

Q 10.A
 29 ऄगस्त 1947 को, संक्तवधान सभा ने एक प्रस्ताव के माध्यम से संवैधाक्तनक सिाहकार द्वारा तैयार दकए गए भारतीय
संक्तवधान के पाठ के प्रारूप की समीक्षा करने के क्तिए और सक्तमक्तत द्वारा यथा संिोक्तधत संक्तवधान प्रारूप के पाठ को संक्तवधान
सभा के समक्ष क्तवचार हेतु प्रस्तुत करने के क्तिए एक प्रारूप सक्तमक्तत का गठन दकया।

4 www.visionias.in ©Vision IAS


 प्रारूप सक्तमक्तत में सात सदस्य िाक्तमि थे:
o ऄल्िादी कृ ष्णस्वामी ऄय्यर;
o एन. गोपािस्वामी अयंगर;
o बी. अर. ऄम्बेडकर;
o के .एम. मुि
ं ी;
o सैय्यद मोहम्मद सादुल्िा;
o बी. एि. क्तमत्र; और
o डी. पी. खेतान
 प्रारूप सक्तमक्तत ने 30 ऄगस्त 1947 को ऄपनी प्रथम बैठक में बी. अर. ऄम्बेडकर को ऄपना ऄध्यक्ष चुना।
 ऄक्िू बर 1947 के ऄंत में, प्रारूप सक्तमक्तत ने संवध
ै ाक्तनक सिाहकार बी. एन. राव द्वारा तैयार संक्तवधान प्रारूप की समीक्षा प्रारं भ
की। आसने प्रारूप में ऄनेक पररवतणन दकए तथा 21 फरवरी 1948 को संक्तवधान सभा के ऄध्यक्ष के समक्ष संक्तवधान प्रारूप को
प्रस्तुत दकया।
 संक्तवधान सभा में हुए वाद-क्तववादों में से ऄक्तधकांि चचाणएं प्रारूप सक्तमक्तत द्वारा तैयार दकए गए संक्तवधान प्रारूप के आदण-क्तगदण ही
थे। संक्तवधान सभा के 165 बैठकों में से 114 बैठकों में के वि संक्तवधान प्रारूप पर ही चचाण की गइ थी।
 आसक्तिए क्तवकल्प (d) सही ईत्तर है।

Q 11.D
 संक्तवधान सभा का गठन नवंबर 1946 में कै क्तबनेि क्तमिन योजना द्वारा तैयार की गइ योजना के तहत दकया गया था।
 संक्तवधान सभा को एक संप्रभुता संपन्न क्तनकाय बनाया गया था, जो ऄपनी खुिी से कोइ भी संक्तवधान तैयार सकती थी। सभा को
भारत के संबंध में क्तब्ररिि संसद द्वारा बनाए गए दकसी भी कानून को रद्द या पररवर्मतत करने के क्तिए ऄक्तधकृ त दकया गया था।
आसक्तिए कथन 1 सही है।
 यह एक क्तवधायी क्तनकाय के रूप में कायण करता था ऄथाणत आसे दो पृथक कायण सौंपे गए थे - पहिा, स्वतंत्र भारत के क्तिए एक
संक्तवधान का क्तनमाणण करना तथा दूसरा, देि के क्तिए साधारण कानूनों का क्तनमाणण करना। आस प्रकार, संक्तवधान सभा स्वतंत्र
भारत की पहिी संसद (ऄक्तधराज्य क्तवधानमंडि) बन गइ। आसक्तिए कथन 2 सही है।
 जब भी सभा की बैठक संक्तवधान सभा के रूप में की जाती थी, आसकी ऄध्यक्षता डॉ. राजेंद्र प्रसाद के द्वारा की जाती थी और जब
आसकी बैठक संसद के रूप में होती थी, तब आसकी ऄध्यक्षता जी. वी. माविंकर के द्वारा की जाती थी। ये दोनों कायण 26 नवंबर,
1949 तक जारी रहे, जब तक संक्तवधान के क्तनमाणण का कायण समाप्त नहीं हुअ।
 हािांदक, भारत के िोगों द्वारा संक्तवधान सभा के सदस्यों का चुनाव प्रत्यक्ष वयस्क मताक्तधकार के अधार पर नहीं दकया गया
था। ककतु संक्तवधान सभा में भारतीय समाज के सभी वगों के प्रक्ततक्तनक्तध िाक्तमि थे- लहदू, मुक्तस्िम, क्तसख, पारसी, अंग्ि-
भारतीय, भारतीय इसाइ, ऄनुसक्तू चत जाक्तत, ऄनुसूक्तचत जनजाक्तत सक्तहत आन सभी वगों की मक्तहिाएं भी िाक्तमि थीं। आसक्तिए
कथन 3 सही है।

Q 12. D
 ऄनुच्छेद 25, सभी व्यक्तियों को ऄंतःकरण की स्वतंत्रता (Freedom of Conscience) और धमण के ऄबाध रूप से मानने,
अचरण करने एवं प्रसार करने की स्वतंत्रता का समान ऄक्तधकार प्रदान करता है।
 आनके क्तनक्तहताथण हैं:
o ऄंतःकरण की स्वतंत्रता: सभी को इश्वर या ईसके रूपों के साथ ऄपने ढंग से संबंध स्थाक्तपत करने की अंतररक स्वतंत्रता।
o धमण को ऄबाध रूप से मानने का ऄक्तधकार: ऄपने धार्ममक क्तवश्वास और अस्था की सावणजक्तनक और क्तबना भय के घोषणा
करने का ऄक्तधकार।
o धमाणचरण का ऄक्तधकार: धार्ममक पूजा, रीक्तत-ररवाज़, समारोह और ऄपनी अस्था एवं क्तवचारों के प्रदिणन करने की
स्वतंत्रता।
o प्रसार का ऄक्तधकार: ऄपने धार्ममक मान्यताओं को दूसरों तक पहुंचाना और ईसका प्रसार करना या ऄपने धमण के क्तसिांतों
को ईजागर करना। परन्तु, आसमें दकसी व्यक्ति को ऄपने धमण में धमाणन्तररत करने का ऄक्तधकार सक्तम्मक्तित नहीं है।

5 www.visionias.in ©Vision IAS


 ईपयुि
ण प्रावधानों से यह स्पष्ट होता है दक ऄनुच्छेद 25 न के वि धार्ममक क्तवश्वासों को बक्तल्क धार्ममक अचरणों (ऄनुष्ठानों) को
भी समाक्तहत करता है। आसके ऄक्ततररि, ये ऄक्तधकार सभी व्यक्तियों ऄथाणत,् नागररकों एवं गैर-नागररकों के क्तिए ईपिब्ध हैं।
आसक्तिए कथन 1 सही नहीं है।
 हािांदक, ये ऄक्तधकार सावणजक्तनक व्यवस्थाओं, नैक्ततकता, स्वास््य एवं मूि ऄक्तधकारों से संबंक्तधत ऄन्य प्रावधानों के ऄनुसार है।
आसके ऄक्ततररि, यह राज्य को क्तनम्नक्तिक्तखत बात की ऄनुमक्तत देता है:
o धार्ममक अचरण से संबंक्तधत दकसी अर्मथक, क्तवत्तीय, राजनीक्ततक या ऄन्य िौदकक दक्रयाकिाप का क्तवक्तनयमन या क्तनबंधन
करे ।
o सामाक्तजक कल्याण और सुधार के क्तिए प्रावधान करना या सावणजक्तनक प्रकार की लहदुओं की धार्ममक संस्थाओं को क्तहन्दुओं
के सभी वगों और ऄनुभागों के क्तिए खोिना।
 ऄनुच्छेद 28 के ऄंतगणत, राज्य क्तनक्तध से पूणत
ण : पोक्तषत दकसी क्तिक्षा संस्था में कोइ धार्ममक क्तिक्षा नहीं दी जाएगी।
 हािांदक, ऐसा कोइ बात ऐसी क्तिक्षा संस्था को िागू नहीं होगी क्तजसका प्रिासन राज्य करता है ककतु जो दकसी ऐसे क्तवन्यास या
न्यास के ऄधीन स्थाक्तपत हुइ है क्तजसके ऄनुसार ईस संस्था में धार्ममक क्तिक्षा देना अवश्यक है। आसक्तिए कथन 2 सही नहीं है।
 आसके ऄक्ततररि, राज्य द्वारा मान्यता प्राप्त या राज्य क्तनक्तध से सहायता प्राप्त दकसी भी िैक्षक्तणक संस्थान में, ईपक्तस्थत होने वािे
दकसी भी व्यक्ति को ईसकी सहमक्तत के क्तबना ईस संस्था में दकसी भी धार्ममक क्तिक्षा या ईपासना में िाक्तमि होने के क्तिए बाध्य
नहीं दकया जाएगा।
 आस प्रकार, ऄनुच्छेद 28 चार प्रकार के िैक्तक्षक संस्थानों के मध्य क्तवभेद करता है:
(a) ऐसे संस्थान, क्तजनका रख-रखाव पूणत
ण ः राज्य द्वारा दकया जाता है।
(b) ऐसे संस्थान, क्तजनका प्रिासन राज्य द्वारा दकया जाता है िेदकन ईनकी स्थापना दकसी क्तवन्यास या न्यास के ऄंतगणत हुइ हो।
(c) राज्य द्वारा मान्यता प्राप्त संस्थान।
(d) राज्य द्वारा क्तवत्तीय सहायता प्राप्त करने वािे संस्थान।
 प्रावधान (a) में धार्ममक क्तनदेि पूरी तरह से प्रक्ततबंक्तधत है जबदक (b) में, धार्ममक क्तिक्षा की ऄनुमक्तत है। (c) और (d) में
स्वैक्तच्छक अधार पर धार्ममक क्तिक्षा की ऄनुमक्तत है।

Q 13. A
 दोहरी नागररकता दकसी संघीय राजव्यवस्था की ऄक्तनवायण क्तविेषता नहीं है। भारत जैसे कु छ संघीय देिों में एकि नागररकता
है। दकसी संघीय राजव्यवस्था की ऄक्तनवायण क्तविेषताएं है:
 राजव्यवस्था के दो समुच्चय- राष्ट्रीय स्तर और क्षेत्रीय स्तर पर
 क्तिक्तखत संक्तवधान
 कठोर संक्तवधान
 संक्तवधान की सवोच्चता
 स्वतंत्र न्यायपाक्तिका
 क्तद्वसदनीय क्तवधानमंडि
 राष्ट्रीय और क्षेत्रीय सरकारों के मध्य िक्तियों का पृथक्करण
 आसक्तिए क्तवकल्प (a) सही ईत्तर है।

Q 14. A
 पहि (Initiative) एक तरीका है, क्तजसके माध्यम से जनता, क्तवधाक्तयका को दकसी क्तवधेयक को िागू करने के क्तिए प्रस्ताक्तवत कर
सकती हैं। जनमत संग्रह (Referendum) एक ऐसी प्रदक्रया है क्तजसमें प्रस्ताक्तवत क्तवधान क्तनवाणचक मंडि को संदर्मभत दकया
जाता है तादक ईसका क्तनधाणरण ईनके प्रत्यक्ष मतों के माध्यम से दकया जाए। पयाणप्त समथणन के साथ दकसी भी प्रस्ताक्तवत क्तवक्तध
को चुनाव में मतदान के क्तिए रखा जा सकता है। आसक्तिए युग्म 1 सही सुमक्ते ित है।
 वापस बुिाना (Recall), एक ईपाय है क्तजसके माध्यम से मतदाता एक जनप्रक्ततक्तनक्तध ऄथवा दकसी ऄक्तधकारी को कतणव्यों का
पािन करने में क्तवफि होने पर ईसके कायणकाि से पहिे ही हिा सकते हैं। आसक्तिए युग्म 2 सही सुमक्ते ित नहीं है।
 िोकमत (Plebiscite), सावणजक्तनक महत्व के दकसी भी मुद्दे पर िोगों की राय प्राप्त करने की क्तवक्तध है। दकसी महत्वपूणण
सावणजक्तनक प्रश्न के संबंध में यह दकसी राज्य के योग्य मतदाताओं का प्रत्यक्ष मत है। आसक्तिए युग्म 3 सही सुमक्ते ित नहीं है।

6 www.visionias.in ©Vision IAS


Q 15. D
 संक्तवधान के भाग IVA में ऄनुच्छेद 51A के तहत ग्यारह मौक्तिक कतणव्यों को प्रक्ततस्ठाक्तपत दकया गया हैं-
 भारतीय संक्तवधान का पािन करना और ईसके अदिों और संस्थाओं जैसे राष्ट्र गान और राष्ट्र ध्वज का अदर करना। आसक्तिए
कथन 1 सही है।
 स्वतंत्रता के क्तिए राष्ट्रीय अंदोिन को प्रेररत करने वािे ईच्च अदिों को हृदय में संजोए रखना और ईनका पािन करना।
 भारत की संप्रभुता, एकता और ऄखंडता की रक्षा करना।
 देि की रक्षा करना और अह्वान दकए जाने पर राष्ट्र की सेवा करना।
 भारत के सभी िोगों में समरसता और समान भ्रातृत्व की भावना का क्तनमाणण करना और ऐसी प्रथाओं का त्याग करना जो
क्तस्त्रयों के सम्मान के क्तवरुि हैं। आसक्तिए कथन 4 सही है।
 हमारी सामाक्तसक संस्कृ क्तत की गौरविािी परं परा का महत्व समझना और ईसका परररक्षण करना।
 प्राकृ क्ततक पयाणवरण जैसे झीि, वन्य जीवों, नददयों और वनों अदद की रक्षा करना और संवधणन करना। आसक्तिए कथन 2 सही है।
 वैज्ञाक्तनक दृक्तष्टकोण मानववाद और ज्ञानाजणन तथा सुधार की भावना का क्तवकास करना।
 सावणजक्तनक संपक्तत्त को सुरक्तक्षत रखना और लहसा से दूर रहना।
 व्यक्तिगत और सामूक्तहक गक्ततक्तवक्तधयों के सभी क्षेत्रों में ईत्कषण की ओर बढ़ने का सतत प्रयास करना।
 समान न्याय को बढ़ावा देना और िोगों का कल्याण, क्रमिः ऄनुच्छेद 39A और ऄनुच्छेद 38 के ऄंतगणत अता है। यह संक्तवधान
के भाग IV में प्रदत्त राज्य के नीक्तत के क्तनदेिक तत्व का भाग है। आसक्तिए कथन 3 सही नहीं है।

Q 16. D
 ऄनुच्छेद 39(b) सामूक्तहक क्तहत के क्तिए समुदाय के भौक्ततक संसाधनों के समान क्तवतरण का प्रावधान करता है। ऄनुच्छेद 39(c)
धन एवं ईत्पादन-साधनों के संकेंद्रण को रोकने से संबक्तं धत ईपबंध करता है।
 ऄनुच्छेद 39(b) ईपबंध करता है दक राज्य ऄपनी नीक्तत का, क्तवक्तिष्टतया, आस प्रकार संचािन करे गा दक सुक्तनक्तित रूप से:
समुदाय के भौक्ततक संसाधनों का स्वाक्तमत्व और क्तनयंत्रण आस प्रकार बंिा हो क्तजससे सामूक्तहक क्तहत का सवोत्तम रूप से साधन हो।
 ऄनुच्छेद 39(c) ईपबंध करता है दक राज्य ऄपनी नीक्तत का, क्तवक्तिष्टतया, आस प्रकार संचािन करे गा दक सुक्तनक्तित रूप से:
अर्मथक व्यवस्था आस प्रकार चिे क्तजससे धन और ईत्पादन के साधनों का सवणसाधारण के क्तिए ऄक्तहतकारी संकेन्द्रण न हो।
 गोिकनाथ वाद (1967) में, ईच्चतम न्यायािय ने स्पष्ट दकया था दक संसद, राज्य के नीक्तत क्तनदेिक तत्व को प्रभावी करने के
क्तिए मौक्तिक ऄक्तधकारों में संिोधन नहीं कर सकती है। संसद ने गोिकनाथ वाद (1967) में ईच्चतम न्यायािय के क्तनणणय पर
संक्तवधान के 25वें संिोधन ऄक्तधक्तनयम के द्वारा प्रक्ततदक्रया व्यि की, क्तजसके फिस्वरूप भाग III में एक नया ऄनुच्छेद 31C को
जोडा गया। ऄनुच्छेद 31C में दो प्रावधान ऄन्तर्मवष्ट थे:
o यदद ऄनुच्छेद 39(b) और ऄनुच्छेद 39(c) में वर्मणत राज्य के नीक्तत क्तनदेिक तत्वों को िागू कराने वािी दकसी क्तवक्तध को
आस अधार पर ऄसंवध
ै ाक्तनक और िून्य घोक्तषत नहीं दकया जा सकता दक वह ऄनुच्छेद 14, ऄनुच्छेद 19 या ऄनुच्छेद 31
द्वारा प्रदत्त मौक्तिक ऄक्तधकारों का ईल्िंघन करता है।
o कोइ क्तवक्तध, क्तजसमें यह घोषणा है दक वह ऄनुच्छेद 39(b) और ऄनुच्छेद 39(c) में वर्मणत राज्य के नीक्तत क्तनदेिक तत्वों को
प्रभावी करने के क्तिए है, दकसी न्यायािय में आस अधार पर प्रश्नगत नहीं की जाएगी दक वह ऐसी नीक्तत को प्रभावी नहीं
करती है।
 बाद में संसद द्वारा 42वें संक्तवधान संिोधन को पाररत दकया गया, आसके माध्यम से, ऄनुच्छेद 31C की पहिी व्यवस्था के
प्रावधानों को क्तवस्ताररत दकया गया। आसमें न के वि ऄनुच्छेद 39(b) या (c) में वर्मणत नीक्तत को बक्तल्क संक्तवधान के भाग IV में
ददए गए दकसी भी राज्य के नीक्तत क्तनदेिक तत्व को िागू करने वािी सभी क्तवक्तधयों को संरक्षण प्रदान दकया गया। हािांदक, आस
क्तवस्तार को ईच्चतम न्यायािय द्वारा क्तमनवाण क्तमल्स मामिे (1980) में ऄसंवैधाक्तनक और िून्य घोक्तषत कर ददया गया था।
 आस प्रकार आन दो क्तनदेिक क्तसिांतों को [ऄनुच्छेद 39(b) और ऄनुच्छेद 39(c) के ऄंतगणत] ऄनुच्छेद 14 (समानता का ऄक्तधकार)
और ऄनुच्छेद 19 (वाक् एवं ऄक्तभव्यक्ति की स्वतंत्रता) पर वरीयता दी गइ है। आसक्तिए क्तवकल्प (d) सही ईत्तर है।

7 www.visionias.in ©Vision IAS


Q 17.C
 संक्तवधान में नीक्तत के क्तनदेिक तत्वों का कोइ वगीकरण नहीं है। हािांदक,आनकी क्तवषय-वस्तु एवं ददिा के अधार पर आन्हें तीन
व्यापक श्रेक्तणयों जैसे दक समाजवादी, गांधीवादी एवं ईदार-बुक्तिजीवी में क्तवभि दकया जा सकता है।
 गांधीवादी क्तसिांत गांधीवादी क्तवचारधारा पर अधाररत हैं। ये राष्ट्रीय अंदोिन के दौरान गांधी द्वारा पुनस्थाणक्तपत योजनाओं का
प्रक्ततक्तनक्तधत्व करते हैं। गांधी के सपनों को साकार करने के क्तिए ईनके कु छ क्तवचारों को नीक्तत क्तनदेिक तत्वों के रूप में िाक्तमि
दकया गया है। ये राज्य से ऄपेक्षा करते हैं:
o ग्राम पंचायतों के गठन और ईन्हें अवश्यक िक्तियां प्रदान कर स्विासन की आकाआयों के रूप में कायण करने की िक्ति प्रदान
करना (ऄनुच्छेद 40)।
o ग्रामीण क्षेत्रों में व्यक्तिगत या सहकारी अधार पर कु िीर ईद्योगों को प्रोत्साहन देना (ऄनुच्छेद 43)।
o सहकारी सक्तमक्ततयों के स्वैक्तच्छक गठन, स्वायत्त संचािन, िोकतांक्तत्रक क्तनयंत्रण और व्यावसाक्तयक प्रबंधन को बढ़ावा देना
(ऄनुच्छेद 43 B)।
o ऄनुसक्तू चत जाक्तत (SCs), ऄनुसक्तू चत जनजाक्तत (STs) और समाज के ऄन्य कमजोर वगों के िैक्षक्तणक एवं अर्मथक क्तहतों को
प्रोत्साहन तथा आन्हें सामाक्तजक ऄन्याय एवं िोषण से सुरक्षा प्रदान करना (ऄनुच्छेद 46)।
o स्वास््य के क्तिए हाक्तनकारक निीिी दवाओं एवं मादक पेय के औषधीय प्रयोजनों से क्तभन्न ईपभोग पर प्रक्ततबंध अरोक्तपत
करना (ऄनुच्छेद 47)।
o गायों, बछडों और ऄन्य दुधारू एवं भार वाहक पिुओं के वध पर प्रक्ततबंध तथा ईनकी नस्िों में सुधार को प्रोत्साक्तहत करना
(ऄनुच्छेद 48)।
 काम पाने का, क्तिक्षा प्राप्त करने का और बेरोजगारी, बुढ़ापा, बीमारी और क्तनःििता की दिाओं में िोक सहायता प्राप्त करने के
ऄक्तधकार को सुक्तनक्तित करना (ऄनुच्छेद 41)। यह एक समाजवादी नीक्तत क्तनदेिक तत्व है।
 आसक्तिए क्तवकल्प (c) सही ईत्तर है।

Q 18.B
 ऄनुच्छेद 33, संसद को सिस्त्र बिों, ऄधण-सैन्य बिों, पुक्तिस बिों, खुदफया एजेंक्तसयों और समरूप बिों के सदस्यों के मूि
ऄक्तधकारों को प्रक्ततबंक्तधत या 'क्तनरस्त' करने का ऄक्तधकार देता है। आसक्तिए कथन 1 सही नहीं है।
 आस प्रावधान का ईद्देश्य ईनके कतणव्यों के समुक्तचत क्तनवणहन और ईनके बीच ऄनुिासन को बनाए रखना सुक्तनक्तित करना है।
 ऄनुच्छेद 33 के तहत क्तवक्तध क्तनमाणण का ऄक्तधकार के वि संसद को है न दक राज्य क्तवधानमंडिों को। संसद द्वारा क्तनर्ममत आस
प्रकार की दकसी भी क्तवक्तध को दकसी मूि ऄक्तधकार के ईल्िंघन के अधार पर दकसी न्यायािय में चुनौती नहीं दी जा सकती है।
 तदनुसार, संसद द्वारा सैन्य ऄक्तधक्तनयम (1950), नौसेना ऄक्तधक्तनयम (1950), वायु सेना ऄक्तधक्तनयम (1950), पुक्तिस बि
(ऄक्तधकारों पर क्तनषेध) ऄक्तधक्तनयम, 1966, सीमा सुरक्षा बि ऄक्तधक्तनयम, अदद को ऄक्तधक्तनयक्तमत दकया गया है।
 ये ईनकी ऄक्तभव्यक्ति की स्वतंत्रता, संगठन बनाने के ऄक्तधकार, श्रक्तमक संघों या राजनीक्ततक संगठनों के सदस्य बनने के ऄक्तधकार,
प्रेस के साथ संवाद करने के ऄक्तधकार, सावणजक्तनक बैठकों या प्रदिणनों में सक्तम्मक्तित होने के ऄक्तधकार अदद पर प्रक्ततबंध अरोक्तपत
करते हैं।
 ‘सिस्त्र बिों के सदस्य' की ऄक्तभव्यक्ति से ऄक्तभप्राय सिस्त्र बिों के ईन कमणचाररयों से है,जो सेना में नाइ, बढ़इ, मैकेक्तनक,
बावची, चौकीदार, बूि बनाने, दजी अदद का कायण करते हैं, जो गैर -िडाकू होते हैं। आसक्तिए कथन 2 सही है।
 ऄनुच्छेद 33 के ऄंतगणत ऄक्तधक्तनयक्तमत संसदीय क्तवक्तध, जहां तक मूि ऄक्तधकारों के प्रवतणन का संबंध है, कोिण-मािणि (सैन्य क्तवक्तध
के ऄंतगणत स्थाक्तपत ऄक्तधकरण) को ईच्चतम न्यायािय और ईच्च न्यायाियों के ररि क्षेत्राक्तधकार से ऄपवर्मजत कर सकती है।
Q 19.D
 1858 के ऄक्तधक्तनयम को भारत के िासन को बेहतर बनाने वािे ऄक्तधक्तनयम के रूप में भी जाना जाता है; आसने इस्ि आं क्तडया
कं पनी को समाप्त कर ददया और गवनणरों/सरकार, क्षेत्रों और राजस्व संबंधी िक्तियों को क्तब्ररिि राजिाही को स्थानांतररत कर
ददया। ऄक्तधक्तनयम की क्तविेषताएं:
o आसके तहत एक नए पद, ‘भारत का राज्य सक्तचव’ सृक्तजत दकया गया, क्तजसमें भारतीय प्रिासन पर पूणण ऄक्तधकार और
क्तनयंत्रण की िक्ति क्तनक्तहत थी। आसक्तिए कथन 1 सही है। राज्य सक्तचव क्तब्ररिि कै क्तबनेि का सदस्य था और ऄंततः क्तब्ररिि
संसद के प्रक्तत ईत्तरदायी था।

8 www.visionias.in ©Vision IAS


o आसने क्तनयंत्रण बोडण (बोडण ऑफ कं ट्रोि) एंड क्तनदेिक मंडि (कोिण ऑफ डायरे क्िसण) को समाप्त करके द्वैध िासन प्रणािी को
समाप्त कर ददया। आसक्तिए कथन 2 सही है।
o आसमें यह प्रावधान दकया गया दक भारत को महारानी के द्वारा एवं ईनके नाम पर िाक्तसत दकया जाएगा। आसके तहत
भारत के गवनणर-जनरि के पदनाम को बदिकर भारत का वायसराय कर ददया गया। वह (वायसराय) भारत में क्तब्ररिि
ताज का प्रत्यक्ष प्रक्ततक्तनक्तध था। आस प्रकार िॉडण कै लनग भारत के प्रथम वायसराय बने। आसक्तिए कथन 3 सही है।
o आसके तहत भारत के राज्य सक्तचव की सहायता के क्तिए भारत की 15 सदस्यीय पररषद की स्थापना की गयी। पररषद एक
परामिणदात्री क्तनकाय थी। पररषद का ऄध्यक्ष भारत के राज्य सक्तचव को बनाया गया।
o आसके तहत एक क्तनगक्तमत क्तनकाय के रूप में भारत सक्तचव की पररषद (secretary of state-in-council) का गठन दकया
गया, जो भारत और आंग्िैंड में मुकदमा करने में सक्षम थी और आस पर भी मुकदमा दकया जा सकता था।
Q 20.B
 िोकतंत्र सरकार का एक रूप है क्तजसमें िोगों को ऄपने िासी क्तवधायकों को चुनने का ऄक्तधकार होता है। वषण 1950 से वषण
2000 के मध्य 50 वषों के क्तिए िोकतंत्र और तानािाही का अकिन करने से ज्ञात होता है दक तानािाही में अर्मथक संवक्तृ ि की
दर थोडी ऄक्तधक है। अर्मथक क्तवकास कइ कारकों पर क्तनभणर करता है,जैसे: देि की जनसंख्या का अकार, वैक्तश्वक पररक्तस्थक्तत।
समग्रतः, हम यह नहीं कह सकते दक िोकतंत्र अर्मथक क्तवकास की गारं िी है। आसक्तिए कथन 1 सही नहीं है।
 गैर-िोकतांक्तत्रक िासन में प्रायः अंतररक सामाक्तजक मतभेदों को या तो नजरऄंदाज कर ददया जाता है या दबा ददया जाता है।
िोकतांक्तत्रक प्रणािी सामान्यतः क्तवक्तभन्न सामाक्तजक क्तवभाजनों को समायोक्तजत करने हेतु प्रदक्रयाओं का क्तवकास करती है, क्तजससे
सामंजस्यपूणण सामाक्तजक जीवन के क्तनमाणण की ऄक्तधक संभावना होती है। आसक्तिए कथन 2 सही है।
 िोकतंत्र को व्यक्ति की गररमा और स्वतंत्रता को बढ़ावा हेतु सरकार के दकसी ऄन्य रूप से ऄक्तधक बेहतर माना जाता है। भारत
में िोकतंत्र ने वंक्तचत और भेदभाव वािी जाक्ततयों के समान दजाण और समान ऄवसर के दावों को सुदढ़ृ दकया है। आसक्तिए कथन
3 सही है।
Q 21.D
 संक्तवधान सभा की प्रमुख सक्तमक्ततयां:
o संघ िक्ति सक्तमक्तत - जवाहरिाि नेहरू। आसक्तिए युग्म 3 सही सुमक्ते ित नहीं है।
o संघीय संक्तवधान सक्तमक्तत - जवाहरिाि नेहरू। आसक्तिए युग्म 1 सही सुमक्ते ित नहीं है।
o प्रांतीय संक्तवधान सक्तमक्तत - सरदार पिेि
o प्रारूप सक्तमक्तत - डॉ बी.अर. ऄम्बेडकर
o मूि ऄक्तधकार, ऄल्पसंख्यकों एवं जनजातीय और ऄपवर्मजत क्षेत्रों संबंधी सिाहकारी सक्तमक्तत - सरदार पिेि। आस सक्तमक्तत
के ऄंतगणत क्तनम्नक्तिक्तखत ईप सक्तमक्ततयां िाक्तमि थीं:
 मूि ऄक्तधकार ईप-सक्तमक्तत - जे.बी. कृ पिानी
 ऄल्पसंख्यक ईप-सक्तमक्तत - एच.सी. मुखजी
 पूवोत्तर सीमांत जनजातीय क्षेत्रों और असाम के ऄपवर्मजत और अंक्तिक रूप से ऄपवर्मजत क्षेत्रों संबंधी ईपसक्तमक्तत -
गोपीनाथ बोरदोिोइ
 ऄपवर्मजत और अंक्तिक रूप से ऄपवर्मजत क्षेत्रों (ऄसम के क्षेत्रों को छोडकर) संबंधी ईपसक्तमक्तत - ए.वी. ठक्कर
o प्रदक्रया क्तवषयक क्तनयमों संबंधी सक्तमक्तत - डॉ राजेंद्र प्रसाद
o राज्यों के क्तिए सक्तमक्तत (राज्यों के साथ वाताण के क्तिए सक्तमक्तत) - जवाहरिाि नेहरू
o संचािन सक्तमक्तत - डॉ राजेंद्र प्रसाद। आसक्तिए युग्म 2 सही सुमक्ते ित नहीं है।
Q 22.D
 के िवानंद भारती वाद (1973) में ईच्चतम न्यायािय ने संक्तवधान की मूि संरचना’ (या अधारभूत क्तविेषताओं) के एक नए
क्तसिांत का प्रक्ततपादन दकया। हािांदक ईच्चतम न्यायािय द्वारा यह पररभाक्तषत या स्पष्ट करना ऄभी िेष है दक संक्तवधान की
‘मूि संरचना ’ में क्या िाक्तमि है।
 क्तवक्तभन्न क्तनणणयों से, ‘संक्तवधान की संक्तवधान की मूि संरचना’ के तत्वों के रूप में क्तनम्नक्तिक्तखत को िाक्तमि दकया गया हैं:
o संक्तवधान की सवोच्चता,
o भारतीय राजव्यवस्था की संप्रभु, िोकतांक्तत्रक और गणतंत्रात्मक प्रकृ क्तत,
o संक्तवधान का धमणक्तनरपेक्ष स्वरूप,
o क्तवधाक्तयका, कायणपाक्तिका एवं न्यायपाक्तिका के बीच िक्तियों का पृथक्करण,

9 www.visionias.in ©Vision IAS


o संक्तवधान का संघीय स्वरुप,
o न्याक्तयक पुनर्मविोकन,
o व्यक्ति की स्वतंत्रता एवं गररमा,
o मौक्तिक ऄक्तधकारों एवं नीक्तत-क्तनदेिक तत्वों के बीच सौहादण और संतुिन,
o स्वतंत्र एवं क्तनष्पक्ष चुनाव,
o संक्तवधान में संिोधन हेतु संसद की सीक्तमत िक्ति अदद।
 आसक्तिए सभी कथन सही हैं।

Q 23.A
 सरकार न्यायाियों में दायर वादों में सवणप्रमुख वाददयों में से एक है और ईसे क्तवक्तधक सहायता प्रदान करने हेतु क्तवक्तभन्न क्तवक्तधक
ऄक्तधकाररयों की व्यवस्था की गइ है, क्तजनमें महान्यायवादी, सॉक्तिक्तसिर जनरि (महा न्यायक्तभकताण) और एक्तडिनि सॉक्तिक्तसिर
जनरि (ऄक्ततररि महा न्यायक्तभकताण) सक्तम्मक्तित हैं।
 कथन 1 सही है: संक्तवधान के ऄनुच्छेद 76 में भारत के महान्यायवादी पद का प्रावधान दकया गया है और यह देि का सवोच्च
क्तवक्तध ऄक्तधकारी होता है। भारत का सॉक्तिक्तसिर जनरि महान्यायवादी को ईसके अक्तधकाररक कतणव्यों और ईत्तरदाक्तयत्वों को
पूरा करने में सहायता करता है।
 कथन 2 सही नहीं है: भारत का सॉक्तिक्तसिर जनरि भारत के महान्यायवादी का ऄधीनस्थ ऄक्तधकारी होता है। यह देि का
दूसरा सवोच्च क्तवक्तध ऄक्तधकारी होता है, जो महान्यायवादी की सहायता करता है। सॉक्तिक्तसिर जनरि की सहायता भारत के
एक्तडिनि सॉक्तिक्तसिर जनरि द्वारा की जाती है।
 भारत का सॉक्तिक्तसिर जनरि क्तनजी क्तवक्तधक कायणवाक्तहयों(practice) से वंक्तचत नहीं दकया गया है। हािांदक कतणव्य की दकसी भी
जरििता और संघषण से बचने के क्तिए सॉक्तिक्तसिर जनरि पर कु छ सीमाएं ऄक्तधरोक्तपत की गयी हैं।
o ईसे भारत सरकार के क्तवरुि परामिण या सूचना नहीं देनी चाक्तहए।
o ईसे ऐसे मामिों में परामिण नहीं देना चाक्तहए क्तजसमें ईसे भारत सरकार के क्तिए परामिण देने या प्रस्तुत होने के क्तिए
अमंक्तत्रत दकया जाता है।
o ईसे भारत सरकार की ऄनुमक्तत के क्तबना अपराक्तधक ऄक्तभयोग में दकसी ऄक्तभयुि व्यक्ति का बचाव नहीं करना चाक्तहए।
o ईसे भारत सरकार की ऄनुमक्तत के क्तबना दकसी कं पनी या क्तनगम में क्तनदेिक के रूप में क्तनयुक्ति स्वीकार नहीं करनी चाक्तहए।

Q 24.C
 सरफे स-एनहांस्ड रमन स्पेक्ट्रोस्कोपी ऄथवा सरफे स-एनहांस्ड रमन स्कै िररग (SERS) एक सतह-संवेदी तकनीक है, जो
खुरदरी धातु की सतहों पर ऄविोक्तषत ऄणुओं द्वारा रमन प्रकीणणन को संवर्मधत करती है।
 SERS अणक्तवक पहचान और िक्षणों का वणणन करने की एक तकनीक है, यह ईन ऄणुओं के संवर्मधत रमन प्रकीणणन पर
क्तनभणर करती है, जो SERS-सदक्रय सतहों, जैसे दक नैनो–संरचना वािे सोने या चांदी के द्वारा या ईनके क्तनकि ऄविोक्तषत कर
क्तिए जाते हैं। सरफे स-एनहांस्ड रमन स्कै िररग (SERS) तकनीक को रासायक्तनक और जैक्तवक संवेदन ऄनुप्रयोगों के क्तिए एक
प्रभाविािी क्तवक्तध के रूप में जाना जाता है। आसक्तिए कथन 1 सही है।
 भारतीय प्रौद्योक्तगकी संस्थान ददल्िी के िोधकताणओं ने डेंगू के िीघ्र क्तनदान (Diagnosis) के क्तिए हाथ में धारण करने योग्य
एक सरफे स-एनहांस्ड रमन स्पेक्ट्रोस्कोपी (SERS)-अधाररत प्िेिफॉमण क्तवकक्तसत दकया है और यह एक घंिे के भीतर डेंगू के
परीक्षण के पररणाम (तीव्र क्तनदान) भी दे देता है। आसक्तिए कथन 2 सही है।
 ररवसण ट्रांसदक्रपिेस पोिीमरे ज़ चेन ररएक्िन (RT-PCR) का ईपयोग करके न्यूक्तक्िक एक्तसड का पता िगाने वािे परं परागत
नैदाक्तनक ईपकरण ऄक्तधक साध्य ईपकरण हैं तथा आनमें डेंगू के क्तनदान के क्तिए महंगे ईपकरण और ऄक्तभकमणकों की भी
अवश्यकता होती है।
 दकसी रोगी की सेहत क्तबगडने से रोकने के क्तिए डेंगू का िीघ्र क्तनदान महत्वपूणण है। आस ऄत्यक्तधक संवेदनिीि और सुक्तवधाजनक
ईपकरण में वायरि बीमाररयों का प्रारं क्तभक चरण में पता िगाने के क्तिए क्तवस्तृत ऄनुप्रयोग हैं। साथ ही यह एक घंिे के भीतर
जांच की ऄंक्ततम ररपोिण भी तैयार कर सकता है।
 यह ऄनुसंधान कायण ईद्योग भागीदार के रूप में न्यू एज आं स्ूमेंट्स एंड मैिेररयल्स प्राआवेि क्तिक्तमिेड के साथ क्तिक्षा मंत्रािय के
आं पैलक्िग ररसचण, आनोवेिन एंड िेक्नोिॉजी (IMPRINT) आं क्तडया कायणक्रम द्वारा क्तवत्त पोक्तषत था।

10 www.visionias.in ©Vision IAS


Q 25.C
 राष्ट्रीय ऄनुसूक्तचत जाक्तत अयोग (NCSC) संक्तवधान के ऄनुच्छेद 338 द्वारा स्थाक्तपत एक संवैधाक्तनक क्तनकाय है। पृथक राष्ट्रीय
ऄनुसूक्तचत जाक्तत अयोग (NCSC) वषण 2004 में ऄक्तस्तत्व में अया था। आसे 89वें संवैधाक्तनक संिोधन ऄक्तधक्तनयम, 2003 के
द्वारा राष्ट्रीय ऄनुसूक्तचत जनजाक्तत अयोग से पृथक कर ददया गया था। NCSC में ऄध्यक्ष, ईपाध्यक्ष और तीन ऄन्य सदस्य
िाक्तमि होते हैं। आनकी क्तनयुक्ति राष्ट्रपक्तत द्वारा ऄपने हस्ताक्षर और मुद्रा सक्तहत ऄक्तधपत्र द्वारा की जाती है।
 कथन 1 सही है: यह ऄनुसक्तू चत जाक्ततयों और अंग्ि-भारतीय समुदायों के सदस्यों को िोषण के क्तवरुि रक्षोपाय प्रदान करता है।
आसका ईद्देश्य संक्तवधान में प्रदत्त ईनके सामाक्तजक, िैक्षक्तणक, अर्मथक एवं सांस्कृ क्ततक क्तहतों को बढ़ावा देना और ईनकी रक्षा
करना है। यह ऄनुसूक्तचत जाक्तत और अंग्ि-भारतीयों के सामाक्तजक-अर्मथक क्तवकास की योजना प्रदक्रया में भी भाग िेता है।
 कथन 2 सही नहीं है: राष्ट्रपक्तत क्तनर्ददष्ट करता है दक प्रत्येक राज्य और संघ राज्य क्षेत्र में दकन जाक्ततयों को ऄनुसक्तू चत जाक्ततयों के
रूप में माना जाए। परं तु राष्ट्रपक्तत की ऄक्तधसूचना से दकसी भी जाक्तत का समावेिन या बक्तहष्करण के वि संसद के द्वारा दकया जा
सकता है। आस संदभण में, राष्ट्रीय ऄनुसूक्तचत जाक्तत अयोग के वि एक सिाहकार की भूक्तमका में है।
 कथन 3 सही है: NCSC, राष्ट्रपक्तत को वार्मषक रूप से और जब भी अवश्यक हो ऄनुसक्तू चत जाक्ततयों और अंग्ि-भारतीयों की
सुरक्षा ईपायों पर प्रक्ततवेदन प्रस्तुत करता है। यह ऄनुसूक्तचत जाक्ततयों के संरक्षण, कल्याण और क्तवकास से संबंक्तधत ऐसे ऄन्य
प्रकायों का क्तनष्पादन भी करता है क्तजन्हें राष्ट्रपक्तत क्तनर्ददष्ट करता है।

Q 26.D
 86वें संवध
ै ाक्तनक संिोधन ऄक्तधक्तनयम, 2002 द्वारा संक्तवधान के भाग IV-A में ऄनुच्छेद 51A के ऄंतगणत ग्यारहवें मूि कतणव्य को
ऄंत:स्थाक्तपत दकया गया था। आसके ऄनुसार यह प्रत्येक माता क्तपता या संरक्षक का कतणव्य है दक वह "छह वषण से चौदह वषण तक
की अयु वािे ऄपने, यथाक्तस्थक्तत, बािक या प्रक्ततपाल्य के क्तिए क्तिक्षा के ऄवसर प्रदान करे "। आसक्तिए कथन 1 सही है।
 86वें संवैधाक्तनक संिोधन ऄक्तधक्तनयम के द्वारा ‘क्तिक्षा के ऄक्तधकार को मूि ऄक्तधकार’ के रूप में संक्तवधान के भाग III में ऄनुच्छेद
21A के ऄंतगणत ऄंत:स्थाक्तपत दकया गया। आसमें कहा गया है दक "राज्य, छह वषण से चौदह वषण तक की अयु वािे सभी बच्चों के
क्तिए क्तनःिुल्क और ऄक्तनवायण क्तिक्षा देने का ऐसी रीक्तत में, जो राज्य क्तवक्तध द्वारा, ऄवधाररत करे , ईपबंध करे गा"। आसक्तिए कथन
2 सही है।
 आस संिोधन के द्वारा संक्तवधान में राज्य के नीक्तत क्तनदेिक तत्व के ऄंतगणत नए ऄनुच्छेद 45 को भी जोडा गया। आसमें ईपबंक्तधत
दकया गया है, "राज्य सभी बािकों के क्तिए छह वषण की अयु पूरी करने तक, प्रारं क्तभक बाल्यावस्था देख-रे ख और क्तिक्षा देने के
क्तिए ईपबंध करने का प्रयास करे गा”। आसक्तिए कथन 3 सही है।

Q 27.A
 रासायक्तनक हक्तथयार क्तनषेध संगठन (OPCW) एक स्वतंत्र, स्वायत्त ऄंतराणष्ट्रीय संगठन है, क्तजसका संयुि राष्ट्र के साथ कायणकारी
संबंध है। रासायक्तनक हक्तथयार क्तनषेध संगठन, वषण 1997 में िागू रासायक्तनक हक्तथयार कन्वेंिन (CWC) का कायाणन्वयन
क्तनकाय है। आसक्तिए कथन 1 सही है।
 OPCW ऄपने 193 सदस्य राष्ट्रों के साथ, रासायक्तनक हक्तथयारों को स्थायी और प्रमाक्तणक रूप से समाप्त करने के वैक्तश्वक
प्रयासों की क्तनगरानी करता है। आस कन्वेंिन के तहत वैक्तश्वक जनसंख्या के िगभग 98% भाग को संरक्षण प्राप्त है। स्वामी देिों
के द्वारा रासायक्तनक हक्तथयारों के 98% घोक्तषत भंडार को पूरी तरह से नष्ट कर ददया गया है।
 OPCW, हस्ताक्षरकताण देिों द्वारा कन्वेंिन के ऄनुपािन को सत्याक्तपत करने हेतु क्तनरीक्षण करने के क्तिए ऄक्तधकृ त है। वषण
2001 में OPCW और संयुि राष्ट्र के मध्य संबंध समझौते (Relationship Agreement) के तहत, OPCW के द्वारा दकए
गए क्तनरीक्षणों एवं ऄन्य कारण वाइयों के संबंध में महासक्तचव के माध्यम से संयुि राष्ट्र को सूचना दी जाती थी। आसक्तिए कथन 2
सही है।
 भारत रासायक्तनक हक्तथयार क्तनषेध संगठन का एक सदस्य है। हाि ही में, भारत के क्तनयंत्रक-महािेखापरीक्षक (CAG) को
रासायक्तनक हक्तथयारों के ईन्मूिन हेतु कायण करने वािे एक प्रक्ततक्तष्ठत ऄंतर सरकारी संगठन द्वारा बाह्य िेखा परीक्षक के रूप में
चुना गया है। आसक्तिए कथन 3 सही नहीं है।

11 www.visionias.in ©Vision IAS


Q 28.B
 कें द्रीय सतकण ता अयोग (CVC) एक िीषण सतकण ता संस्थान है जो कें द्रीय सरकार के ऄंतगणत सभी ऄपभ्रष्ट प्रयोगों या भ्रष्टाचार
पूणण कृ त्यों को रोकता है। यह दकसी भी कायणकारी प्राक्तधकरण से स्वतंत्र रूप से कायण करता है। यह कें द्र सरकार के ऄंतगणत क्तवक्तभन्न
प्राक्तधकरणों की सभी सतकण ता गक्ततक्तवक्तधयों की क्तनगरानी करता है। यह क्तवक्तभन्न कें द्रीय संगठनों को ईनके सतकण ता कायों की
योजना बनाने, क्तनष्पादन करने, सुधार और समीक्षा करने की सिाह भी देता है।
 कथन 1 सही नहीं है: कें द्रीय सतकण ता अयोग को वषण 1964 में कें द्र सरकार द्वारा पाररत कायणकारी प्रस्ताव के द्वारा स्थाक्तपत
दकया गया था। आस प्रकार, मूित: यह न तो संवैधाक्तनक क्तनकाय था और न ही सांक्तवक्तधक क्तनकाय था। वषण 2003 में, संसद द्वारा
पाररत कें द्रीय सतकण ता अयोग (CVC) ऄक्तधक्तनयम, 2003 के द्वारा आसे ‘सांक्तवक्तधक दजाण’ प्रदान दकया गया था।
 कथन 2 सही नहीं है: कें द्रीय सतकण ता अयोग को श्री के . संथानम की ऄध्यक्षता वािी भ्रष्टाचार क्तनवारण सक्तमक्तत की ऄनुिस
ं ा पर
स्थाक्तपत दकया गया था।
 कथन 3 सही है: आसका ऄक्तधकार क्षेत्र संघ के कायों के संबंध में सेवारत ऄक्तखि भारतीय सेवाओं के सदस्यों, कें द्र सरकार के
समूह ‘क’ के ऄक्तधकाररयों, सावणजक्तनक क्षेत्र के बैंकों में श्रेणी V और आसके ईपर के स्तर के ऄक्तधकाररयों अदद तक क्तवस्ताररत है।

Q 29.A
 हाक्तिया संदभण: संयि
ु राष्ट्र जनसंख्या कोष (UNFPA) द्वारा ‘माय बॉडी आज़ माय ओन’ (My Body is My Own) नामक
िीषणक से क्तवश्व जनसंख्या क्तस्थक्तत ररपोिण- 2021 जारी की गइ। आसक्तिए क्तवकल्प (a) सही ईत्तर है।
 ररपोिण में दैक्तहक स्वायत्तता (Bodily autonomy) पर ध्यान कें दद्रत दकया गया है। आसे लहसा के डर के क्तबना ऄथवा ऄपने िरीर
के बारे में दकसी ऄन्य व्यक्ति के द्वारा क्तिए क्तनणणय के क्तबना, स्वयं के द्वारा ऄपने िरीर के क्तवषय में क्तनणणय िेने की िक्ति और
कतणव्य के रूप में पररभाक्तषत दकया गया है।
 संयुि राष्ट्र जनसंख्या कोष संयि
ु राष्ट्र महासभा का एक सहायक ऄंग है। यह एक यौन एवं प्रजनन स्वास््य एजेंसी के रूप में
कायण करता है। आसका पररचािन वषण 1969 में िुरू दकया गया था।
 आसका िक्ष्य, एक ऐसे क्तवश्व का क्तनमाणण करना है जहां प्रत्येक गभाणवस्था आक्तच्छत हो, प्रत्येक प्रसव सुरक्तक्षत हो और प्रत्येक युवा
की प्रक्ततभा पूणण रूप से क्तवकक्तसत हो।

Q 30.B
 भारत का संक्तवधान बदिती पररक्तस्थक्ततयों और अवश्यकताओं के ऄनुसार स्वयं को समायोक्तजत करने के ईद्देश्य से संिोधन का
प्रावधान करता है। हािांदक, संक्तवधान के संिोधन के क्तिए क्तनधाणररत प्रदक्रया न तो क्तब्रिेन की भांक्तत असान है और न ही
ऄमेररका की भांक्तत जरिि है।
 संक्तवधान के संिोधन की प्रदक्रया ऄनुच्छेद 368 के तहत क्तनधाणररत की गइ है:
o संक्तवधान के संिोधन की प्रदक्रया का अरं भ संसद के दकसी भी सदन में आस प्रयोजन के क्तिए क्तवधेयक पुन:स्थाक्तपत करके ही
दकया जा सकता है, न दक दकसी राज्य क्तवधानमंडि में।
o क्तवधेयक को दकसी मंत्री या गैर-सरकारी सदस्य द्वारा प्रस्तुत दकया जा सकता है और आसके क्तिए राष्ट्रपक्तत की पूवण स्वीकृ क्तत
अवश्यक नहीं है। आसक्तिए कथन 1 सही नहीं है।
o क्तवधेयक को प्रत्येक सदन में क्तविेष बहुमत द्वारा पाररत करना ऄक्तनवायण है। आसका ऄथण यह है दक क्तवधेयक सदन के
ईपक्तस्थत और मत देने वािे सदस्यों के कम से कम दो-क्ततहाइ बहुमत द्वारा पाररत दकया जाना ऄक्तनवायण है। आसक्तिए कथन
2 सही नहीं है।
o प्रत्येक सदन द्वारा क्तवधेयक को ऄिग-ऄिग पाररत करना ऄक्तनवायण है। दोनों सदनों के बीच ऄसहमक्तत की क्तस्थक्तत में, दोनों
सदनों की संयि
ु बैठक में क्तवधेयक पर क्तवचार करने और पाररत करने का कोइ प्रावधान नहीं है। आसक्तिए कथन 3 सही है।
o यदद क्तवधेयक संक्तवधान के संघीय प्रावधानों में संिोधन के मुद्दे से संबंक्तधत है, तो आसे अधे राज्यों के क्तवधानमंडि द्वारा
साधारण बहुमत ऄथाणत, सदन के ईपक्तस्थत और मत देने वािे सदस्यों के द्वारा ऄनुसमर्मथत होना ऄक्तनवायण है।
Q 31.A
 मेड आन ऄमेररका िैक्स प्िान के तहत प्रॉदफि क्तिलफ्िग (profit shifting) और ऑफिोररग आं सरें िव (offshoring incentive)
को प्रबंक्तधत करने के क्तिए कॉरपोरे ि कर सुधारों को प्रस्ताक्तवत दकया गया है।

12 www.visionias.in ©Vision IAS


 ऄमेररकी राष्ट्रपक्तत जो बाआडेन को योजना से 15 वषों में 2.5 रट्रक्तियन ऄमेररकी डॉिर प्राप्त करने की ऄपेक्षा है।
 यह ऄमेररका द्वारा प्रारं भ सबसे महत्वपूणण प्रस्तावों में से एक है जो वैक्तश्वक न्यूनतम कॉपोरे ि कर (Global Minimum
Corporate tax) को ऄपनाने से संबंक्तधत है।
 यह "30-इयर रे स िू द बॉिम" (30-year race to the bottom) को ईििने का एक प्रयास है, क्तजसमें देिों ने बहुराष्ट्रीय
क्तनगमों (MNCs) को अकर्मषत करने के क्तिए कॉरपोरे ि कर दरों में कमी का सहारा क्तिया है।
 ऄमेररकी प्रस्ताव में वैक्तश्वक रूप से 21% न्यूनतम कॉरपोरे ि कर दर की पररकल्पना की गइ है, जो ईन देिों से अय पर छू ि को
रद्द करने के साथ है जो बहुराष्ट्रीय संचािन और मुनाफे को क्तवदेिों में स्थानांतररत करने के क्तिए एक न्यूनतम कर का क्तवधान
नहीं करते हैं।
 बेस आरोजन एंड प्रॉदफि क्तिलफ्िग (BEPS) क्या है?
o यह एक कर पररवजणन रणनीक्तत को संदर्मभत करता है, क्तजसमें कम कर का भुगतान करने के क्तिए कं पक्तनयां कर छू ि का
ऄनुक्तचत िाभ ईठाती हैं।
o कर पररवजणन रणनीक्तत के क्तहस्से के रूप में, बहुराष्ट्रीय कं पक्तनयां कम कर का भुगतान करने के क्तिए ऄपने िाभ को ईच्च कर
ऄक्तधकार क्षेत्रों से क्तनम्न कर ऄक्तधकार क्षेत्रों (िैक्स हैवन्स कं ट्री) में स्थानांतररत करती हैं।
o आससे ईच्च कर ऄक्तधकार क्षेत्रों के कर अधार का क्षरण होता है। क्तजससे ईच्च कर ऄक्तधकार क्षेत्रों में ऄत्यक्तधक राजस्व हाक्तन
होती है।
o अर्मथक सहयोग और क्तवकास संगठन (OECD) द्वारा वषण 2017 में प्रकाक्तित एक ररपोिण के ऄनुसार, BEPS वैक्तश्वक स्तर
पर िगभग 200 क्तबक्तियन ऄमेररकी डॉिर की कर हाक्तन के क्तिए ईत्तरदायी है। दोहरा कराधान बचाव संक्तध (DTAA),
राईं ड रट्रलपग (Round Tripping), ट्रीिी िॉलपग (Treaty Shopping) BEPS के कु छ साधन हैं क्तजनका दुरुपयोग होता
है।

Q 32.B
 हाक्तिया संदभण: नािो (ईत्तरी ऄििांरिक संक्तध संगठन-NATO) के साथ-साथ ऄमेररका ने ऄफगाक्तनस्तान से क्तनकिने का क्तनणणय
क्तिया है।
 नािो यूरोप और ईत्तरी ऄमेररका के देिों का एक सैन्य गठबंधन है। यह दो महाद्वीपों के 'पक्तिमी देिों' के मध्य एक संपकण प्रदान
करता है। आसक्तिए कथन 1 सही नहीं है।
 नािो के क्तसिांतों का पािन करने और ईत्तरी ऄििांरिक क्षेत्र की सुरक्षा में योगदान देने वािा कोइ भी यूरोपीय देि नािो का
सदस्य बन सकता है।
 नािो का कोइ क्तनणणय सभी 30 सदस्य देिों की सामूक्तहक आच्छा की ऄक्तभव्यक्ति होता है, क्योंदक सभी क्तनणणय सवणसम्मक्तत से क्तिए
जाते हैं।
 नािो के मुख्य कायण सामूक्तहक रक्षा (Collective Defense), संकि-प्रबंधन और सहकारी सुरक्षा हैं।
o ‘सामूक्तहक रक्षा’ के क्तसिांत के तहत एक या ऄक्तधक नािो सदस्य देिों पर अक्रमण सभी सदस्य देिों पर अक्रमण माना
जाता है। सामूक्तहक रक्षा के क्तसिांत को सवणप्रथम संयुि राज्य ऄमेररका पर हुए अतंकवादी हमिे (11 क्तसतंबर, 2001) के
बाद िागू दकया गया था। आसक्तिए कथन 2 सही है।

Q 33.D
 भारतीय पररषद ऄक्तधक्तनयम, 1861 भारत के संवैधाक्तनक और राजनीक्ततक आक्ततहास में एक महत्त्वपूणण ऄक्तधक्तनयम था। 1861 के
ऄक्तधक्तनयम की क्तविेषताएं क्तनम्नानुसार थीं:
o आसके द्वारा क्तवक्तध-क्तनमाणण की प्रदक्रया में भारतीय प्रक्ततक्तनक्तधयों को िाक्तमि करने की िुरुअत हुइ। आस प्रकार यह प्रावधान
दकया दक वायसराय कु छ भारतीयों को ऄपनी क्तवस्ताररत पररषद में गैर-सरकारी सदस्यों के रूप में नामांदकत कर सकता
था। 1862 में, तत्कािीन वायसराय, िॉडण कै लनग ने तीन भारतीयों - बनारस के राजा, परियािा के महाराजा और सर
ददनकर राव को ऄपनी क्तवधान पररषद में मनोनीत दकया। आसक्तिए कथन 2 सही है।
o आस ऄक्तधक्तनयम ने बंबइ और मद्रास प्रेसीडेंक्तसयों की क्तवधायी िक्तियों को बहाि कर क्तवकें द्रीकरण की प्रदक्रया की िुरुअत
की। आस प्रकार आसने कें द्रीयकरण की प्रवृक्तत्त को ईिि ददया जो 1773 के रे ग्युिेरिग एक्ि से िुरू हुइ थी और 1833 के

13 www.visionias.in ©Vision IAS


चािणर ऄक्तधक्तनयम के तहत ऄपने चरमोत्कषण पर पहुंच गइ थी। क्तवधायी िक्तियों के क्तवके न्द्रीकरण की आस नीक्तत के
पररणामस्वरूप 1937 में प्रांतों को िगभग पूणण अंतररक स्वायत्तता प्रदान की गइ। आसक्तिए कथन 3 सही है।
o आसमें बंगाि, ईत्तर-पक्तिमी सीमांत प्रांत (NWFP) और पंजाब के क्तिए नइ क्तवधान पररषदों की स्थापना के क्तिए भी
प्रावधान दकया गया, जो क्रमिः 1862, 1866 और 1897 में स्थाक्तपत की गइ थी।
o आसने वायसराय को पररषद में ऄक्तधक सुक्तवधाजनक कायण संचािन के क्तिए क्तनयम और अदेि बनाने की िक्तियां प्रदान की।
आसने िॉडण कै लनग द्वारा 1859 में िुरू की गइ 'पोिणफोक्तियो' प्रणािी को भी मान्यता दी। आसके ऄंतगणत, वायसराय की
पररषद के एक सदस्य को सरकार के एक या एक से ऄक्तधक क्तवभागों का प्रभारी बनाया गया था और ईसे पररषद की ओर
से ईसके क्तवभाग/क्तवभागों के मामिों पर ऄंक्ततम अदेि जारी करने का ऄक्तधकार था।
o आसने वायसराय को अपातकाि के दौरान क्तवधान पररषद की संस्तुक्तत के क्तबना, ऄध्यादेि जारी करने के क्तिए ऄक्तधकृ त
दकया। ऐसे ऄध्यादेि की समयावक्तध छह माह थी। आसक्तिए कथन 1 सही है।

Q 34.C
 मूि कतणव्य क्तवक्तध द्वारा प्रवतणनीय हैं। संसद दकसी भी मूि कतणव्य का ऄनुपािन न करने के क्तिए ईक्तचत दंड देने का प्रावधान कर
सकती है। वषण 1999 में जक्तस्िस वमाण सक्तमक्तत ने कु छ मूि कतणव्यों के कायाणन्वयन के क्तिए क्तवक्तधक प्रावधानों को क्तनधाणररत करने
दक ऄनुिसं ा की।
 मूि कतणव्य दकसी क्तवक्तध की संवध
ै ाक्तनक वैधता का क्तनधाणरण करने और ईनका परीक्षण करने में न्यायाियों की सहायता करते हैं।
वषण 1992 में, ईच्चतम न्यायािय ने क्तनणणय ददया दक दकसी क्तवक्तध की संवैधाक्तनकता का क्तनधाणरण करने में, यदद वह क्तवक्तध दकसी
मूि कतणव्य को कायाणक्तन्वत करती है तो ऐसी क्तवक्तध को ऄनुच्छेद 14 (क्तवक्तध के समक्ष समानता) और ऄनुच्छेद 19 (छह स्वतंत्रता)
के संबध
ं में युक्तियुि माना जाएगा और आस क्तवक्तध को ऄसंवध
ै ाक्तनक नहीं कहा जाएगा।
 मूि कतणव्य के वि नागररकों तक ही सीक्तमत हैं तथा क्तवदेक्तियों के क्तिए क्तवस्ताररत नहीं हैं। हािांदक, कु छ मूि ऄक्तधकार (ऄनुच्छेद
15, ऄनुच्छेद 16, ऄनुच्छेद 19, ऄनुच्छेद 29, ऄनुच्छेद 30 को छोडकर) क्तवदेक्तियों के क्तिए भी क्तवस्ताररत हैं।
 भारत की संप्रभुता, एकता और ऄखंडता की रक्षा करना और ईसे ऄक्षुण्ण रखना एक मूि कतणव्य है।
 आसक्तिए क्तवकल्प (c) सही ईत्तर है।

Q 35.D
 संयि
ु राज्य ऄमेररका के राष्ट्रपक्तत ने जिवायु संकि से क्तनपिने हेतु प्रमुख ऄथणव्यवस्थाओं द्वारा दकए जा रहे प्रयासों को गक्तत देने
के क्तिए 40 वैक्तश्वक नेताओं को "जिवायु पर नेताओं का क्तिखर सम्मेिन” (Leaders' Summit on Climate) के क्तिए
अमंक्तत्रत दकया है।
 जिवायु पर नेताओं का क्तिखर सम्मेिन, जिवायु संबंक्तधत सुदढ़ृ कारण वाइ की तात्काक्तिक अवश्यकता - एवं अर्मथक िाभों - को
रे खांदकत करे गा। यह ग्िासगो में नवंबर 2021 में अयोक्तजत होने वािे संयुि राष्ट्र जिवायु पररवतणन सम्मेिन (COP26) के
क्तिए एक महत्वपूणण मीि का पत्थर क्तसि होगा।
 जिवायु पर नेताओं का क्तिखर सम्मेिन और COP26 दोनों का प्रमुख िक्ष्य ईन प्रयासों को ईत्प्रेररत करना होगा जो वैक्तश्वक
ईष्मन के 1.5-क्तडग्री के िक्ष्य को पहुंच के भीतर रखते है। यह क्तिखर सम्मेिन ऐसे ईदाहरणों को भी प्रकि करे गा दक कै से बढ़ी
हुइ जिवायु महत्वाकांक्षा ऄच्छे वेतन वािी नौकररयों, ईन्नत नवीन प्रौद्योक्तगदकयों का सृजन करे गी, और सुभेद्य देिों को
जिवायु प्रभावों के ऄनुकूि बनाने में सहायता करे गी।
 यह क्तिखर सम्मेिन उजाण और जिवायु पर ऄमेररका के नेतृत्व वािे प्रमुख ऄथणव्यवस्था मंच को पुनसंयोक्तजत करे गा, जो वैक्तश्वक
ईत्सजणन और वैक्तश्वक GDP के िगभग 80 प्रक्ततित के क्तिए ईत्तरदायी 17 देिों को एक साथ िाता है।
 आसक्तिए क्तवकल्प (d) सही ईत्तर है।

Q 36.D
 हाक्तिया संदभण: वषण 2022 के ऄंत तक दकसानों की अय दोगुनी करने की दृक्तष्ट से सरकार को कृ क्तष-क्तनयाणत को बढ़ावा देने की
अवश्यकता है।

14 www.visionias.in ©Vision IAS


 आस पररकल्पना को साकार करने में सहायता करने के क्तिए, हमें ऄपने दकसानों को क्तवस्तृत बाजार तक पहुुँच प्रदान करने की
अवश्यकता है। आस संदभण में, कृ क्तष क्तनयाणत नीक्तत, 2018 ने "बेक आन आं क्तडया" पर जोर ददया, ऄथाणत मूल्य वधणन और प्रसंस्कृ त
कृ क्तष ईत्पादों पर नए क्तसरे से ध्यान कें दद्रत करना।
 1991 में LPG सुधारों के बाद से, भारत 2018-19 में िगभग 2.7 िाख करोड रूपये के क्तनयाणत के साथ कृ क्तष-ईत्पादों का
क्तनरं तर िुि क्तनयाणतक बना हुअ है। आसक्तिए कथन 1 सही है।
 खाद्यान्नों तथा फिों एवं सक्तब्जयों के सबसे बडे ईत्पादकों में से एक होने के बावजूद, कृ क्तष-वस्तुओं के वैक्तश्वक क्तनयाणत में भारत की
क्तहस्सेदारी मात्र 2% (9वीं रैं क) है। यह क्तनयाणत, थाइिैंड और आं डोनेक्तिया जैसे देिों की तुिना में भी बहुत कम है, क्तजनके पास
बहुत कम कृ क्तष योग्य भूक्तम है। आसक्तिए कथन 2 सही नहीं है।
 भारत की क्तनयाणत बास्के ि में मूि रूप से बासमती चावि और समुद्री ईत्पादों की प्रमुखता है। आसक्तिए कथन 3 सही है।
 कृ क्तष क्तनयाणत नीक्तत 2018 के व्यापक ईद्देश्यों और िक्ष्यों में िाक्तमि हैं:
o वषण 2022 तक कृ क्तष क्तनयाणत को दोगुना करना ऄथाणत मौजूदा िगभग 30+ क्तबक्तियन ऄमेररकी डॉिर से िगभग 60+
क्तबक्तियन ऄमेररकी डॉिर करना तथा ऄगिे कु छ वषों में 100 क्तबक्तियन ऄमेररकी डॉिर तक पहुंचाना है।
o हमारे क्तनयाणत बास्के ि, गंतव्यों में क्तवक्तवधता िाना तथा ईच्च-मूल्य और मूल्य वर्मधत कृ क्तष क्तनयाणत को बढ़ावा देना क्तजसमें
ख़राब होने वािे कृ क्तष ईत्पादों (perishables) पर ध्यान कें दद्रत दकया गया है।
o िीघ्राक्ततिीघ्र वैक्तश्वक मूल्य िृंखिा के साथ एकीकरण करके क्तवश्व कृ क्तष क्तनयाणत में भारत की क्तहस्सेदारी को दोगुना करने का
प्रयास करना।

Q 37.B
 दकसी राज्य में राष्ट्रपक्तत िासन िागू करना, क्तनम्नक्तिक्तखत पररक्तस्थक्ततयों में ईक्तचत होगा:
o जब अम चुनावों के बाद, क्तवधानसभा में दकसी भी दि को स्पष्ट बहुमत प्राप्त नहीं हुअ हो, ऄथाणत 'क्तत्रिंकु क्तवधानसभा' हो।
आसक्तिए कथन 1 सही नहीं है।
o जब बहुमत प्राप्त करने वािा दि सरकार बनाने से आनकार कर दे और राज्यपाि के समक्ष क्तवधानसभा में स्पष्ट बहुमत प्राप्त
करने वािा कोइ गठबंधन न हो।
o जब एक मंत्रािय क्तवधानसभा में ऄपनी हार के बाद त्यागपत्र दे दे और कोइ ऄन्य दि क्तवधानसभा में बहुमत क्तसि कर
सरकार बनाने के क्तिए आच्छु क न हो या सक्षम न हो। आसक्तिए कथन 2 सही नहीं है।
o जब राज्य सरकार द्वारा कें द्र सरकार के संवैधाक्तनक क्तनदेिों की ऄवहेिना की जाए।
o जहाुँ अंतररक ईच्छेदन हो, ईदाहरण के क्तिए, जब राज्य सरकार जानबूझकर संक्तवधान व कानून के क्तवरुि कायण करे या
एक लहसक क्तवद्रोह करे ।
o भौक्ततक क्तवखंडन, जहां सरकार आच्छापूवणक राज्य की सुरक्षा को खतरे में डािकर ऄपने संवध
ै ाक्तनक दाक्तयत्वों का क्तनवणहन
करने से आंकार कर दे।
 दकसी राज्य में राष्ट्रपक्तत िासन िागू करना, क्तनम्नक्तिक्तखत पररक्तस्थक्ततयों में ऄनुक्तचत होगा:
o जब मंक्तत्रमंडि त्यागपत्र दे दे ऄथवा क्तवधानसभा में बहुमत के ऄभाव के कारण बखाणस्त कर दी जाए और राज्यपाि एक
वैकक्तल्पक सरकार बनाने की संभावनाओं को जांचे क्तबना राष्ट्रपक्तत िासन िगाने की क्तसफाररि करता है।
o जब राज्यपाि क्तवधानसभा में एक मंक्तत्रमंडि के समथणन का स्वयं मूल्यांकन करे और मंक्तत्रमंडि को क्तवधानसभा के पिि पर
बहुमत साक्तबत करने की ऄनुमक्तत ददए क्तबना राष्ट्रपक्तत िासन िगाने की क्तसफाररि करे ।
o जब क्तवधानसभा में बहुमत प्राप्त करने वािे सत्ताधारी दि को िोकसभा के अम चुनावों में भारी हार का सामना करना पडे
जैसा दक 1977 और 1980 में हुअ था।
o अंतररक गडबडी क्तजससे कोइ अंतररक ईच्छेदन या भौक्ततक क्तवघिन/गडबडी ईत्पन्न न हो।
o राज्य में व्याप्त कु िासन ऄथवा मंक्तत्रपररषद के क्तवरुि भ्रष्टाचार के अरोप या राज्य की कठोर क्तवत्तीय अपदा की क्तस्थक्तत में।
आसक्तिए कथन 3 सही है।
o जहां क्तवनािकारी पररणामों के क्तिए ऄग्रणी ऄत्यावश्यक मामिे को छोडकर, राज्य सरकार को ऄपनी गिती सुधारने के
क्तिए पूवण चेतावनी नहीं दी गइ हो।
o जहां सत्ता पक्ष की ऄंतर-दिीय समस्याओं को सुिझाने के क्तिए या बाह्य ऄथवा ऄप्रासंक्तगक ईद्देश्य के क्तिए िक्ति का
ईपयोग संक्तवधान के क्तवरुि दकया जाए।

15 www.visionias.in ©Vision IAS


Q 38.D
 भारत की ऄंतररम सरकार का गठन वषण 1946 में हुअ था। आसमें क्तब्रिेन के वायसराय एवं कमांडर-आन-चीफ तथा ऑि आं क्तडया
मुक्तस्िम िीग के ऄन्य सदस्य भी िाक्तमि थे। आस सरकार का गठन क्तब्ररिि भारत के भारत और पादकस्तान में सत्ता हस्तांतरण
हेतु सहायता प्रदान करने के क्तिए दकया गया था। आसके सदस्यों में िाक्तमि थे:
 ईपाध्यक्ष के रूप में जवाहरिाि नेहरू
 सरदार पिेि
 डॉ. राजेंद्र प्रसाद
 डॉ. जॉन मथाइ
 जगजीवन राम
 सरदार बिदेव लसह
 सी. राजगोपािाचारी
 क्तियाकत ऄिी खान
 आसक्तिये क्तवकल्प (d) सही ईत्तर है।

Q 39.D
 िोकतांक्तत्रक बदिाव के साधन के रूप में संक्तवधान: आस त्य पर व्यापक सहमक्तत है दक संक्तवधान को ऄंगीकार करने का एक
बडा कारण है सत्ता को क्तनरं कुि होने से रोकना। राज्य की िक्ति का क्तवश्व भर का ऄनुभव बताता है दक ऄक्तधकांि राज्य कु छ
व्यक्ति ऄथवा समूहों के क्तहत को नुकसान पहुंचाने की ददिा में कायण करते हैं। यदद ऐसा है, तो हमें सत्ता के खेि के क्तनयमों को आस
तरह बनाना चाक्तहए दक राज्य की आस प्रवृक्तत्त पर ऄंकुि िगाया जा सके । संक्तवधान आन बुक्तनयादी क्तनयमों को प्रदान करता है
और आस प्रकार राज्यों को क्तनरं कुि बनने से रोकता है। आसक्तिए कथन 1 सही है।
 संक्तवधान हमें गहन सामाक्तजक पररवतणन िाने के क्तिए िांक्ततपूण,ण िोकतांक्तत्रक साधन भी प्रदान करता है। आसक्तिए कथन 3 सही
है।
 आसके ऄक्ततररि, औपक्तनवेक्तिक दासता में रहे िोगों के क्तिए संक्तवधान राजनीक्ततक अत्मक्तनणणयन का ईद्घोष है और आसका पहिा
वास्तक्तवक ऄनुभव भी है। आसक्तिए कथन 2 सही है।

Q 40.B
 भारत सरकार ऄक्तधक्तनयम, 1935: यह क्तब्ररिि संसद द्वारा पाररत दकया गया सबसे िंबा ऄक्तधक्तनयम था। आस ऄक्तधक्तनयम में
क्तब्ररिि भारत के प्रांतों और कु छ या सभी ररयासतों को क्तमिाकर एक ऄक्तखि भारतीय संघ की स्थापना का प्रावधान दकया गया
था। आसके मुख्य प्रावधान क्तनम्नानुसार थे:
o आस ऄक्तधक्तनयम ने प्रांतीय द्वैघ िासन व्यवस्था समाप्त कर दी और कें द्र में द्वैघ िासन व्यवस्था का िुभारं भ दकया। आसक्तिए
कथन 1 सही है।
o आसने भारतीय पररषद को समाप्त कर ददया और ईसके स्थान पर एक सिाहकारी क्तनकाय का गठन दकया।
o आसने क्तब्ररिि भारतीय क्षेत्रों और देसी ररयासतों को क्तमिाकर एक ऄक्तखि भारतीय संघ की स्थापना के क्तिए प्रावधान
दकया।
o आसमें ऄल्पसंख्यकों के क्तिए क्तवस्तृत सुरक्षोपायों और सुरक्षात्मक साधनों का प्रावधान दकया गया।
o आसने क्तब्ररिि संसद की सवोच्चता स्थाक्तपत की।
o आसने क्तवधाक्तयका के अकार में वृक्ति की, मताक्तधकार का क्तवस्तार दकया, तीन सूक्तचयों में क्तवषयों का क्तवभाजन दकया तथा
सांप्रदाक्तयक प्रक्ततक्तनक्तधत्व का क्तवस्तार/प्रक्ततधारण दकया।
o आसने बमाण का भारत से क्तवभाजन दकया।
o कु छ मामिों में दोनों सदनों की संयुि बैठक का प्रावधान दकया गया।
o आसने प्रांतीय स्वायत्तता का िुभारं भ दकया और प्रांतीय सरकारों के मंक्तत्रयों को प्रांतीय क्तवधाक्तयका के प्रक्तत ईत्तरदायी
बनाया।
 भारत सरकार ऄक्तधक्तनयम, 1919: क्तब्ररिि भारत के प्रांतों के क्तिए द्वैध िासन की व्यवस्था का िुभारं भ दकया गया। आसने भारत
के क्तब्ररिि प्रिासन की कायणकारी िाखा में िोकतांक्तत्रक क्तसिांत को पहिी बार अरं भ दकया। आस प्रकार, पहिी बार प्रांतीय स्तर
पर ईत्तरदायी सरकार को देखा गया था।
o आसने प्रांतीय क्तवधान सभाओं के अकार में वृक्ति की। ऄब िगभग 70% सदस्य क्तनवाणक्तचत दकए गए थे।

16 www.visionias.in ©Vision IAS


o आसने सांप्रदाक्तयक प्रक्ततक्तनक्तधत्व का क्तवस्तार दकया तथा आसमें क्तसखों, यूरोपीय और अंग्ि-भारतीयों को िाक्तमि दकया गया।
कु छ मक्तहिाएं भी मतदान कर सकती थी। वषण 1921 में, बॉम्बे (ऄब मुब
ं इ) और मद्रास (ऄब चेन्नइ) मक्तहिाओं को सीक्तमत
मताक्तधकार प्रदान करने वािे पहिे प्रांत बने। आसक्तिए कथन 2 सही नहीं है।
o आस ऄक्तधक्तनयम ने कें द्रीय क्तवधाक्तयका में क्तद्वसदनीय व्यवस्था स्थाक्तपत की।
 भारतीय पररषद ऄक्तधक्तनयम,1892: आस के माध्यम से कें द्रीय और प्रांतीय क्तवधान पररषदों में ऄक्ततररि (गैर-सरकारी) सदस्यों
की संख्या में वृक्ति की गइ, ककतु ईनमें सरकारी सदस्यों को बहुमत को बनाए रखा।
o आसने क्तवधान पररषदों को, पहिी बार बजि पर चचाण करने और कायणपाक्तिका के प्रश्नों का ईत्तर देने के क्तिए ऄक्तधकृ त
दकया। आसक्तिए कथन 3 सही है।
o आस ऄक्तधक्तनयम ने कें द्रीय और प्रांतीय क्तवधान पररषदों दोनों में गैर-सरकारी सदस्यों की क्तनयुक्ति के क्तिए एक सीक्तमत और
परोक्ष रूप से चुनाव का प्रावधान दकया। हािांदक, "चुनाव" पद का, आस ऄक्तधक्तनयम में प्रयोग नहीं दकया गया था।

Q 41.C
 मूि कतणव्य वाद योग्य नहीं हैं क्योंदक ईनके ईल्िंघन के क्तवरुि दकसी क्तवक्तधक दंड का प्रावधान नहीं है। आसके ऄिावा, संक्तवधान
भी न्यायाियों द्वारा ईनके दक्रयान्वयन का सीधा प्रावधान नहीं करता है। हािांदक, संसद ईपयुि क्तवधान द्वारा आनके
दक्रयान्वयन के क्तिए स्वतंत्र है। आसक्तिए कथन 2 सही है।
 राज्य की नीक्तत के क्तनदेिक तत्व भी वाद योग्य नहीं हैं। आनके ईल्िंघन पर भी आन्हें न्यायाियों द्वारा प्रवर्मतत नहीं दकया जा
सकता है। आसका तात्पयण यह है दक सरकार ईन्हें िागू करने के क्तिए बाध्य नहीं है। हािांदक, ऄनुच्छेद 37 में कहा गया है दक
राज्य की नीक्तत के क्तनदेिक तत्व देि के िासन में मूिभूत हैं और क्तवक्तध बनाने में आन तत्वों को िागू करना राज्य का कतणव्य
होगा। आसक्तिए कथन 3 सही है।
 मूि ऄक्तधकार वाद योग्य है और पीक्तडत व्यक्ति ऄपने ऄक्तधकारों के हनन के क्तिए न्यायाियों का रुख कर सकता है। ईच्चतम
न्यायािय द्वारा ईनका संरक्षण दकया जाता है और पीक्तडत व्यक्ति सीधे ईच्चतम न्यायािय जा सकता है। आसक्तिए कथन 1 सही
नहीं है।

Q 42.B
 बेनग
े ि नरलसह राव (बी. एन. राव) ने देखा दक "क्तनदेिक तत्वों की प्रकृ क्तत राज्य के ऄक्तधकाररयों के क्तिए नैक्ततक ईपदेिों के रूप
में है और ये सहज अिोचना के क्तिए खुिे हैं हािांदक संक्तवधान नैक्ततक ईपदेिों की जगह नहीं है। िेदकन ईनमें िैक्तक्षक मूल्य है
और ऄक्तधकांि अधुक्तनक संक्तवधान आस प्रकार के सामान्य तत्वों का प्रावधान करते हैं।" आसक्तिए क्तवकल्प (b) सही ईत्तर है।
 बेनग
े ि नरलसह राव संक्तवधान सभा के संवध
ै ाक्तनक सिाहकार थे।
 प्रख्यात न्यायक्तवद और कू िनीक्ततज्ञ, एि. एम. लसघवी के ऄनुसार क्तनदेिक तत्व संक्तवधान को जीवनदान देने वािी व्यवस्थाएं हैं।
 िी.िी.कृ ष्णामचारी ने क्तनदेिक तत्वों को ‘भावनाओं का एक स्थायी कू डाघर’ बताया।
 के . संथानम ने आिारा दकया दक क्तनदेिक तत्व एक संवैधाक्तनक िकराव पैदा करता है
o कें द्र और राज्यों के बीच,
o राष्ट्रपक्तत और प्रधानमंत्री के बीच, और
o राज्यपाि और मुख्यमंत्री के बीच।
 ईनके ऄनुसार, कें द्र आन तत्वों को िागू करने के क्तिए राज्यों को क्तनदेि दे सकता है और आनके ऄनुपािन न होने की क्तस्थक्तत में वह
राज्य सरकार को बखाणस्त कर सकता है।

Q 43.A
 कें द्र सरकार अवेदन प्राप्त होने पर दकसी भी व्यक्ति (ऄवैध प्रवासी न होने की क्तस्थक्तत में) को देिीयकरण प्रमाण-पत्र प्रदान कर
सकती है, यदद वह क्तनम्न ऄहणताएं रखता है:
o वह दकसी ऐसे देि की प्रजा या नागररक नहीं है जहां भारत के नागररक देिीयकरण द्वारा ईस देि की प्रजा या नागररक
बनने से ईस देि की क्तवक्तध या प्रथा द्वारा क्तनवाररत है।
o यदद वह दकसी देि का नागररक है, तो वह भारतीय नागररकता के क्तिए ऄपना अवेदन स्वीकार होने की दिा में ईस देि
की नागररकता त्यागने का वचन देता है। आसक्तिए कथन 1 सही है।
17 www.visionias.in ©Vision IAS
o अवेदन की तारीख से ऄव्यवक्तहत पूवण बारह मास की कािावक्तध भर वह या तो भारत में क्तनवासी रहा है या भारत में की
दकसी सरकार की सेवा में रहा है ऄथवा भागतः क्तनवासी रहा है और भागतः सेवा में रहा है।
o बारह मास की ईि कािावक्तध पूवण के चौदह वषों के दौरान वह ऐसी कािावक्तधयों के क्तिए जो कु ि क्तमिाकर ग्यारह वषण से
कम नहीं है या तो भारत में क्तनवासी रहा या भारत में की दकसी सरकार की सेवा में रहा है ऄथवा भागतः क्तनवासी रहा है
और भागतः सेवा में रहा है। आसक्तिए कथन 3 सही नहीं है।
o वह ऄच्छे िीि का है।
o वह संक्तवधान की ऄष्टम् ऄनुसच
ू ी में क्तवक्तनर्ददष्ट दकसी भाषा का यथायोग्य ज्ञान रखता है। आसक्तिए कथन 2 सही है।
o देिीयकरण का प्रमाणपत्र ईसे ऄनुदत्त दकए जाने की दिा में वह भारत में क्तनवास करने या भारत में दकसी सरकार के
ऄधीन ऄथवा दकसी ऐसे ऄन्तराणष्ट्रीय संगठन के ऄधीन क्तजसका भारत सदस्य है या भारत में स्थाक्तपत दकसी सोसाआिी,
कं पनी या व्यक्ति क्तनकाय के ऄधीन सेवा में प्रवेि करने या सेवा में बने रहने का अिय रखता है।

Q 44.B
 भारत के संक्तवधान के ऄनुच्छेद 148 के ऄंतगणत भारत का क्तनयंत्रक-महािेखापरीक्षक (CAG) के पद का प्रावधान दकया गया है।
वह कें द्र और राज्य दोनों स्तरों पर क्तवत्तीय व्यवस्था को क्तनयंक्तत्रत करता है। यही कारण है दक आसे िोक क्तवत्त के संरक्षक के रूप में
जाना जाता है।
 क्तवकल्प 1 सही है: CAG क्तवक्तभन्न प्रकार के िेखा परीक्षा करता है जैसे क्तवक्तधक एवं क्तवक्तनयामक िेखा परीक्षा जो CAG के क्तिए
बाध्यकारी होती है जबदक औक्तचत्य िेखा परीक्षा क्तववेकाधीन होती है। औक्तचत्य िेखा परीक्षा के ऄंतगणत CAG सरकारी व्यय की
तकण संगतता, सत्यता और क्तमतव्यक्तयता की जांच करता है। मंत्राियों के िेखा परीक्षण से ईत्पन्न रिप्पक्तणयों को ऄिग-ऄिग
प्रक्ततवेदनों में िाक्तमि दकया जाता है।
 क्तवकल्प 2 सही है: सावणजक्तनक क्तनगमों के िेखा परीक्षण में CAG की भूक्तमका को क्तनम्नक्तिक्तखत तीन कोरियों के ऄंतगणत देखा जा
सकता है-
o कु छ क्तनगमों की िेखा परीक्षा पूरी तरह से एवं प्रत्यक्ष तौर पर CAG द्वारा की जाती है, जैसे एयर आं क्तडया, तेि एवं
प्राकृ क्ततक गैस क्तनगम क्तिक्तमिेड।
o कु छ क्तनगमों जैसे के न्द्रीय भण्डारण क्तनगम की िेखा परीक्षा क्तनजी िेखा परीक्षकों के द्वारा की जाती है और CAG पूरक
िेखा परीक्षा कर सकती है।
o कु छ क्तनगमों जैसे भारतीय स्िेि बैंक की िेखा परीक्षा पूरी तरह से क्तनजी िेखा परीक्षकों द्वारा की जाती है।
 क्तवकल्प 3 सही नहीं है: CAG राज्य सहकारी सक्तमक्ततयों के िेखाओं का िेखा परीक्षण नहीं करता है। राज्य क्तवधानमंडि
सहकारी सक्तमक्ततयों के िेखाओं के िेखा-परीक्षण के क्तिए प्रावधान कर सकते हैं। सहकारी सक्तमक्तत के सामान्य क्तनकाय द्वारा
क्तनयुि एक िेखा परीक्षक द्वारा प्रत्येक सहकारी सक्तमक्तत का िेखा परीक्षण दकया जाएगा।

Q 45. C
 राष्ट्रीय मानव ऄक्तधकार अयोग (NHRC) एक स्वतंत्र वैधाक्तनक क्तनकाय है। आसकी स्थापना मानव ऄक्तधकार संरक्षण ऄक्तधक्तनयम,
1993 के ऄंतगणत की गइ है। यह भारत में मानव ऄक्तधकारों की क्तनगरानी करता है, ऄथाणत, यह संक्तवधान के साथ-साथ
ऄंतराणष्ट्रीय सम्मेिनों द्वारा गारं िीकृ त जीवन, स्वतंत्रता, समानता और व्यक्ति की गररमा से संबंक्तधत ऄक्तधकारों की क्तनगरानी
करता है।
 राष्ट्रीय मानव ऄक्तधकार अयोग की संरचना:
o ऄध्यक्ष- जो ईच्चतम न्यायािय के मुख्य न्यायाधीि या ईच्चतम न्यायािय के न्यायाधीि रहे हो।
o दो सदस्य- क्तजन्हें मानव ऄक्तधकारों का ज्ञान हो, राष्ट्रीय मानव ऄक्तधकार अयोग के सदस्यों के रूप में क्तनयुि दकए जायेंगे।
o क्तवक्तभन्न अयोगों के ऄध्यक्ष जैसे दक-
 राष्ट्रीय ऄनुसूक्तचत जाक्तत अयोग
 राष्ट्रीय ऄनुसूक्तचत जनजाक्तत अयोग
 राष्ट्रीय मक्तहिा अयोग
 राष्ट्रीय क्तपछडा वगण अयोग
 राष्ट्रीय बाि ऄक्तधकार संरक्षण अयोग
 ददव्यांगजन व्यक्तियों के क्तिए मुख्य अयुि

18 www.visionias.in ©Vision IAS


 कथन 1 सही है: राष्ट्रीय मानव ऄक्तधकार अयोग या तो स्वत: संज्ञान िे सकता है या याक्तचका प्राप्त करने के ईपरांत सिस्त्र बिों
द्वारा मानव ऄक्तधकारों के ईल्िंघन के संबध
ं में कें द्र सरकार से ररपोिण मांग सकता है। ररपोिण प्राप्त होने के पिात, वह या तो
मामिे को अगे नहीं बढ़ा सकता है या मामिे के ऄनुसार सरकार को ऄपनी ऄनुिस
ं ा प्रस्तुत कर सकता है।
 कथन 2 सही है: सरकार तीन माह या अयोग द्वारा ऄनुमत दकसी ऄन्य समयावक्तध के भीतर, ऄनुिस
ं ाओं पर की गइ कारण वाइ
के संदभण में अयोग को सूक्तचत करे गी। तत्पिात अयोग सरकार को की गइ ऄनुिस
ं ाओं और सरकार द्वारा ऄनुिंसाओं पर की
गइ कारण वाइ के बारे में ररपोिण प्रकाक्तित करे गा।

Q 46. B
 ऄनुच्छेद 5 से 8 के ऄंतगणत भारतीय संक्तवधान के वि ईन िोगों की पहचान करता है जो संक्तवधान िागू होने के समय नागररक
बने। संक्तवधान िागू होने के बाद, नागररकता प्राप्त करने के तरीकों को संसद पर छोड ददया गया। तदनुसार, संसद द्वारा
नागररकता ऄक्तधक्तनयम 1955 ऄक्तधक्तनयक्तमत दकया गया था। आसक्तिए कथन 1 सही नहीं है।
 संक्तवधान के ऄनुच्छेद 8 के ऄंतगणत कोइ व्यक्ति जो या क्तजसके माता या क्तपता में से कोइ ऄथवा क्तपतामह या क्तपतामही या
मातामह या मातामही में से कोइ ऄक्तवभाक्तजत भारत में जन्मा हों िेदकन वह भारत के बाहर दकसी देि में मामूिी तौर पर
क्तनवास कर रहा हो, पंजीकरण के जररए वह भारतीय नागररक बन जाएगा।
 ऄनुच्छेद 5 से 8 के ऄंतगणत प्रदत्त संवैधाक्तनक प्रावधान, नागररकता के संदभण में वणणन करता है:
o एक व्यक्ति जो भारत का मूि क्तनवासी है;
o पादकस्तान से प्रव्रजन करने वािे व्यक्ति ; आसक्तिए कथन 3 सही है।
o पादकस्तान को प्रव्रजन करने वािे व्यक्ति जो बाद में वापस िौि अए, और
o भारतीय मूि के व्यक्ति जो भारत के बाहर रह रहे हो। आसक्तिए कथन 2 सही है।

Q 47. A
 42वें संिोधन ऄक्तधक्तनयम, 1976 के द्वारा राज्य की नीक्तत के क्तनदेिक तत्त्व (DPSPs) की मूि सूची में चार नए क्तनदेिक तत्व
जोडे गए। वे हैं:
o बच्चों के स्वस्थ क्तवकास के क्तिए ऄवसरों को सुरक्तक्षत करना [ऄनुच्छेद 39(f)]।
o समान न्याय को बढ़ावा देना और गरीबों को क्तनःिुल्क क्तवक्तधक सहायता ईपिब्ध कराना (ऄनुच्छेद 39A)।
o ईद्योगों के प्रबंध में कमणकारों की भागीदारी के क्तिए कदम ईठाना (ऄनुच्छेद 43A)।
o पयाणवरण का संरक्षण तथा संवधणन और वन तथा वन्यजीवों की रक्षा (ऄनुच्छेद 48A)।
 44वें संिोधन ऄक्तधक्तनयम, 1978 के द्वारा राज्य की नीक्तत के क्तनदेिक तत्व में एक और क्तनदेिक तत्व जोडा गया, जो राज्य से
ऄपेक्षा रखता है दक वह अय, प्रक्ततष्ठा एवं सुक्तवधाओं और ऄवसरों की ऄसमानता को समाप्त करे (ऄनुच्छेद 38)।
 86वें संिोधन ऄक्तधक्तनयम,2002 में ऄनुच्छेद 45 के क्तवषय-वस्तु को बदिा गया और प्राथक्तमक क्तिक्षा को ऄनुच्छेद 21A के
ऄंतगणत एक मूि ऄक्तधकार बनाया गया। संिोक्तधत क्तनदेिक तत्व में राज्य से यह ऄपेक्षा की गइ है दक वह छोिे बच्चों को छह वषण
की अयु पूणण होने तक बाल्यावस्था देखरे ख और क्तिक्षा का ईपबंध करे ।
 97वें संिोधन ऄक्तधक्तनयम, 2011 द्वारा सहकारी सक्तमक्ततयों से संबंक्तधत एक नया क्तनदेिक तत्व जोडा गया। आसमें राज्य से यह
ऄपेक्षा की गइ है दक वे सहकारी सोसाआरियों की स्वैक्तच्छक क्तवरचना, स्विासी कायणकरण, िोकतांक्तत्रक क्तनयंत्रण तथा वृक्तत्तक
प्रबंधन का संवधणन करें (ऄनुच्छेद 43B)।
 ऄनुच्छेद 47 के ऄंतगणत पोषाहार स्तर और जीवन स्तर को उुँचा करना तथा िोक स्वास््य का सुधार करना, संक्तवधान को
ऄपनाने के बाद से ही है और आसे संिोधन के माध्यम से नहीं जोडा गया है।
 आसक्तिए क्तवकल्प (a) सही ईत्तर है।
Q 48. B
 कै क्तबनेि क्तमिन योजना द्वारा सुझाए गए प्रस्तावों के ऄंतगणत नवंबर 1946 में संक्तवधान सभा का गठन दकया गया था। संक्तवधान
सभा में सदस्यों की कु ि संख्या 389 होनी थी। आनमें से 296 सीिें क्तब्ररिि भारत और 93 सीिें देिी ररयासतों को अवंरित की
जानी थीं।

19 www.visionias.in ©Vision IAS


 संक्तवधान सभा अंक्तिक रूप से क्तनवाणक्तचत और अंक्तिक रूप से नामांदकत क्तनकाय थी। आसके ऄिावा, सदस्यों का चयन ऄप्रत्यक्ष
रूप से प्रांतीय व्यवस्थाक्तपका के सदस्यों द्वारा दकया जाना था।
 संक्तवधान क्तनमाणण और सामान्य क्तवक्तधयों को िागू करने के ऄिावा, संक्तवधान सभा ने क्तनम्नक्तिक्तखत कायण भी दकए:
o आसने 22 जुिाइ, 1947 को राष्ट्रीय ध्वज को ऄपनाया।
o आसने मइ, 1949 में राष्ट्रमंडि में भारत की सदस्यता की पुक्तष्ट की।
o आसने 24 जनवरी, 1950 को राष्ट्रगान को ऄपनाया।
o आसने 24 जनवरी, 1950 को राष्ट्रीय गीत को ऄपनाया।
o आसने 24 जनवरी, 1950 को डॉ. राजेंद्र प्रसाद को भारत के पहिे राष्ट्रपक्तत के रूप में क्तनवाणक्तचत दकया।
 आसक्तिए क्तवकल्प (b) सही ईत्तर है।

Q 49. A
 हमारे संक्तवधान का भाग III सभी नागररकों को छह मूि ऄक्तधकारों की गारं िी देता है:
o समता का ऄक्तधकार (ऄनुच्छेद 14-18)।
o स्वातंत्र्य ऄक्तधकार/ स्वतंत्रता का ऄक्तधकार (ऄनुच्छेद 19-22)।
o िोषण के क्तवरुि ऄक्तधकार (ऄनुच्छेद 23-24)।
o धमण की स्वतंत्रता का ऄक्तधकार (ऄनुच्छेद 25-28)।
o संस्कृ क्तत और क्तिक्षा संबंधी ऄक्तधकार (ऄनुच्छेद 29-30)।
o संक्तवधाक्तनक ईपचारों का ऄक्तधकार (ऄनुच्छेद 32)।
 आनमें से कु छ मूि ऄक्तधकार क्तविेष रूप से भारत के नागररकों को ईपिब्ध हैं ककतु क्तवदेक्तियों के क्तिए ईपिब्ध नहीं हैं। वे हैं;
o ऄनुच्छेद 15: धमण, मूिवंि, जाक्तत, लिग या जन्म स्थान के अधार पर क्तवभेद का प्रक्ततषेध।
o ऄनुच्छेद 16: िोक क्तनयोजन के क्तवषय में ऄवसर की समानता।
o ऄनुच्छेद 19: स्वतंत्रता के संबंध में छह ऄक्तधकारों का संरक्षण;
(i) वाक् एवं ऄक्तभव्यक्ति की स्वतंत्रता
(ii) सम्मेिन करने,
(iii) संघ बनाने,
(iv) संचरण का ऄक्तधकार,
(v) क्तनवास करने का ऄक्तधकार, और
(vi) वृक्तत्त का ऄक्तधकार।
o ऄनुच्छेद 29: ऄल्पसंख्यक-वगों के क्तहतों का संरक्षण।
o ऄनुच्छेद 30: ऄल्पसंख्यक-वगों को क्तिक्षण संस्थाओं की स्थापना और प्रिासन करने का ऄक्तधकार।
 मानव दुव्याणपार और बिात् श्रम का प्रक्ततषेध, क्तवदेक्तियों और भारतीय नागररकों दोनों के क्तिए ईपिब्ध है।
 आसक्तिए क्तवकल्प (a) सही ईत्तर है।

Q 50.B
 ऄनुच्छेद 1 संघ के नाम और राज्यक्षेत्र से संबंक्तधत है:
o ऄनुच्छेद 1 (1) में कहा गया है दक भारत ऄथाणत आंक्तडया, राज्यों का एक संघ होगा।
o ऄनुच्छेद 1 (2) में कहा गया है दक राज्य और ईनके राज्यक्षेत्र में वे होंगे, जो पहिी ऄनुसच
ू ी में क्तवक्तनर्ददष्ट हैं।
o ऄनुच्छेद 1(3) में कहा गया है दक भारत के क्षेत्र में-
o राज्यों के राज्यक्षेत्र;
o पहिी ऄनुसच
ू ी में क्तवक्तनर्ददष्ट संघ राज्यक्षेत्र और
o ऐसे राज्यक्षेत्र जो ऄर्मजत दकए जाएंग,े समाक्तवष्ट होंगे

20 www.visionias.in ©Vision IAS


 आस प्रकार, 'भारत का क्षेत्र' 'राज्यों के संघ' की तुिना में एक व्यापक ऄक्तभव्यक्ति है क्योंदक ईत्तरवती में के वि राज्य िाक्तमि हैं,
जबदक पूवव
ण ती में न के वि राज्य, बक्तल्क संघ िाक्तसत प्रदेि और ऐसे क्षेत्र भी िाक्तमि हैं, क्तजन्हें भारत सरकार द्वारा भक्तवष्य में
ऄक्तधगृहीत दकया जा सकता है। आसक्तिए के वि क्तवकल्प 1 सही है। राज्य, संघीय प्रणािी के सदस्य हैं और कें द्र के साथ िक्तियों
के क्तवतरण में क्तहस्सेदार हैं। दूसरी ओर संघ िाक्तसत प्रदेि और कें द्र सरकार द्वारा ऄक्तधगृहीत प्रदेि में सीधे कें द्र सरकार का
प्रिासन होता है।

Q 51.A
 समान क्तसक्तवि संक्तहता वह है, जो देि भर के क्तिए एक समान क्तवक्तध प्रदान करे गी, जो सभी धार्ममक समुदायों के क्तिए ईनके
व्यक्तिगत मामिों जैसे दक क्तववाह, तिाक, पैतृक सम्पक्तत, गोद िेने/दत्तक गृहण अदद में िागू होगी। संक्तवधान का ऄनुच्छेद 44
यह क्तनर्ददष्ट करता है दक राज्य, भारत के समस्त राज्य क्षेत्र के नागररकों के क्तिए एक समान क्तसक्तवि संक्तहता सुक्तनक्तित करने का
प्रयास करे गा। आसक्तिए कथन 1 सही है।
 ऄनुच्छेद 44, राज्य के नीक्तत क्तनदेिक तत्वों में से एक है। जैसा दक ऄनुच्छेद 37 में पररभाक्तषत दकया गया है, ये प्रकृ क्तत में गैर-
न्यायोक्तचत हैं (दकसी भी न्यायािय द्वारा िागू करने योग्य नहीं हैं) दकन्तु आसमें क्तनधाणररत क्तसिांत िासन में मूिभूत हैं।
 भारत में गोवा एकमात्र ऐसा राज्य है, जहाुँ पर धमण, लिग, जाक्तत की परवाह दकए क्तबना एक समान क्तसक्तवि संक्तहता है। गोवा में
एक समान पाररवाररक क्तवक्तधयां हैं। हाि ही में भारत के मुख्य न्यायाधीि (CJI) ने गोवा की समान क्तसक्तवि संक्तहता की
सराहना की है। आसक्तिए कथन 2 सही नहीं है।

Q 52.B
 भारतीय संक्तवधान की प्रस्तावना 13 ददसंबर 1946 को जवाहरिाि नेहरू द्वारा प्रारूक्तपत एवं प्रस्तुत दकए गए ’ईद्देश्य प्रस्ताव’
पर अधाररत है। आसे संक्तवधान सभा द्वारा 22 जनवरी 1947 को सवणसम्मक्तत से ऄंगीकृ त दकया गया था। आसक्तिए कथन 2 सही
है।
 प्रस्तावना को ऄंगीकृ त करने के पिात्, आसे संक्तवधान का भाग माना गया है। आस प्रकार ईच्चतम न्यायािय द्वारा प्रस्तावना पर
ददए गए वतणमान ऄक्तभमत आसके संस्थापक जनकों के ऄक्तभमतों के ऄनुरूप हैं।
 संक्तवधान सभा द्वारा संक्तवधान के ऄन्य सभी प्रावधान िागू दकए जाने के पिात् प्रस्तावना को िागू दकया गया था। आसक्तिए
कथन 1 सही नहीं है।
 प्रस्तावना को ऄंत में िागू करने का कारण यह सुक्तनक्तित करना था दक संक्तवधान के ऄनुरूप हो, क्तजसे संक्तवधान सभा द्वारा
ऄंगीकृ त दकया गया था।
Q 53.B
 स्वास््य एवं पररवार कल्याण मंत्रािय (H&FW) द्वारा आस क्षेत्र के क्तवक्तभन्न क्तहतधारकों और क्तविेषज्ञों के साथ कइ परामिण के
पिात् दुिणभ रोगों के क्तिए राष्ट्रीय नीक्तत 2021 को ऄंक्ततम रूप ददया गया है।
 दुिभ
ण रोगों के क्तिए राष्ट्रीय नीक्तत की मुख्य क्तविेषताएं:
o 'दुिभ
ण रोगों' को नीक्तत में पररभाक्तषत नहीं दकया गया है, दकन्तु क्तविेषज्ञ की राय के अधार पर आन्हें तीन समूहों में वगीकृ त
दकया गया है। आसक्तिए कथन 1 सही नहीं है।
o दुिभ
ण रोग नीक्तत का िक्ष्य एक राष्ट्रीय सहायता संघ (National Consortium) की मदद से स्वदेिी ऄनुसंधान पर ध्यान
कें दद्रत करने के साथ दुिभ
ण रोगों के ईपचार की ईच्च िागत को कम करना है। राष्ट्रीय सहायता संघ स्वास््य ऄनुसंधान
क्तवभाग, स्वास््य और पररवार कल्याण मंत्रािय के संयोजक के रूप में स्थाक्तपत दकया जाएगा। ऄनुसंधान एवं क्तवकास तथा
औषक्तधयों के स्थानीय ईत्पादन पर ऄक्तधक ध्यान देने से दुिभ
ण रोगों के ईपचार में होने वािा व्यय कम होगा।
o नीक्तत में दुिभ
ण रोगों की एक राष्ट्रीय स्तर की ऄस्पताि अधाररत रक्तजस्ट्री के क्तनमाणण की पररकल्पना भी की गइ है, क्तजससे
दुिभ
ण रोगों की पररभाषा और देि के भीतर दुिणभ रोगों से संबंक्तधत ऄनुसंधान एवं क्तवकास के क्तिए पयाणप्त डेिा ईपिब्ध हो।
o आस नीक्तत के तहत स्वास््य एवं कल्याण के न्द्रों और क्तजिा प्रारं क्तभक हस्तक्षेप के न्द्रों (DEICs) जैसी प्राथक्तमक और क्तद्वतीयक
स्वास््य देखभाि ऄवसंरचना और ऄक्तधक जोक्तखम वािे मरीजों के क्तिए परामिण के माध्यम से त्वररत जांच एवं बचाव पर
भी ध्यान कें दद्रत दकया गया है।
o ऐसे दुिभ
ण रोगों, क्तजनके क्तिए एकमुश्त ईपचार की अवश्यकता होती है, के ईपचार के क्तिए राष्ट्रीय अरोग्य क्तनक्तध की
ऄम्ब्रेिा योजना के तहत 20 िाख रुपये तक के क्तवत्तीय समथणन के प्रावधान का प्रस्ताव है।
21 www.visionias.in ©Vision IAS
o आसके ऄक्ततररि, नीक्तत में एक क्राईड फं लडग तंत्र की भी पररकल्पना की गइ है, क्तजसमें कं पक्तनयों और िोगों को दुिभ
ण रोगों
के ईपचार के क्तिए एक मजबूत अइिी मंच के माध्यम से क्तवत्तीय समथणन देने हेतु प्रोत्साक्तहत दकया जाएगा। आसक्तिए कथन
2 सही है।
 दुिभ
ण रोगों के ईपचार के क्तिए क्तवत्तीय समथणन राष्ट्रीय अरोग्य क्तनक्तध (RAN) की ऄम्ब्रेिा योजना के तहत प्रस्ताक्तवत है, न दक
अयुष्मान भारत, प्रधानमंत्री जन अरोग्य योजना (PMJAY) के तहत। आसक्तिए कथन 3 सही नहीं है।

Q 54.B
 प्रारं भ में क्तवक्तभन्न सामाक्तजक-अर्मथक ऄक्तधकारों की क्तववेचना करते समय ईच्चतम न्यायािय द्वारा नीक्तत क्तनदेिक तत्वों की ईपेक्षा
की गयी थी। ऐसा 1973 में के िवानंद भारती वाद में ऐक्ततहाक्तसक क्तनणणय के पिात दकया गया था, क्तजसमें िीषण न्यायािय द्वारा
यह कहा गया दक क्तनदेिक तत्वों को ईन ददिा-क्तनदेिों के रूप में देखा जाना चाक्तहए क्तजनके द्वारा मूि ऄक्तधकार साक्तधत
(realized) होते हैं।
 1980 के क्तमनवाण क्तमल्स मामिे में न्यायमूर्मत पीएन भगवती ने कहा था दक "भारतीय संक्तवधान की संस्थापना मूि ऄक्तधकारों
और राज्य के नीक्तत क्तनदेिक तत्वों के मध्य संति
ु न के अधार पर की गइ है। एक को दूसरे के उपर पूणण प्रधानता देने से संक्तवधान
का सामंजस्य (harmony) क्तबगडता है। यह सामंजस्य और संति
ु न संक्तवधान की मूि संरचना की एक ऄक्तनवायण क्तविेषता है। "
 चंपकम दोराआराजन वाद (1951) में, ईच्चतम न्यायािय ने यह क्तनणणय ददया दक मूि ऄक्तधकारों और क्तनदेिक तत्वों के मध्य
दकसी भी िकराव की क्तस्थक्तत में मूि ऄक्तधकार प्रभावी होंगे।
 ऄनुच्छेद 21 के कायाणन्वयन हेतु मेनका गांधी वाद एक ऐक्ततहाक्तसक वाद है। आस मामिे में, न्यायािय द्वारा यह कहा गया दक
ऄनुच्छेद 21 में ऄक्तभव्यक्ति 'व्यक्तिगत स्वतंत्रता' का व्यापक अयाम है और आसमें कइ प्रकार के ऄक्तधकार सक्तम्मक्तित हैं, जो
व्यक्ति की व्यक्तिगत स्वतंत्रता का गठन करते हैं। और आनमें से कु छ ने क्तवक्तिष्ट मूि ऄक्तधकारों की प्रक्तस्थक्तत में वृक्ति की है तथा
ऄनुच्छेद 19 के तहत ऄक्ततररि सुरक्षा प्रदान की गयी है।
 आसक्तिए क्तवकल्प (b) सही ईत्तर है।

Q 55.B
 जब तक ऄथणव्यवस्था वापस पिरी पर नहीं अती है, तब तक वतणमान समायोजन नीक्तत के रुख को बनाए रखने के क्तिए भारतीय
ररजवण बैंक (RBI) की प्रक्ततबिता को दोहराते हुए, गवनणर द्वारा एक नए कायणक्रम-सरकारी प्रक्ततभूक्तत ऄक्तधग्रहण कायणक्रम
(Government Securities Acquisition Programme:G-SAP) के साथ बाजारों को ईत्साक्तहत दकया गया है, क्तजसके
माध्यम से RBI क्तवत्त वषण 2022 की पहिी क्ततमाही में 1 िाख करोड रुपये के मूल्य की सरकारी प्रक्ततभूक्ततयों की (सीधे सरकार
से नहीं) खरीदारी खुिे बाजार से करे गा। आसक्तिए कथन 1 सही नहीं है।
 कें द्रीय बैंक ने कहा है दक कायणक्रम के माध्यम से अर्मथक सुधार के क्तिए कषणण (Traction) प्राप्त करने हेतु ऄनुकूि क्तवत्तीय
पररक्तस्थक्ततयों को सुक्तनक्तित करने के क्तिए प्रयास दकया गया है।
 G-SAP घोषणा के माध्यम से RBI द्वारा बाजार सहभाक्तगयों को खुिे बाजार पररचािन (OMO) खरीद कै िेंडर प्रदान दकया
गया है, क्तजसे बाजार सहभागी हमेिा से जानना चाहते हैं। एडिवाआस म्यूचऄ
ु ि फं ड की एक ररपोिण के ऄनुसार यह RBI द्वारा
क्तवत्त वषण 2022 में बंधपत्र बाजार (Bond market) की सहायता की प्रक्ततबिता के संदभण में बंधपत्र बाजार सहभाक्तगयों को
क्तनक्तितता प्रदान करे गा।
 आसके समान अकार के एक संरक्तचत खरीद कायणक्रम से बाजार के सहभाक्तगयों को ऄक्तधसूक्तचत नीिामी में बेहतर बोिी िगाने
और बंधपत्र की कीमतों में ऄक्तस्थरता को कम करने में सहायता क्तमिेगी।
 आस संरक्तचत कायणक्रम की घोषणा से रे पो दर और 10 वषण के सरकारी बंधपत्र िाभ के बीच प्रसार को कम करने में सहायता
क्तमिेगा। पररणामस्वरूप, यह क्तवत्त वषण 2022 में कें द्र और राज्यों के क्तिए ईधार की समग्र िागत को कम करने में सहायता
करे गा। आसक्तिए कथन 2 सही है।

22 www.visionias.in ©Vision IAS


Q 56.C
 भारत का संक्तवधान कें द्र स्तर पर संघ िोक सेवा अयोग (UPSC) की भांक्तत राज्य स्तर पर राज्य िोक सेवा अयोग (SPSC)
का प्रावधान करता है। संक्तवधान के ऄनुच्छेद 315 से 323 तक SPSC की स्वतंत्रता, गठन, कायों एवं िक्तियों तथा सदस्यों की
क्तनयुक्ति एवं हिाने के संबंध में प्रावधान दकये गए हैं।
 क्तवकल्प (a) सही है: राज्य िोक सेवा अयोग के ऄध्यक्ष और सदस्यों की क्तनयुक्ति राज्य के राज्यपाि द्वारा की जाती है। यद्यक्तप
आनकी क्तनयुक्ति राज्यपाि द्वारा की जाती है दकन्तु आन्हें के वि राष्ट्रपक्तत द्वारा ही पद से हिाया जा सकता है। राष्ट्रपक्तत आन्हें ईसी
अधार एवं रीक्तत से हिा सकता है क्तजस अधार एवं रीक्तत से वह UPSC के ऄध्यक्ष या सदस्यों को हिाता है।
 क्तवकल्प (b) सही है: SPSC का सदस्य सेवाक्तनवृक्तत्त के पिात् UPSC का ऄध्यक्ष एवं सदस्य ऄथवा ईसी SPSC या दकसी
ऄन्य SPSC के ऄध्यक्ष के रूप में क्तनयुि होने के क्तिए ऄहण होता है, दकन्तु वह भारत सरकार या दकसी राज्य सरकार के ऄधीन
दकसी ऄन्य क्तनयोजन के क्तिए ऄहण नहीं होता है। आसके ऄक्ततररि, SPSC का ऄध्यक्ष सेवाक्तनवृक्तत्त ऄथवा पद छोडने के पिात्
UPSC का ऄध्यक्ष एवं सदस्य ऄथवा दकसी ऄन्य SPSC के ऄध्यक्ष के रूप में क्तनयुि होने के क्तिए ऄहण होता है, दकन्तु वह
भारत सरकार या दकसी राज्य सरकार के ऄधीन दकसी ऄन्य क्तनयोजन के क्तिए ऄहण नहीं होता है।
 क्तवकल्प (c) सही नहीं है: SPSC के ऄध्यक्ष और सदस्यों के वेतन, भत्ते और पेंिन सक्तहत SPSC का व्यय राज्य की संक्तचत
क्तनक्तध (भारत की संक्तचत क्तनक्तध पर नहीं) पर भाररत होता है। ऄतः आन पर राज्य क्तवधानमंडिों में मतदान नहीं दकया जाता है।
आसके ऄक्ततररि, ऄध्यक्ष और सदस्यों की सेवा की ितों में गैर िाभकारी पररवतणन नहीं नहीं दकये जा सकते हैं।
 क्तवकल्प (d) सही है: SPSC की भूक्तमका सीक्तमत है और आसके द्वारा की गइ ऄनुिस
ं ाओं की प्रकृ क्तत के वि परामिणकारी होती हैं
और आसक्तिए ये सरकार पर गैर-बाध्यकारी हैं। यह राज्य सरकार पर क्तनभणर है दक वह परामिण को स्वीकार करें ऄथवा नहीं।
हािांदक, सरकार को अयोग की ऄनुिस
ं ाओं को स्वीकार नहीं करने के बारे में संबंक्तधत राज्य क्तवधानमंडि को ईत्तर देना होता
है।

Q 57.C
 क्तवकल्प 1 सही नहीं है: संक्तवधान के कइ प्रावधानों को ऄनुच्छेद 368 के दायरे से बाहर संसद के दोनों सदनों में साधारण बहुमत
के द्वारा संिोक्तधत दकया जा सकता है। आन प्रावधानों में सक्तम्मक्तित हैं:
o संसद में गणपूर्मत।
o संसद सदस्यों के वेतन और भत्ते।
o संसद में प्रदक्रया के क्तनयम।
o संसद, ईसके सदस्यों और ईसकी सक्तमक्ततयों के क्तविेषाक्तधकार।
o संसद में ऄंग्रज
े ी भाषा का ईपयोग।
 क्तवकल्प 4 सही नहीं है: राजव्यवस्था के संघीय ढांचे से संबंक्तधत संक्तवधान के प्रावधानों को संसद के क्तविेष बहुमत द्वारा और
साथ ही अधे राज्यों के क्तवधान-मंडिों का ऄनुसमथणन से संिोक्तधत दकए जा सकता है। क्तनम्नक्तिक्तखत प्रावधानों को ईपरोि
प्रदक्रया से संिोक्तधत दकया जा सकता है:
o राष्ट्रपक्तत का क्तनवाणचन और क्तनवाणचन की रीक्तत।
o संघ और राज्यों की कायणपाक्तिका िक्तियों का क्तवस्तार।
o ईच्चतम न्यायािय और ईच्च न्यायािय से सम्बंक्तधत प्रावधान।
o संघ और राज्य के बीच क्तवधायी िक्तियों का क्तवभाजन।
 क्तवकल्प 2 और 3 सही हैं: संक्तवधान में ऄक्तधकांि ईपबंधों को संसद के क्तविेष बहुमत द्वारा संिोक्तधत करने की अवश्यकता होती
है ऄथाणत प्रत्येक सदन के कु ि सदस्यों का बहुमत तथा प्रत्येक सदन के ईपक्तस्थत और मतदान करने वािे सदस्यों का दो-क्ततहाइ
बहुमत। आस रीक्तत से संिोक्तधत दकए जाने वािे ईपबंधों में सक्तम्मक्तित हैं:
o मौक्तिक ऄक्तधकार।
o राज्य के नीक्तत-क्तनदेिक तत्व।

Q 58.B
 वषण 1969 में ईच्चतम न्यायािय ने क्तनणणय ददया था दक भारत और दकसी ऄन्य देि के बीच सीमा क्तववाद के क्तनपिारे के क्तिए
दकसी संक्तवधान संिोधन की अवश्यकता नहीं है। आसका क्तनपिान कायणकारी कारण वाइ द्वारा दकया जा सकता है क्योंदक आसमें
क्तवदेिी क्षेत्र को भारतीय क्षेत्र का ऄध्यपणण सक्तम्मक्तित नहीं है। आसक्तिए क्तवकल्प (b) सही ईत्तर है।

23 www.visionias.in ©Vision IAS


 जबदक कें द्र सरकार द्वारा पादकस्तान को बेरूबारी यूक्तनयन (पक्तिम बंगाि) के एक क्षेत्र का क्तहस्सा सौंपे जाने के क्तनणणय से
राजनीक्ततक प्रक्ततरोध और क्तववाद ईत्पन्न हो गया था और आस कारण राष्ट्रपक्तत से परामिण की अवश्यकता ईत्पन्न हुइ थी।
 ईच्चतम न्यायािय ने माना दक दकसी राज्य के क्षेत्र को घिाने के क्तिए संसद की िक्ति (ऄनुच्छेद 3 के तहत) एक भारतीय क्षेत्र
को ऄन्य देि को ऄध्यपणण करने को सक्तम्मक्तित नहीं करती है। आसक्तिए भारतीय क्षेत्र को के वि ऄनुच्छेद 368 के तहत संक्तवधान
में संिोधन करके ही एक क्तवदेिी राज्य को सौंपा जा सकता है। आसके फिस्वरूप, 9वें संक्तवधान संिोधन ऄक्तधक्तनयम (1960) को
ईि क्षेत्र को पादकस्तान को स्थानांतररत करने के क्तिए ऄक्तधक्तनयक्तमत दकया गया था।

Q 59.D
 संक्तवधान के भाग तीन में सभी मौक्तिक ऄक्तधकार राज्य के कायों के क्तवरुि सुरक्षा प्रदान करते हैं परन्तु आनमें से कु छ क्तनजी
व्यक्तियों के कायों के क्तवरुि भी ईपिब्ध हैं जैस,े
 ऄनुच्छेद 15(2): आसमें ईपबंक्तधत है दक कोइ नागररक के वि धमण, मूिवंि, जाक्तत, लिग, जन्मस्थान या आनमें से दकसी के अधार
पर--
(a) दुकानों, सावणजक्तनक भोजनाियों, होििों और सावणजक्तनक मनोरं जन के स्थानों में प्रवेि, या
(b) पूणत
ण ः या भागतः राज्य-क्तनक्तध से पोक्तषत या साधारण जनता के प्रयोग के क्तिए समर्मपत कु ओं, तािाबों, स्नानघािों, सडकों
और सावणजक्तनक समागम के स्थानों के ईपयोग,
के संबंध में दकसी भी क्तनयोषयता, दाक्तयत्व, क्तनबणन्धन या ितण के ऄधीन नहीं होगा।
 ऄनुच्छेद 17: यह ऄस्पृश्यता को समाप्त करता है और दकसी भी रूप में आसका अचरण क्तनक्तषि करता है।
 ऄनुच्छेद 23: यह मानव दुव्याणपार, बेगार (बिात् श्रम) और आसी प्रकार के ऄन्य बिात् श्रम के प्रकारों पर भी प्रक्ततक्तषि करता
है।
 ऄनुच्छेद 24: यह दकसी भी कारखाने, खान ऄथवा ऄन्य पररसंकिमय गक्ततक्तवक्तधयों यथा क्तनमाणण कायण ऄथवा रे िवे में 14 वषण
से कम अयु के बािकों का क्तनयोजन क्तनषेध करता है।
 ईपयुि ण ऄनुच्छेदों के ऄंतगणत ऄक्तधकारों के ईल्िंघन के क्तिए क्तनजी व्यक्तियों के क्तवरुि भी सुरक्षा ईपिब्ध हैं और राज्य यह
सुक्तनक्तित करता है दक ईक्तचत कारण वाइ की जाए।
 आसक्तिए क्तवकल्प (d) सही ईत्तर है।

Q 60.C
 ऄनुच्छेद 19, सभी नागररकों को छह ऄक्तधकारों की गारं िी प्रदान करता है। यह हैं:
o वाक् -स्वातंत्र्य और ऄक्तभव्यक्ति-स्वातंत्र्य का ऄक्तधकार।
o िांक्ततपूवणक और क्तनरायुध सम्मेिन का ऄक्तधकार।
o संगम या संघ (या सहकारी सक्तमक्ततयों) बनाने का ऄक्तधकार।
o भारत के राज्यक्षेत्र में सवणत्र ऄबाध संचरण का ऄक्तधकार।
o भारत के राज्यक्षेत्र के दकसी भाग में क्तनवास करने और बसने का ऄक्तधकार।
o कोइ वृक्तत्त, ईपजीक्तवका, व्यापार या कारोबार करने का ऄक्तधकार।
 संक्तवधान में 97वां संिोधन सहकारी सक्तमक्ततयों को ऄनुच्छेद 19(1)(c) में ऄंत:स्थाक्तपत करता है, क्तजससे सहकारी सक्तमक्ततयों को
मौक्तिक ऄक्तधकार मान कर िोगों के ऄक्तधकार के रूप में मान्यता दी जा सके । आसक्तिए कथन 1 सही है।
 ये छह ऄक्तधकार के वि राज्य की कारण वाइ के क्तवरुि सुरक्षा प्रदान करते हैं, न दक क्तनजी व्यक्तियों के क्तवरुि। आसके ऄक्ततररि, ये
ऄक्तधकार के वि नागररकों और दकसी कं पनी के िेयरधारकों के क्तिए ईपिब्ध हैं, िेदकन क्तवदेिी या कानूनी व्यक्तियों जैसे
कं पक्तनयों या क्तनगमों अदद के क्तिए नहीं। आसक्तिए कथन 2 सही है।

Q 61.A
 कथन 1 सही: वस्तु एवं सेवा कर (GST) पररषद एक संवध
ै ाक्तनक क्तनकाय है। आसकी स्थापना वस्तु एवं सेवा कर से संबक्तं धत
मुद्दों पर कें द्र और राज्य सरकार को ऄनुिस
ं ाएं प्रस्तुत करने के क्तिए ऄनुच्छेद 279A के तहत की गइ है। GST पररषद की
स्थापना 101वें संक्तवधान संिोधन ऄक्तधक्तनयम, 2016 के तहत की गयी है। यह कें द्र और राज्यों का एक संयुि मंच है और

24 www.visionias.in ©Vision IAS


आसकी ऄध्यक्षता कें द्रीय क्तवत्त मंत्री द्वारा की जाती है। पररषद की गणपूर्मत (कोरम) आसकी सदस्यता का 50% है ऄथाणत पररषद
की कु ि सदस्य संख्या के अधे सदस्यों का ईपक्तस्थत होना ऄक्तनवायण है।
 कथन 2 सही नहीं है: GST पररषद का प्रत्येक क्तनणणय ईपक्तस्थत और मतदान करने वािे सदस्यों के तीन चौथाइ बहुमत (75%
बहुमत) होने के बाद ही ऄनुमोददत दकया जाता है। (कु ि संख्या बि का अधा भाग नहीं)।
 कथन 3 सही नहीं है: GST पररषद में प्रत्येक राज्य की मतदान क्तहस्सेदारी ईसकी जनसंख्या पर क्तनभणर नहीं करती है। प्रत्येक
राज्य की मतदान क्तहस्सेदारी समान है और सभी राज्यों के संयुि मतों में डािे गए कु ि मतों का दो-क्ततहाइ भारांि होगा। कें द्र
सरकार का मत भारांि कु ि मतों का एक क्ततहाइ होगा।

Q 62.B
 भारत में सरकार की संसदीय व्यवस्था ऄक्तधकांितः क्तब्ररिि संसदीय व्यवस्था पर अधाररत है। हािांदक यह कभी भी क्तब्ररिि
व्यवस्था की प्रक्ततकृ क्तत नहीं रही है और क्तनम्नक्तिक्तखत मामिों में आससे क्तभन्न है:
o क्तब्ररिि राजतंत्रीय व्यवस्था के स्थान पर भारत में एक गणतंत्रीय व्यवस्था को ऄपनाया गया है। दूसरे िब्दों में, भारत में राज्य
का मुक्तखया (ऄथाणत, राष्ट्रपक्तत) क्तनवाणक्तचत होता है जबदक क्तब्रिेन में राज्य का मुक्तखया (ऄथाणत सम्राि या साम्राज्ञी) वंिानुगत होता
है।
o क्तब्ररिि व्यवस्था संसद की संप्रभुता के क्तसिांत पर अधाररत है जबदक भारत में संसद सवोच्च नहीं है। भारत में संसद की िक्तियों
पर एक क्तिक्तखत संक्तवधान, संघीय व्यवस्था, न्याक्तयक समीक्षा और मूि ऄक्तधकार अदद के माध्यम से ऄनेक प्रक्ततबंध अरोक्तपत
दकए गए हैं। आसक्तिए कथन 1 सही नहीं है।
o क्तब्रिेन में प्रधानमंत्री को संसद के क्तनचिे सदन (हाईस ऑफ कॉमन्स) का सदस्य होना चाक्तहए। भारत में प्रधानमंत्री संसद के
दोनों सदनों में से दकसी एक का सदस्य हो सकता है।
o सामान्यतः क्तब्रिेन में संसद के सदस्यों को मंत्री के रूप में क्तनयुि दकया जाता है। भारत में एक ऐसे व्यक्ति जो संसद का सदस्य
नहीं है, को भी ऄक्तधकतम छह माह की ऄवक्तध के क्तिए मंत्री के रूप में क्तनयुि दकया जा सकता है।
o क्तब्रिेन में मंक्तत्रयों के पास क्तवक्तधक ईत्तरदाक्तयत्व होता है, जबदक भारत में ऐसी कोइ व्यवस्था नहीं है। क्तब्रिेन के क्तवपरीत, भारत में
मंक्तत्रयों को राज्य के प्रमुख/राष्ट्रपक्तत के दकसी अक्तधकाररक कायण पर प्रक्तत-हस्ताक्षर करना अवश्यक नहीं होता है। आसक्तिए कथन
2 सही है।
o क्तब्ररिि मंक्तत्रमंडिीय व्यवस्था में ‘छाया मंक्तत्रमंडि' (िैडो कै क्तबनेि) एक क्तवक्तिष्ट संस्था है। आसे क्तवपक्षी दिों द्वारा गरठत दकया
जाता है तादक सत्तारूढ़ दि के साथ संति
ु न बना रहे और ऄपने सदस्यों को भावी मंत्राियी कायों के क्तिए तैयार दकया जा सके ।
भारत में ऐसी कोइ संस्था नहीं है।

Q 63.B
 ऄनुच्छेद 359 राष्ट्रपक्तत को राष्ट्रीय अपातकाि के दौरान मौक्तिक ऄक्तधकारों के प्रवतणन के क्तिए दकसी भी न्यायािय में वाद
दायर करने के ऄक्तधकार को क्तनिंक्तबत करने के क्तिए ऄक्तधकृ त करता है। आसका ऄथण है दक ऄनुच्छेद 359 के तहत, मौक्तिक
ऄक्तधकार को क्तनिंक्तबत नहीं दकया जाता है, बक्तल्क के वि ईनका प्रवतणन क्तनिंक्तबत होता है। ईि ऄक्तधकार सैिांक्ततक रूप से तो
ईपिब्ध रहते हैं िेदकन प्रवतणन का ऄक्तधकार क्तनिंक्तबत हो जाता है। प्रवतणन का क्तनिंबन के वि ईन मौक्तिक ऄक्तधकारों से संबंक्तधत
है जो राष्ट्रपक्तत के अदेि में क्तनर्ददष्ट हों। आसक्तिए कथन 2 सही है।
 44वें संिोधन ऄक्तधक्तनयम, 1978 ने ऄनुच्छेद 359 के दायरे को दो प्रकार से प्रक्ततबंक्तधत कर ददया था।
o सवणप्रथम, राष्ट्रपक्तत ऄनुच्छेद 20 एवं 21 द्वारा प्रदत्त मौक्तिक ऄक्तधकारों के प्रवतणन के क्तिए न्यायािय में वाद दायर करने के
ऄक्तधकार को क्तनिंक्तबत नहीं कर सकता। दूसरे िब्दों में, ऄपराधों के क्तिए दोषक्तसक्ति के संबंध में संरक्षण (ऄनुच्छेद 20) और
प्राण और दैक्तहक स्वतंत्रता का संरक्षण (ऄनुच्छेद 21) अपातकाि के दौरान भी िागू होंगे।
o दूसरे , के वि ईन क्तवक्तधयों को चुनौती से संरक्षण प्राप्त है जो अपातकाि से संबंक्तधत हैं, ईन सभी क्तवक्तधयों व कायों को नहीं
जो आनके तहत बनाए गए हैं।
 ऄनुच्छेद 359 दोनों मामिे में यथा युि या बाह्य अक्रमण और सिस्त्र क्तवद्रोह के अधार पर अपात की ईद्घोषणा, में िागू
होता है।
 ऄनुच्छेद 359 संपण
ू ण देि या आसके दकसी भाग तक क्तवस्ताररत हो सकता है। आसक्तिए कथन 3 सही नहीं है।
 ऄनुच्छेद 359, राष्ट्रपक्तत को दकसी राज्य के क्तिए ऄध्यादेि प्रख्याक्तपत करने का प्राक्तधकार प्रदान नहीं करता है। आसक्तिए कथन 1
सही नहीं है।
25 www.visionias.in ©Vision IAS
Q 64.C
 ऄनुच्छेद 29 ऄल्पसंख्यकों के क्तहतों की सुरक्षा से संबंक्तधत है। यह ईपबंक्तधत करता है दक भारत के राज्यक्षेत्र या ईसके दकसी भाग
के क्तनवासी नागररकों के दकसी ऄनुभाग को, क्तजसकी ऄपनी क्तविेष भाषा, क्तिक्तप या संस्कृ क्तत है, ईसे बनाए रखने का ऄक्तधकार
होगा।
 आसके ऄक्ततररि राज्य द्वारा पोक्तषत या राज्य-क्तनक्तध से सहायता पाने वािी दकसी क्तिक्षा संस्था में प्रवेि से दकसी भी नागररक को
के वि धमण, मूिवंि, जाक्तत, भाषा या आनमें से दकसी के अधार पर वंक्तचत नहीं दकया जाएगा।
 ऄनुच्छेद 29 दोनों, धार्ममक ऄल्पसंख्यकों के साथ-साथ भाषाइ ऄल्पसंख्यकों को संरक्षण प्रदान करता है।
 हािांदक, ईच्चतम न्यायािय ने कहा है दक आस ऄनुच्छेद का दायरा के वि ऄल्पसंख्यकों तक ही सीक्तमत नहीं है, जैसा दक
सामान्यतः माना जाता है। आसका कारण यह है दक ऄनुच्छेद 29 में 'नागररकों के ऄनुभाग' िब्द का ऄक्तभप्राय ऄल्पसंख्यक और
बहुसंख्यक दोनों से है। आसक्तिए कथन 1 सही है।
 ऄनुच्छेद 30 ऄल्पसंख्यकों को क्तनम्नक्तिक्तखत ऄक्तधकार देता है, चाहे वह धार्ममक हो या भाषाइ:
o धमण या भाषा पर अधाररत सभी ऄल्पसंख्यक-वगों को ऄपनी रुक्तच की क्तिक्षा संस्थाओं की स्थापना और प्रिासन का
ऄक्तधकार होगा।
o राज्य द्वारा ऄल्पसंख्यक क्तिक्षण संस्थान की दकसी भी संपक्तत्त के ऄक्तनवायण ऄजणन के क्तिए क्तनधाणररत क्षक्ततपूर्मत रकम से ईनके
क्तिए प्रत्याभूत ऄक्तधकार क्तनबंक्तधत या क्तनराकृ त नहीं होंगे।
o राज्य अर्मथक सहायता में ऄल्पसंख्यकों द्वारा प्रबंक्तधत दकसी भी िैक्षक्तणक संस्थान के साथ भेदभाव नहीं करे गा।
 आस प्रकार, ऄनुच्छेद 30 के तहत संरक्षण के वि ऄल्पसंख्यकों (धार्ममक या भाषाइ) तक ही सीक्तमत है और नागररकों के दकसी
ऄन्य ऄनुभाग के क्तिए (जैसा दक ऄनुच्छेद 29 के तहत) ईपिब्ध नहीं है। हािांदक, संक्तवधान में कहीं भी, ऄल्पसंख्यक िब्द को
पररभाक्तषत नहीं दकया गया है। आसक्तिए कथन 2 सही है।
 ऄल्पसंख्यक क्तिक्षण संस्थान तीन प्रकार के होते हैं:
o ऐसा संस्थान जो राज्य से मान्यता प्राप्त करने के साथ-साथ अर्मथक सहायता भी प्राप्त करते हैं।
o ऐसा संस्थान जो के वि राज्य से मान्यता प्राप्त करते हैं िेदकन अर्मथक सहायता प्राप्त नहीं करते।
o ऐसा संस्थान जो न तो राज्य से मान्यता प्राप्त करते हैं और न ही अर्मथक सहायता प्राप्त करते हैं
 पहिे और दूसरे प्रकार के संस्थान राज्य की क्तनयामक िक्ति के ऄधीन होते हैं, क्तजसमें पाठ्यक्रम, िैक्षक्तणक मानक, ऄनुिासन,
स्वच्छता, क्तिक्षण कमणचाररयों का क्तनयोजन अदद िाक्तमि होते हैं।
 तीसरे प्रकार के संस्थान ऄपने मामिों का प्रबंधन करने के क्तिए स्वतंत्र हैं, िेदकन ऄनुबध
ं कानून, श्रम कानून, औद्योक्तगक कानून,
कर कानून, अर्मथक क्तनयम अदद जैसे सामान्य कानूनों के संचािन के ऄधीन होते हैं।
Q 65.C
 हाक्तिया संदभण: पेंिन क्तनक्तध क्तवक्तनयामक और क्तवकास प्राक्तधकरण (Pension Fund Regulatory and Development
Authority: PFRDA) के ऄध्यक्ष ने कहा है दक राष्ट्रीय पेंिन प्रणािी (National Pension Scheme: NPS) के तहत ऄब
क्तनवेिकों को ईनकी संक्तचत सेवाक्तनवृक्तत्त मूि क्तनक्तध के 40% को वार्मषकी में पररवर्मतत करना बाध्यकारी नहीं है, क्योंदक वार्मषक
अय और ईच्च मुद्रास्फीक्तत पर क्तनम्न प्रक्ततफि ऊणात्मक ररिनण में रूपांतररत हो रही है। क्तनयामक िीघ्र ही NPS में 5 िाख रुपए
तक की बचत करने वािे ऄक्तभदाताओं को सेवाक्तनवृक्तत्त के समय पूरी राक्ति िेने की ऄनुमक्तत प्रदान करे गा, जो दक वतणमान में 2
िाख रुपए है।
 राष्ट्रीय पेंिन योजना कें द्र सरकार की एक सामाक्तजक सुरक्षा पहि है। आससे पहिे, NPS योजना में के वि कें द्र सरकार के
कमणचाररयों को सक्तम्मक्तित दकया जाता था। हािांदक वतणमान में, PFRDA ने स्वैक्तच्छक अधार पर सभी भारतीय नागररकों के
क्तिए आसे ईपिब्ध करा ददया गया है। आसक्तिए कथन 1 सही है।
 NPS योजना ईस प्रत्येक व्यक्ति के क्तिए बहुत महत्व रखती है जो क्तनजी क्षेत्र में काम करता है और ईसे सेवाक्तनवृक्तत्त के बाद
क्तनयक्तमत पेंिन की अवश्यकता होगी।
 ऄक्तनवासी भारतीय(NRI), NPS खाता खोि सकते हैं। NRI के द्वारा दकए गए योगदान समय-समय पर भारतीय ररजवण बैंक
(RBI) और फे मा (FEMA) द्वारा क्तनधाणररत क्तनयामक अवश्यकताओं के ऄधीन होते हैं। हािांदक, भारत के प्रवासी नागररक
(Overseas Citizen of India: OCI) और PIO (भारतीय मूि के व्यक्ति) काडणधारक और लहदू ऄक्तवभाक्तजत पररवार (
Hindu Undivided Family: HUF), NPS खाता खोिने के क्तिए पात्र नहीं हैं। आसक्तिए कथन 2 सही है।

26 www.visionias.in ©Vision IAS


 NPS खाता के वि व्यक्तिगत रूप से खोिा जा सकता है। आसे संयुि रूप से या HUF की ओर से खोिा या संचाक्तित नहीं दकया
जा सकता है।

Q 66.C
 भारतीय संक्तवधान की प्रस्तावना में संिोधन दकया जा सकता है, जबदक प्रस्तावना में क्तनक्तहत संक्तवधान की मूि क्तविेषताओं में
कोइ संिोधन नहीं दकया जा सकता है।
 प्रस्तावना में वषण 1976 में 42वें संक्तवधान संिोधन ऄक्तधक्तनयम के द्वारा के वि एक बार संिोधन दकया गया था। आस संिोधन के
द्वारा प्रस्तावना में तीन िब्द जोडे गए जो समाजवादी, पंथक्तनरपेक्ष और ऄखंडता है।
 आसक्तिए क्तवकल्प (c) सही ईत्तर है।

Q 67.C
 हाक्तिया संदभण: हाि ही में, ईच्चतम न्यायािय ने एक क्तनणणय में कहा है दक “प्रत्येक स्तर पर क्तिक्षा तक पहुुँच को सुक्तवधाजनक
बनाने की स्वीकारात्मक बाध्यता" राज्य की होगी।
न्यायािय ने अर्मथक, सामाक्तजक और सांस्कृ क्ततक ऄक्तधकारों पर सक्तमक्तत द्वारा पररकक्तल्पत दृक्तष्टकोण का ईल्िेख दकया। न्यायािय
ने सक्तमक्तत के एक ईपवाक्य ईदृत करते हुए कहा दक "सिक्तिकरण ऄक्तधकार के रूप में, क्तिक्षा एक प्राथक्तमक माध्यम है क्तजसके
द्वारा अर्मथक और सामाक्तजक रूप से संवद
े निीि वयस्क एवं बच्चे स्वयं को क्तनधणनता से बाहर क्तनकाि सकते हैं और ऄपने
समुदायों में पूणत
ण या सहभागी बनने के साधन प्राप्त कर सकते हैं"। न्यायािय ने कहा दक भारत संयि
ु राष्ट्र के अर्मथक, सामाक्तजक
और सांस्कृ क्ततक ऄक्तधकारों पर ऄंतराणष्ट्रीय वाचा (ICESCR) का हस्ताक्षरकताण था।
 अर्मथक, सामाक्तजक और सांस्कृ क्ततक ऄक्तधकारों पर ऄंतराणष्ट्रीय वाचा (ICESCR) ऄपनी सह-वाचा ऄथाणत, नागररक और
राजनीक्ततक ऄक्तधकारों पर ऄंतराणष्ट्रीय वाचा (ICCPR) और सावणभौक्तमक घोषणा के साथ क्तमिकर ऄंतराणष्ट्रीय मानवाक्तधकार
क्तबि (International Bill of Human Rights) को तैयार करता है, जो संयुि राष्ट्र के भीतर मानवाक्तधकार संरक्षण का एक
स्तंभ है।
 मानव ऄक्तधकारों की सावणभौक्तमक घोषणा (गैर-कानूनी रूप से एक बाध्यकारी दस्तावेज) को वषण 1948 में ऄपनाया गया था।
 वषण 1966 में, दो ऄिग संक्तधयों द्वारा िगभग पूणत
ण ः मानव ऄक्तधकारों की सावणभौक्तमक घोषणा में क्तनक्तहत सभी ऄक्तधकारों को
समाक्तहत करते हुए, करीब 20 वषों की बातचीत के बाद ऄपनाया गया था। ICESCR एक ऄंतराणष्ट्रीय मानवाक्तधकार संक्तध है,
यह ईन राज्यों के क्तिए कानूनी रूप से बाध्यकारी ऄंतराणष्ट्रीय दाक्तयत्वों का क्तनमाणण करती है जो आसमें क्तनक्तहत मानकों से ऄंगीकृ त
करने के क्तिए सहमत हुए हैं। आसक्तिए कथन 1 सही है।
 अर्मथक, सामाक्तजक और सांस्कृ क्ततक ऄक्तधकारों पर सक्तमक्तत, ICESCR का पयणवेक्षी क्तनकाय है। आसे संयुि राष्ट्र अर्मथक और
सामाक्तजक पररषद (ECOSOC) संकल्प के ऄंतगणत स्थाक्तपत दकया गया था। आसक्तिए कथन 2 सही है।
Q 68.B
 भारत में, जन्म से नागररकता प्राप्त व्यक्ति साथ ही साथ प्राकृ क्ततक रूप से नागररकता प्राप्त व्यक्ति, दोनों ही राष्ट्रपक्तत पद के क्तिए
पात्र हैं, जबदक संयुि राज्य ऄमेररका में, के वि जन्म से कोइ नागररक राष्ट्रपक्तत पद के क्तिए पात्र है न दक प्राकृ क्ततक रूप से
नागररक होने की क्तस्थक्तत में। आसक्तिए कथन 1 सही नहीं है।
 सभी भारतीय नागररकों को संसद और राज्य क्तवधानमंडिों की सदस्यता के क्तिए चुनाव िडने का ऄक्तधकार है। आसक्तिए कथन 2
सही है।
o संसद का सदस्य चुने जाने के क्तिए, दकसी व्यक्ति को भारत का नागररक होना अवश्यक है। राज्यसभा के मामिे में ईसकी
अयु 30 वषण से कम और िोकसभा के मामिे में 25 वषण से कम नहीं होनी चाक्तहए। ऄन्य ऄहणताएं संसद द्वारा बनाइ गइ
दकसी क्तवक्तध द्वारा क्तवक्तहत की जाएंगी।
o राज्य क्तवधानमंडि का सदस्य चुने जाने के क्तिए, दकसी व्यक्ति को भारत का नागररक होना ऄक्तनवायण है। आसके ऄक्ततररि,
व्यक्ति की अयु 25 वषण से कम नहीं होनी चाक्तहए, वह मानक्तसक रूप से स्वस्थ होना चाक्तहए, ददवाक्तिया नहीं होना चाक्तहए
और संबंक्तधत राज्य की मतदाता सूची में नामांदकत होना चाक्तहए, जहां से वह चुनाव िड रहा/रही है।

27 www.visionias.in ©Vision IAS


Q 69.B
 यद्यक्तप राज्य के नीक्तत क्तनदेिक तत्त्व गैर-न्यायोक्तचत हैं तथाक्तप संक्तवधान (ऄनुच्छेद 37) में आस बात को स्पष्ट दकया गया है दक ये
तत्त्व देि के िासन में मूिभूत हैं और राज्य का यह कतणव्य होगा दक वह आन तत्त्वों का क्तवक्तध क्तनमाणण में प्रयोग करे ।
 संक्तवधान क्तनमाणताओं ने क्तनदेिक तत्त्वों को गैर-न्यायोक्तचत और क्तवक्तधक रूप से गैर-प्रवतणनीय बनाया है क्योंदक:
o देि के पास आन्हें िागू करने के क्तिए पयाणप्त क्तवत्तीय संसाधन ईपिब्ध नहीं थे। सभी क्तनदेिक तत्त्वों को िागू करने के क्तिए
नव स्वतंत्र देि को भारी मात्रा में क्तवत्तीय संसाधनों की अवश्यकता होती और यह देि के क्तिए एक चुनौती थी। आस कारण
आन्हें गैर-न्यायोक्तचत बनाया गया था। आसक्तिए कथन 1 सही है।
o देि में व्यापक क्तवक्तवधता एवं क्तपछडापन आनके कायाणन्वयन में बाधक होते। क्तनदेिक तत्त्वों की प्रकृ क्तत ऄत्यंत क्तवक्तवधतापूणण है
और संक्तवधान क्तनमाणताओं ने ऄनुभव दकया दक ईन्हें न्यायोक्तचत बनाने से नव-स्वतंत्र देि के क्तिए बाधाएं ईत्पन्न हो सकती हैं
क्योंदक कु छ क्तनदेिक तत्त्वों जैसे दक समान नागररक संक्तहता (Uniform Civil Code) का क्तवरोध कु छ समुदायों द्वारा दकया
गया था। वे राज्य को आनके कायाणन्वयन के क्तिए पयाणप्त समय भक्तवष्य के भारतीय नेताओं के चयन का ऄवसर देना चाहते
थे। आसक्तिए कथन 2 सही है।
 मूि ऄक्तधकारों को पहिे से ही न्यायोक्तचत बनाया गया है और क्तनदेिक तत्त्वों को न्यायोक्तचत बनाना क्तनरथणक होता। कथन 3
सही नहीं है क्योंदक क्तनदेिक तत्त्वों को न्यायोक्तचत बनाना क्तनरथणक नहीं होगा और वास्तव में ईन्हें न्यायोक्तचत बनाने से
सामाक्तजक रूप से न्यायपूणण तथा समतावादी समाज का क्तनमाणण हो सकता है। ऄत: ईपरोि दोनों कारणों से ईन्हें न्यायोक्तचत
नहीं बनाया गया है।
 आस प्रकार, संक्तवधान क्तनमाणताओं ने एक व्यावहाररक दृक्तष्टकोण ऄपनाते हुए आन क्तनदेिक तत्त्वों को ऄक्तधक प्रभाविािी नहीं
बनाया। ईनका मानना था दक आन क्तनदेिक तत्त्वों के क्तनवणहन हेतु ऄंक्ततम मंज़ूरी के रूप में ऄदािती प्रदक्रयाओं के बजाय एक
जागृत जनमत बेहतर होगा।
Q 70.B
 ऄनुच्छेद 14 ‘क्तवक्तध के समक्ष समता एवं क्तवक्तधयों के समान संरक्षण’ का ईपबंध करता है। आसके ऄनुसार राज्य, भारत के
राज्यक्षेत्र में दकसी व्यक्ति को क्तवक्तध के समक्ष समता से या क्तवक्तधयों के समान संरक्षण से वंक्तचत नहीं रखेगा। आसक्तिए कथन 1
सही नहीं है।
 'क्तवक्तध के समक्ष समता' की ऄवधारणा क्तब्ररिि मूि की है, जबदक 'क्तवक्तधयों के समान संरक्षण' की ऄवधारणा को ऄमेररका के
संक्तवधान से क्तिया गया है।
 पहिी ऄवधारणा में क्तनक्तहत है:
o दकसी व्यक्ति के पक्ष में दकसी क्तवक्तिष्ट क्तविेषाक्तधकारों की ऄनुपक्तस्थक्तत,
o साधारण क्तवक्तध न्यायाियों द्वारा प्रिाक्तसत क्तवक्तध के समक्ष सभी व्यक्तियों के प्रक्तत समान व्यवहार, और
o कोइ भी व्यक्ति (धनी या क्तनधणन, ईच्च या क्तनम्न, ऄक्तधकारी या गैर-ऄक्तधकारी) क्तवक्तध से उपर नहीं है।
 वहीं, दूसरी ऄवधारणा में क्तनक्तहत है:
o क्तवक्तधयों द्वारा प्रदत्त क्तविेषाक्तधकारों और ऄध्यारोक्तपत दाक्तयत्वों दोनों में समान पररक्तस्थक्ततयों के ऄंतगणत व्यवहार की
समता,
o समान क्तवक्तधयों के ऄंतगणत सभी व्यक्तियों के क्तिये समान क्तनयम,
o क्तबना भेदभाव के समान के साथ समान व्यवहार।
 ऄनुच्छेद 14 वगण अधाररत क्तवक्तध क्तनमाणण को क्तनक्तषि करता है परं तु नीक्तत अदद जैसे युक्तियुि वगीकरण को क्तनक्तषि नहीं करता
है। ईदाहरणत:, दकसी क्तविेष योजना के तहत दकसी धनी व्यक्ति और दकसी क्तनधणन व्यक्ति के साथ समान व्यवहार नहीं दकया जा
सकता। आसक्तिए ईच्चतम न्यायािय ने कहा दक जहां समान और ऄसमान के साथ ऄिग-ऄिग व्यवहार होता है, वहाुँ ऄनुच्छेद
14 िागू नहीं होता है। आसक्तिए कथन 2 सही है।
 ऄनुच्छेद 31ग, ऄनुच्छेद 14 का एक ऄपवाद है। आसके ऄनुसार, दकसी राज्य द्वारा क्तनदेिक तत्त्वों के दक्रयान्वयन के संबंध में
यदद कोइ क्तनयम या कानून बनाया जाता है, क्तजसमें ऄनुच्छेद 39 की ईपधारा (ख) या ईपधारा (ग) का समावेि है तो ईसे आस
अधार पर चुनौती नहीं दी जा सकती दक वे ऄनुच्छेद 14 का ईल्िंघन करते हैं। यह ऄनुच्छेद 39 की ईपधारा (ख) और ईपधारा
(ग) के मामिे में ही िागू होता है न दक ऄनुच्छेद 39 में सभी ईपधाराओं में।
Q 71.D
 संक्तवधान के ऄनेक ईपबंधों या प्रावधानों को संसद के दोनों सदनों के साधारण बहुमत के द्वारा संिोक्तधत दकया जा सकता है। ये
व्यवस्थाएं ऄनुच्छेद 368 की सीमा से बाहर हैं। आन प्रावधानों में िाक्तमि हैं:
28 www.visionias.in ©Vision IAS
o नए राज्यों का प्रवेि या गठन। आसक्तिए क्तवकल्प 1 सही है।
o नए राज्यों का क्तनमाणण और मौज़ूदा राज्यों के क्षेत्रों, सीमाओं या नामों का पररवतणन।
o राज्य क्तवधानपररषदों का क्तनमाणण या ईसकी समाक्तप्त।
o दूसरी ऄनुसच
ू ी- राष्ट्रपक्तत, राज्यपािों, िोकसभा ऄध्यक्ष, न्यायाधीिों अदद हेतु पररिक्तब्धयाुँ, भत्ते और क्तविेषाक्तधकार
अदद।
o संसद में गणपूर्मत।
o संसद सदस्यों के वेतन एवं भत्ते।
o संसद में प्रदक्रया क्तनयम।
o संसद, आसके सदस्यों और आसकी सक्तमक्ततयों को क्तविेषाक्तधकार।
o संसद में ऄंग्रज
े ी भाषा का प्रयोग।
o ईच्चतम न्यायािय में ऄवर न्यायाधीिों की संख्या।
o ईच्चतम न्यायािय के न्यायक्षेत्र को ज़्यादा महत्व प्रदान करना। आसक्तिए क्तवकल्प 4 सही है।
o राजभाषा का प्रयोग। आसक्तिए क्तवकल्प 2 सही है।
o नागररकता की प्राक्तप्त एवं समाक्तप्त।
o संसद और राज्य क्तवधानमंडिों के क्तिए क्तनवाणचन। आसक्तिए क्तवकल्प 3 सही है।
o क्तनवाणचन क्षेत्रों का पुनर्मनधाणरण।
o संघ िाक्तसत प्रदेि।
o पाुँचवीं ऄनुसच ू ी-ऄनुसूक्तचत क्षेत्रों और ऄनुसूक्तचत जनजाक्ततयों का प्रिासन।
o छठी ऄनुसच
ू ी-जनजातीय क्षेत्रों का प्रिासन।

Q 72.D
 भारत में सभी नागररकों को ईनके जन्म स्थान और क्तनवास स्थान के अधार पर क्तबना क्तवभेद के पूरे देि में समान राजनीक्ततक
और नागररक ऄक्तधकार प्राप्त होते हैं, ईनके बीच दकसी तरह का भेदभाव नहीं दकया जा सकता है।
 हािांदक, भेदभावहीनता के सामान्य क्तनयम में कु छ ऄपवाद भी हैं,
o संसद (ऄनुच्छेद 16 के ऄंतगणत ) राज्य या संघ राज्य क्षेत्र के भीतर ईस राज्य या संघ राज्य क्षेत्र में कु छ क्तनयोजन या
क्तनयुक्तियों के क्तिए क्तनवास की ितण अरोक्तपत कर सकती है। तदनुसार, संसद ने सावणजक्तनक रोजगार (क्तनवास के रूप में
जरूरत) ऄक्तधक्तनयम, 1957 को ऄक्तधक्तनयक्तमत दकया है। आसक्तिए कथन 1 सही है।
o संक्तवधान (ऄनुच्छेद 15 के ऄंतगणत) दकसी भी नागररक के क्तवरुि धमण, मूिवंि, जाक्तत, लिग या जन्म-स्थान के अधार पर
क्तवभेद का प्रक्ततषेध करता है, न दक क्तनवास के अधार पर। आसका ऄथण यह है दक राज्य क्तनवास के अधार पर ऄपने
क्तनवाक्तसयों को क्तविेष सुक्तवधाएं प्रदान कर सकता है या ईन्हें प्राथक्तमकता दे सकता है जो दक संक्तवधान द्वारा ददए गए
ऄक्तधकारों के सीमा क्षेत्र में अने वािा मामिा नहीं है। ईदाहरणत:, एक राज्य ऄपने क्तनवाक्तसयों के क्तिए िैक्षक्तणक िुल्क में
छू ि दे सकता है। आसक्तिए कथन 2 सही है।
o ऄबाध संचरण एवं क्तनवास करने की स्वतंत्रता (ऄनुच्छेद 19 के ऄंतगणत) ऄनुसक्तू चत जनजाक्तत के क्तहतों के संरक्षण के ऄधीन
है। दूसरे िब्दों में, जनजातीय क्षेत्रों में बाहरी िोगों के प्रवेि एवं क्तनवास और बसने का ऄक्तधकार प्रक्ततबंक्तधत है। आसक्तिए
कथन 3 सही है।
o जम्मू एवं कश्मीर के मामिे में, राज्य क्तवधानमंडि को यह िक्ति दी गइ थी दक वह ईन व्यक्तियों को पररभाक्तषत करे जो
राज्य के स्थायी क्तनवासी हैं और ईन्हें राज्य सरकार के ऄधीन रोज़गार, राज्य में ऄचि संपक्तत्त के ऄक्तधग्रहण अदद के
मामिों में ऄक्तधकार एवं क्तविेषाक्तधकार प्रदान कर सके ककतु ऄब यह िक्ति प्रभाव में नहीं (कें द्र सरकार द्वारा ऄनुच्छेद 370
की समाक्तप्त के बाद) है।

Q 73.A
 हाक्तिया संदभण: ओक्तडिा ति पर क्तस्थत क्तचल्का झीि में डॉक्तल्फनों की अबादी क्तपछिे वषण की तुिना में आस वषण दोगुनी हो गइ है।
यह भारत में खारे पानी की सबसे बडी झीि है।

29 www.visionias.in ©Vision IAS


 राज्य वन एवं पयाणवरण क्तवभाग के वन्य जीव संभाग ने आस वषण डॉक्तल्फन की गणना पर ऄंक्ततम अंकडे जारी दकए हैं। ये अंकडें
जनवरी और फरवरी में एकक्तत्रत दकए गए थे। यह गणना डॉक्तल्फनों की अबादी में ऄसाधारण वृक्ति के संकेत देती है।
 क्तचल्का, एक्तिया की सबसे बडी खारे पानी का पि जि (िैगन
ू ) है। आसमें ज्वारनदमुखी (एस्चुएरी) क्तविेषताएं पाइ जाती हैं।
यह भारत के प्रथम रामसर कन्वेंिन स्थिों (ऄंतराणष्ट्रीय महत्व की अद्रणभूक्तमयों) में से एक है। आसक्तिए कथन 1 सही है।
 क्तचल्का, ओक्तडिा के तीन क्तजिों में क्तवस्ताररत एक िैगन
ू है। आसका कु छ भाग महासागरीय, कु छ भाग खारा और कु छ भाग मीठे
जि से युि है। आसके बीच में कहीं-कहीं कु छ द्वीप क्तस्थत हैं और यह पूवी घाि की छोिी-बडी पहाक्तडयों से क्तघरी हुइ है।
 ताजे पानी की ऄंतदेिीय नददयों के क्तविय एवं बंगाि की खाडी से खारे पानी के प्रवेि के साथ, िैगून समृि जैव क्तवक्तवधता के
क्तिए ऄनुकूि है। आसमें कु छ दुिभ
ण , संकिापन्न एवं ऄक्ततसंवेदनिीि प्रजाक्ततयां िाक्तमि हैं, जैसे आरावदी डॉक्तल्फन। क्तचल्का झीि
क्तवश्व में आरावदी डॉक्तल्फन के एकमात्र सबसे बडे पयाणवास स्थि के रूप में ईभरकर सामने अइ है। आसक्तिए कथन 2 सही नहीं
है।
 िैगन
ू , भारत के पूवी ति पर दया नदी के मुहाने पर क्तस्थत है। राष्ट्रीय समुद्र क्तवज्ञान संस्थान के समुद्री पुरातत्व क्तवभाग द्वारा
दकये गए एक ऄध्ययन के ऄनुसार क्तचल्का कभी बंगाि की खाडी का क्तहस्सा थी।
Q 74.D
 राष्ट्रपक्तत द्वारा अपात की ईद्घोषणा दकसी भी समय एक दूसरी ईद्घोषणा से समाप्त की जा सकती है। ऐसी ईद्घोषणा को संसदीय
ऄनुमोदन की अवश्यकता नहीं होती है। आसक्तिए कथन 1 सही नहीं है।
 यदद संसद के दोनों सदनों द्वारा आसका ऄनुमोदन हो गया है, तो अपातकाि छह माह तक जारी रहेगा तथा प्रत्येक छह माह में
संसद के ऄनुमोदन से आसे ऄनंतकाि तक बढ़ाया जा सकता है। आसक्तिए कथन 2 सही नहीं है।

ऄक्ततररि जानकारी:

 राष्ट्रीय अपातकाि
 युि, बाह्य अक्रमण या सिस्त्र क्तवद्रोह के अधार पर राष्ट्रीय अपातकाि की ईद्घोषणा की जा सकती है। आस प्रकार के
अपातकाि की ईद्घोषणा करने के क्तिए संक्तवधान ‘अपात की ईद्घोषणा’ का ईपबंध करता है।
 ईद्घोषणा के अधार:
o ऄनुच्छेद 352 के तहत, यदद राष्ट्रपक्तत का यह समाधान हो जाता है दक गंभीर अपात क्तवद्यमान है क्तजससे युि या बाह्य
अक्रमण या सिस्त्र क्तवद्रोह के कारण भारत या ईसके राज्यक्षेत्र के दकसी भाग की सुरक्षा संकि में है तो वह ईद्घोषणा द्वारा
[संपूणण भारत या ईसके राज्यक्षेत्र के ऐसे भाग के संबंध में जो ईद्घोषणा में क्तवक्तनर्ददष्ट दकया जाए] आस अिय की घोषणा
कर सके गा।
o राष्ट्रपक्तत, राष्ट्रीय अपातकाि की ईद्घोषणा वास्तक्तवक युि ऄथवा बाह्य अक्रमण या सिस्त्र क्तवद्रोह से पहिे भी कर
सकता है।
 संसदीय ऄनुमोदन तथा समयावक्तध
o संसद के दोनों सदनों द्वारा अपात की ईद्घोषणा जारी होने की क्ततक्तथ से एक माह के भीतर ऄनुमोददत होनी अवश्यक है।
यदद संसद के दोनों सदनों द्वारा आसका ऄनुमोदन हो गया है, तो अपातकाि छह माह तक जारी रहेगा तथा प्रत्येक छह
माह में संसद के ऄनुमोदन से आसे ऄनंतकाि तक बढ़ाया जा सकता है।
o अपात की ईद्घोषणा ऄथवा आसके जारी रहने का प्रत्येक प्रस्ताव संसद के दोनों सदनों द्वारा क्तविेष बहुमत से पाररत होना
चाक्तहए।
 राष्ट्रीय अपातकाि के प्रभाव
o अपात की ईद्घोषणा के राजनीक्ततक तंत्र पर तीव्र तथा दूरगामी प्रभाव होते हैं। आन पररणामों को क्तनम्नक्तिक्तखत तीन वगों में
रखा जा सकता है:
o कें द्र-राज्य संबंधों पर प्रभाव: जब अपात की ईद्घोषणा िागू होती है, तब कें द्र-राज्य के सामान्य संबध
ं ों में मूिभूत पररवतणन
होते हैं। आसका ऄध्ययन तीन िीषणकों के ऄंतगणत दकया जा सकता है:
o कायणपािक: कें द्र द्वारा ‘दकसी’ भी क्तवषय पर राज्य को कायणकारी क्तनदेि देने की िक्ति प्राप्त हो जाती है।
o क्तवधायी: संसद को राज्य सूची में वर्मणत क्तवषयों पर कानून बनाने का ऄक्तधकार प्राप्त हो जाता है। यदद संसद का सत्र नहीं
चि रहा है, तो राष्ट्रपक्तत राज्य सूची के क्तवषयों पर भी ऄध्यादेि जारी कर सकता है। संसद द्वारा राज्य के क्तवषयों पर बने
कानून अपातकाि की समाक्तप्त के बाद, छह माह तक प्रभावी होते हैं।

30 www.visionias.in ©Vision IAS


o क्तवत्तीय: राष्ट्रपक्तत, कें द्र और राज्यों के मध्य करों के संवैधाक्तनक क्तवतरण को संिोक्तधत कर सकता है।
 िोकसभा और राज्य क्तवधानसभा के कायणकाि पर प्रभाव:
o जब राष्ट्रीय अपातकाि की ईद्घोषणा िागू हो, तब िोकसभा का कायणकाि आसके सामान्य कायणकाि से अगे एक समय में
एक वषण के क्तिए बढ़ाया जा सकता है। हािांदक, यह क्तवस्तार अपातकाि समाक्तप्त के बाद छह माह से ऄक्तधक जारी नहीं रह
सकता।
o आसी प्रकार, राष्ट्रीय अपात के समय संसद दकसी राज्य क्तवधानसभा के सामान्य कायणकाि को प्रत्येक बार एक वषण तक बढ़ा
सकती है, जो अपातकाि की समाक्तप्त के बाद ऄक्तधकतम छह माह तक ही रहता है।
 मूि ऄक्तधकारों पर प्रभाव: ऄनुच्छेद 358 और 359 राष्ट्रीय अपातकाि में मूि ऄक्तधकारों पर प्रभाव का वणणन करते हैं। ये दो
प्रावधान क्तनम्नानुसार वर्मणत दकए गए हैं:
o ऄनुच्छेद 19 के ऄंतगणत प्रदत्त मूि ऄक्तधकारों का क्तनिंबन: ऄनुच्छेद 358 के ऄनुसार, जब राष्ट्रीय अपात की ईद्घोषणा की
जाती है, तो ऄनुच्छेद 19 द्वारा प्रदत्त छह मूि ऄक्तधकार स्वतः ही क्तनिंक्तबत हो जाते हैं। अपातकाि की समाक्तप्त के बाद
ऄनुच्छेद 19 स्वत: ही पुनजीक्तवत हो जाता है।
o 44वें संिोधन ऄक्तधक्तनयम ने यह क्तनधाणररत दकया दक ऄनुच्छेद 19 को के वि तभी क्तनिंक्तबत दकया जा सकता है जब राष्ट्रीय
अपातकाि युि या बाह्य अक्रमण के अधार पर िगाया जाए न दक सिस्त्र क्तवद्रोह के अधार पर।
o ऄन्य मूि ऄक्तधकारों का क्तनिंबन: ऄनुच्छेद 359 के तहत, राष्ट्रपक्तत को अदेि द्वारा, राष्ट्रीय अपातकाि के दौरान मूि
ऄक्तधकारों के प्रवतणन के क्तिए दकसी भी न्यायािय में जाने के ऄक्तधकार को क्तनिंक्तबत करने के क्तिए ऄक्तधकृ त दकया गया है।
आस प्रकार, के वि ईपचारात्मक ईपायों को क्तनिंक्तबत दकया जाता है न दक मूि ऄक्तधकारों को।
o यह क्तनिंबन के वि ईन्हीं मूि ऄक्तधकारों से संबंक्तधत होता है जो राष्ट्रपक्तत के अदेि में वर्मणत होते हैं।
o यह क्तनिंबन अपातकाि की ऄवक्तध या ऄल्पावक्तध के क्तिए हो सकता है।
o आसे संसद के ऄनुमोदन के क्तिए प्रत्येक सदन में प्रस्तुत दकया जाना चाक्तहए।
o 44वें संक्तवधान संिोधन ऄक्तधक्तनयम में यह ऄक्तधदेक्तित दकया गया है दक राष्ट्रपक्तत ऄनुच्छेद 20 और 21 द्वारा प्रदत्त मूि
ऄक्तधकारों को िागू करने के क्तिए न्यायािय में जाने के ऄक्तधकार को क्तनिंक्तबत नहीं कर सकता है।

Q 75.C
 भारत के संक्तवधान की प्रस्तावना चार मूि तत्वों को प्रकि करती है।
 प्रस्तावना में ईल्िेख है दक संक्तवधान भारत के िोगों से िक्ति ऄक्तधगृहीत करता है। आसक्तिए यह संक्तवधान के ऄक्तधकार के स्रोत
को प्रकि करता है।
 प्रस्तावना यह घोषणा करती है दक भारत एक संप्रभु, समाजवादी, धमणक्तनरपेक्ष, िोकतांक्तत्रक व गणतांक्तत्रक राजव्यवस्था वािा
देि है। आसक्तिए यह भारतीय राज्य की प्रकृ क्तत को प्रकि करती है।
 प्रस्तावना संक्तवधान के ईद्देश्यों के रूप में न्याय, स्वतंत्रता, समता और बंधत्ु व को क्तनर्ददष्ट करती है।
 यह 26 नवंबर, 1949 को संक्तवधान को ऄंगीकृ त करने की क्ततक्तथ के रूप में ईक्तल्िक्तखत करती है।
 प्रस्तावना न तो क्तवधाक्तयका की िक्ति का स्रोत है और न ही क्तवक्तध की िक्तियों पर प्रक्ततबंध।
 मूि कतणव्यों और राज्य की नीक्तत के क्तनदेिक तत्वों की तरह, प्रस्तावना भी गैर-न्याक्तयक है, ऄथाणत् आसके प्रावधानों को न्यायािय
में चुनौती नहीं दी जा सकती है।
 आसक्तिए क्तवकल्प (c) सही ईत्तर है।

Q 76.A
 एक ओर जहां प्रधानमंत्री सरकार का प्रमुख होता है, वहीं राष्ट्रपक्तत राज्य का प्रमुख होता है। हमारी राजनैक्ततक व्यवस्था में,
राष्ट्राध्यक्ष के वि नाममात्र के ऄक्तधकारों का प्रयोग करता है। राष्ट्रपक्तत देि के सभी राजनीक्ततक संस्थानों की समग्र कायण पिक्तत
का क्तनरीक्षण करता है तादक वे राज्य के ईद्देश्यों को प्राप्त करने के क्तिए क्तमि-जुिकर कायण कर सकें ।
 संपूणण क्तवश्व में राष्ट्रपक्तत सदैव भारत के राष्ट्रपक्तत के समान नाममात्र के िासनाध्यक्ष नहीं होते हैं। क्तवश्व के ऄनेक देिों में, राष्ट्रपक्तत
राष्टाध्यक्ष भी होता है और सरकार का मुक्तखया भी। संयुि राज्य ऄमेररका का राष्ट्रपक्तत आस प्रकार के राष्ट्रपक्तत का जाना-मान
ईदाहरण है। ऄमेररकी राष्ट्रपक्तत को िोगों द्वारा प्रत्यक्ष रूप से क्तनवाणक्तचत दकया जाता है। वह सभी मंक्तत्रयों का चुनाव और
31 www.visionias.in ©Vision IAS
क्तनयुक्ति व्यक्तिगत रूप से करता है। क्तवक्तध क्तनमाणण का कायण ऄभी भी क्तवधाक्तयका (ऄमेररका में आसे कांग्रेस कहा जाता है) द्वारा ही
दकया जाता है, ककतु राष्ट्रपक्तत दकसी भी क्तवक्तध को वीिो के ऄक्तधकार से रोक सकता है। सबसे महत्वपूणण त्य यह है दक राष्ट्रपक्तत
बनने के क्तिए कांग्रस
े में सदस्यों के बहुमत के समथणन की अवश्यकता नहीं होती है और न ही वह ईनके प्रक्तत ईत्तरदायी होता है।
ईसका चार वषों का क्तनक्तित कायणकाि होता है और यदद कांग्रस े में ईसके दि के पास बहुमत नहीं भी होता है तब भी वह
ऄपना कायणकाि पूरा करता है।
 ऄध्यक्षात्मक व्यवस्था के गुण:
o क्तस्थर सरकार। आसक्तिए क्तवकल्प 2 सही है।
o नीक्ततयों में क्तनक्तितता।
o िक्तियों के पृथक्करण पर अधाररत । आसक्तिए क्तवकल्प 3 सही है।
o क्तविेषज्ञों द्वारा सरकार का संचािन।
 ऄध्यक्षात्मक व्यवस्था के दोष:
o क्तवधाक्तयका एवं कायणपाक्तिका के मध्य संघषण।
o गैर-ईत्तरदायी सरकार। आसक्तिए क्तवकल्प 4 सही नहीं है।
o सरकार क्तनरं कुि हो सकती है।
o सीक्तमत प्रक्ततक्तनक्तधत्व। आसक्तिए क्तवकल्प 1 सही नहीं है।

Q 77.A
 साआबर स्पेस नवाचार, अर्मथक प्रगक्तत, सांस्कृ क्ततक क्तवकास और सूचना तक पहुंच के क्तिए वृहत ऄवसर प्रदान करता है। जबदक
आसका त्वररत क्तवकास ऄनेक मानवीय गक्ततक्तवक्तधयों के क्तिए ऄत्यंत ईपयोगी क्तसि हुअ है, वहीं आससे नए खतरे भी ईत्पन्न हुए
हैं।
 12 नवंबर 2018 को अयोक्तजत पेररस कॉि फॉर ट्रस्ि एंड क्तसक्योररिी आन साआबर स्पेस, नागररकों और ऄवसंरचनाओं के क्तिए
खतरा ईत्पन्न करने वािे नए खतरों का सामना करने के क्तिए िोगों को एक साथ अने का अह्वान करता है। यह साआबर स्पेस
को सुरक्तक्षत करने के क्तिए नौ सामान्य क्तसिांतों पर अधाररत है, जो चचाण और कारण वाइ के क्तिए ऄक्तधक से ऄक्तधक क्षेत्रों में कायण
करते हैं।
 पेररस कॉि सभी साआबर स्पेस के ऄक्तभकताणओं को एक साथ कायण करने के क्तिए अमंक्तत्रत करता है और राज्यों को क्तनजी क्षेत्र के
भागीदारों, वैक्तश्वक ऄनुसध
ं ान और नागररक समाज के साथ सहयोग करने के क्तिए प्रोत्साक्तहत करता है। पेररस कॉि के समथणक
ईत्तरदायी व्यवहार को ऄपनाने और वास्तक्तवक क्तवश्व में िागू होने वािे मूि क्तसिांतों को िागू करने के क्तिए एकजुि होकर कायण
करने के क्तिए प्रक्ततबि हैं। आसक्तिए कथन 1 सही है।
 यूनेस्को (UNESCO) और पेररस िांक्तत मंच में अयोक्तजत आं िरनेि गवनेंस फोरम के दौरान वषण 2018 में फ्ांसीसी गणराज्य के
राष्ट्रपक्तत आमैनए
ु ि मैक्रों द्वारा पेररस कॉि का अह्वान दकया गया था।
 भारत आस ईद्घोषणा का पक्षकार नहीं है तथा हाि ही में माआक्रोसॉफ्ि ने साआबर स्पेस में क्तवश्वास और सुरक्षा हेतु पेररस कॉि में
सक्तम्मक्तित होने के क्तिए भारत से अग्रह दकया था। आस ईद्घोषणा को ऄब तक 75 देिों ने ऄपनाया है जो क्तवश्व के समक्ष नए
साआबर-सुरक्षा से संबंक्तधत खतरों से क्तनपिने के क्तिए एक साथ अए हैं। आसक्तिए कथन 2 सही नहीं है।

Q 78.D
 ऄनुच्छेद 22 दकसी व्यक्ति को क्तगरफ्तारी या क्तनरोध से संरक्षण प्रदान करता है। क्तनरोध दो प्रकार के होते हैं, ऄथाणत,् दंड क्तवषयक
और क्तनवारक। दंड क्तवषयक क्तनरोध में दकसी व्यक्ति को ईसके द्वारा दकए गए ऄपराध के क्तिए न्यायािय में मुकदमे तथा दोषी
क्तसि होने के पिात् दंक्तडत दकया जाता है।
 दूसरी ओर क्तनवारक क्तनरोध का ऄथण है, दकसी व्यक्ति को न्यायािय द्वारा क्तबना सुनवाइ के दोषी ठहराया जाना। आसका ईद्देश्य
दकसी व्यक्ति को क्तपछिे ऄपराध पर दंक्तडत नहीं करना है बक्तल्क भक्तवष्य में ऐसे ऄपराध करने से रोकना है।
 ऄनुच्छेद 22 के दो भाग हैं- पहिा भाग (खंड 1 और 2) साधारण क्तवक्तध के मामिों से संबंक्तधत है और दूसरा भाग (खंड 4, 5, 6,
और 7) क्तनवारक क्तनरोध क्तवक्तध के मामिों से संबंक्तधत है।
 ऄनुच्छेद 22 का दूसरा भाग ईन व्यक्तियों को सुरक्षा प्रदान करता है क्तजन्हें क्तनवारक क्तनरोध क्तवक्तध के ऄंतगणत क्तगरफ्तार दकया
जाता है। यह सुरक्षा नागररकों एवं क्तवदेिी दोनों के क्तिए भी ईपिब्ध है। आसमें िाक्तमि हैं:

32 www.visionias.in ©Vision IAS


o दकसी व्यक्ति की क्तहरासत तीन माह से ऄक्तधक नहीं बढ़ाइ जा सकती है जब तक की सिाहकार बोडण आस संदभण में ईक्तचत
कारण न बताए। आस बोडण में एक ईच्च न्यायािय के न्यायाधीि सक्तम्मक्तित होंगे।
o क्तनरोध का अधार संबक्तं धत व्यक्ति को िीघ्राक्ततिीघ्र बताया जाना चाक्तहए। हािांदक सावणजक्तनक क्तहतों के क्तवरुि आसे बताना
अवश्यक नहीं है। ईसकी क्तगरफ्तारी का कारण बताने के क्तिए 24 घंिे की कोइ समय सीमा नहीं है। आसक्तिए कथन 1 सही
नहीं है।
o क्तनरोध वािे व्यक्ति को क्तनरोध के अदेि के क्तवरुि प्रक्ततवेदन करने का ऄक्तधकार है।
 ऄनुच्छेद 22 संसद को भी यह बताने के क्तिए ऄक्तधकृ त करता है दक
o दकन पररक्तस्थक्ततयों के ऄधीन और दकस वगण या वगों के मामिों में दकसी व्यक्ति को क्तनवारक क्तनरोध का ईपबंध करने वािी
दकसी क्तवक्तध के ऄधीन तीन माह से ऄक्तधक ऄवक्तध के क्तिए सिाहकार बोडण की राय प्राप्त दकए क्तबना क्तहरासत में रखा जा
सके गा।
o दकसी वगण या वगों के मामिों में दकसी व्यक्ति को ऄक्तधकतम दकतनी ऄवक्तध के क्तिए क्तनवारक क्तनरोध क्तवक्तध के तहत
क्तहरासत में क्तिया जा सके गा।
o जांच में सिाहकार बोडण द्वारा ऄनुसरण की जाने वािी प्रदक्रया।
 संक्तवधान ने क्तनवारक क्तनरोध के संबंध में संसद और राज्य क्तवधानमंडि के बीच वैधाक्तनक िक्तियों को क्तवभि दकया है।
 संसद के पास रक्षा, क्तवदेिी मामिों और भारत की सुरक्षा से जुडे मामिों के संबध
ं में क्तनवारक क्तनरोध की क्तवक्तध क्तनमाणण का
क्तविेष ऄक्तधकार है।
 संसद एवं राज्य क्तवधानमंडि दोनों को समुदाय के क्तिए अवश्यक वस्तुओं एवं सेवाओं की अपूर्मत सुक्तनक्तित करने, सावणजक्तनक
व्यवस्था बनाए रखने और राज्य के सुरक्षा मामिों अदद पर क्तनरोध संबध
ं ी क्तवक्तध क्तनमाणण का ऄक्तधकार है। आसक्तिए कथन 2 सही
नहीं है।

Q 79.A
 सूचना का ऄक्तधकार ऄक्तधक्तनयम (2005) के प्रावधानों के ऄंतगणत, कें द्रीय सूचना अयोग (CIC) का गठन कें द्र सरकार द्वारा वषण
2005 में एक गैर संवैधाक्तनक क्तनकाय के रूप में दकया गया था। यह एक स्वतंत्र क्तनकाय है जो कें द्र सरकार एवं संघ राज्यक्षेत्रों के
ऄधीन कायणरत कायाणियों से संबक्तं धत क्तिकायतों और ऄपीिों की सुनवाइ करता है।
 कथन 1 सही है: आसमें एक मुख्य सूचना अयुि और दस से ऄनक्तधक सूचना अयुि होते हैं। आन सभी की क्तनयुक्ति राष्ट्रपक्तत द्वारा
एक सक्तमक्तत की ऄनुिंसा पर की जाती है। आस सक्तमक्तत में ऄध्यक्ष के रूप में प्रधानमंत्री, िोकसभा में क्तवपक्ष का नेता एवं
प्रधानमंत्री द्वारा मनोनीत एक कै क्तबनेि मंत्री सक्तम्मक्तित होता है।
 कथन 2 सही नहीं है: हाि ही में ऄक्तधक्तनयक्तमत RTI संिोधन ऄक्तधक्तनयम, 2019 (RTI amendment Act, 2019) में यह
प्रावधान दकया गया है दक मुख्य सूचना अयुि और सूचना अयुि ईस ऄवक्तध तक पद धारण करें गे जो कें द्र सरकार द्वारा
क्तनधाणररत की गयी हो। आससे पूवण ये 5 वषों के क्तिए पद धारण करते थे।
 कथन 3 सही नहीं है: RTI संिोधन ऄक्तधक्तनयम, 2019 के ऄनुसार मुख्य सूचना अयुि एवं सूचना अयुि के वेतन और भत्ते
कें द्र सरकार द्वारा क्तनधाणररत दकए जाएंग।े आसने पूवण का यह प्रावधान हिा ददया है क्तजसके तहत मुख्य सूचना अयुि के वेतन
और भत्ते मुख्य चुनाव अयुि के समान होते थे।

Q 80.B
 ऄनुच्छेद 368 में संक्तवधान के संिोधन की प्रदक्रया का क्तनम्नक्तिक्तखत तरीके से ईल्िेख दकया गया है:
 संक्तवधान के संिोधन का अरं भ संसद के दकसी भी सदन में आस प्रयोजन के क्तिए क्तवधेयक पुरःस्थाक्तपत करके ही दकया जा सके गा
तथा राज्य क्तवधानमंडि में नहीं। आसक्तिए क्तवकल्प 1 और 2 सही है तथा क्तवकल्प 3 सही नहीं है।
 क्तवधेयक को दकसी मंत्री या गैर-सरकारी सदस्य द्वारा पुरःस्थाक्तपत दकया जा सकता है और आसके क्तिए राष्ट्रपक्तत की पूवण ऄनुमक्तत
की अवश्यकता नहीं होती है।
Q 81. A
 ऄनुच्छेद 360(1) राष्ट्रपक्तत को क्तवत्तीय अपातकाि की घोषणा करने की िक्ति प्रदान करता है, “यदद राष्ट्रपक्तत का यह समाधान
हो जाता है दक ऐसी क्तस्थक्तत ईत्पन्न हो गइ है, क्तजससे भारत या ईसके राज्यक्षेत्र के दकसी भाग का क्तवत्तीय स्थाक्तयत्व या प्रत्यय
संकि में है तो वह ईद्घोषणा द्वारा आस अिय की घोषणा कर सके गा”।
 क्तवत्तीय अपातकाि की घोषणा को, घोक्तषत क्ततक्तथ के दो माह के भीतर संसद के दोनों सदनों की स्वीकृ क्तत क्तमिना ऄक्तनवायण है।

33 www.visionias.in ©Vision IAS


 आसके ऄक्ततररि, क्तवत्तीय अपातकाि की घोषणा को मंजरू ी देने वािे प्रस्ताव, संसद के दकसी भी सदन द्वारा साधारण बहुमत
द्वारा पाररत दकया जा सकता है। आसक्तिए कथन 1 सही है।
 संसद के दोनों सदनों द्वारा मंज़रू ी प्राप्त हो जाने के पिात्, क्तवत्तीय अपातकाि ऄक्तनक्तित काि तक प्रभावी रहेगा, जब तक दक
आसे वापस न क्तिया जाए।
 राष्ट्रपक्तत द्वारा दकसी भी समय ऄनुवती घोषणा द्वारा क्तवत्तीय अपातकाि की ईद्घोषणा वापस िी जा सकती है। आस तरह की
ईद्घोषणा के क्तिए दकसी संसदीय ऄनुमोदन की अवश्यकता नहीं होती है। आसक्तिए कथन 2 सही नहीं है।
 ऄभी तक दकसी क्तवत्तीय अपातकाि की घोषणा नहीं की गइ है। यद्यक्तप, 1991 में क्तवत्तीय संकि अया था। आसक्तिए कथन 3
सही नहीं है।

Q 82. D
 भारत का संक्तवधान ऄनुच्छेद 324 के तहत, देि में स्वतंत्र और क्तनष्पक्ष चुनाव सुक्तनक्तित कराने के क्तिए एक स्थायी और स्वतंत्र
क्तनकाय के रूप में क्तनवाणचन अयोग का गठन करता है। क्तनवाणचन अयोग में संसद, राज्य क्तवधान-मंडि, भारत के राष्ट्रपक्तत और
ईपराष्ट्रपक्तत के पदों के क्तनवाणचन के क्तिए ऄधीक्षण, क्तनदेिन और क्तनयंत्रण की िक्ति क्तनक्तहत है।
 कथन 1 सही नहीं है: संक्तवधान में क्तनवाणचन अयोग के सदस्यों की ऄहणता (क्तवक्तधक, िैक्षक्तणक, प्रिासक्तनक या न्याक्तयक) क्तनधाणररत
नहीं की गइ है। ऄनुच्छेद 324 का खण्ड ईपबंक्तधत करता है दक संसद मुख्य क्तनवाणचन अयुि (CEC) और क्तनवाणचन अयुिों
(ECs) की सेवा की ितों का क्तनधाणरण कर सकती है।
 कथन 2 सही नहीं है: ऄनुच्छेद 324 के ऄनुसार, क्तनवाणचन अयोग एक मुख्य क्तनवाणचन अयुि और ऐसी संख्या में क्तनवाणचन
अयुिों से क्तमिकर बना होगा जो समय-समय पर राष्ट्रपक्तत द्वारा क्तनधाणररत दकए जाएंग।े संसद ने क्तनवाणचन अयोग (क्तनवाणचन
अयुि सेवा ितण और कारबार का संव्यवहार) ऄक्तधक्तनयम, 1991 को ऄक्तधक्तनयक्तमत दकया है। ऄक्तधक्तनयम के ऄनुसार, मुख्य
क्तनवाणचन अयुि और क्तनवाणचन अयुि 6 वषण तक या 65 वषण की अयु प्राप्त होने तक पद धारण कर सकते हैं। हािांदक, वे दकसी
भी समय त्यागपत्र दे सकते हैं या ईन्हें कायणकाि की समाक्तप्त से पूवण पद से हिाया जा सकता है।
 कथन 3 सही नहीं है: संक्तवधान में क्तनवाणचन अयोग के सेवाक्तनवृत्त होने वािे सदस्यों को सरकार के ऄधीन ऄन्य दूसरी क्तनयुियों
के क्तिए प्रक्ततबंक्तधत नहीं दकया गया है। यहां तक दक क्तनवाणचन अयोग (क्तनवाणचन अयुि सेवा ितण और कारबार का संव्यवहार)
ऄक्तधक्तनयम, 1991 भी मुख्य क्तनवाणचन अयुि और क्तनवाणचन अयुि को सरकार के ऄधीन दकसी ऄन्य क्तनयुक्ति से प्रक्ततबंक्तधत
नहीं करता है। क्तनवाणचन अयुि सेवाक्तनवृक्तत्त के पिात दकसी भी राजनीक्ततक दि में िाक्तमि हो सकते हैं या नहीं, आसमें यह भी
ईक्तल्िक्तखत नहीं है।

Q 83. D
 सभी नागररकों को समान दजाण प्रदान करने हेत,ु संक्तवधान में ऄनुच्छेद 18 के ऄंतगणत ईपाक्तधयों का ऄंत कर ददया गया है और
आस संबंध में क्तनम्नक्तिक्तखत ईपबंध दकए गए हैं।
o यह क्तनषेध करता है दक राज्य दकसी नागररक या दकसी क्तवदेिी को कोइ ईपाक्तध (सेना या क्तवद्या संबंधी सम्मान के क्तसवाय)
प्रदान नहीं करे गा।
o यह भारत का दकसी भी नागररक को क्तवदेिी राज्य से कोइ भी ईपाक्तध प्राप्त करने से क्तनषेध करता है।
o कोइ भी व्यक्ति, जो भारत का नागररक नहीं है, राज्य के ऄधीन िाभ या क्तवश्वास के दकसी पद को धारण करते हुए दकसी
क्तवदेिी राज्य से कोइ भी ईपाक्तध राष्ट्रपक्तत की सहमक्तत के क्तबना स्वीकार नहीं कर सकता है।
o राज्य के ऄधीन िाभ या क्तवश्वास का पद धारण करने वािा कोइ नागररक या क्तवदेिी व्यक्ति दकसी क्तवदेिी राज्य से या
ईसके ऄधीन दकसी भी रूप में कोइ भेंि, ईपिक्तब्ध या पद राष्ट्रपक्तत की सहमक्तत के क्तबना स्वीकार नहीं कर सकता है।
 भारतीय राज्यक्षेत्र के ऄधीन िाभ का पद धारण करने वािा कोइ क्तवदेिी व्यक्ति, राष्ट्रपक्तत की पूवण ऄनुमक्तत से क्तवदेिी राज्य
द्वारा प्रदत्त दकसी ईपाक्तध के साथ-साथ ईपिक्तब्ध को स्वीकार कर सकता है। आसक्तिए कथन 2 सही नहीं है।
 ऄनुच्छेद 18, औपक्तनवेक्तिक राज्य के समय ददए जाने वािे वंिानुगत कु िीन ईपाक्तधयों जैसे महाराजा, राज बहादुर, राय
बहादुर, राय साहेब, दीवान बहादुर अदद को भी प्रक्ततबंक्तधत करता है।
 हािांदक, यह दकसी व्यक्ति को महाराजा "नाम" रखने से प्रक्ततबंक्तधत नहीं करता है। ईदाहरणत:, कोइ व्यक्ति महाराजा रक्तव, जैसे
नाम का चयन कर सकता है। यह राज्य को के वि आस तरह के वंिानुगत ईपाक्तधयों को प्रदान करने से रोकता है। आसक्तिए कथन
1 सही नहीं है।

34 www.visionias.in ©Vision IAS


 ईच्चतम न्यायािय ने राष्ट्रीय पुरस्कारों- भारत रत्न, पद्म क्तवभूषण, पद्म भूषण, और पद्म श्री की सांक्तवधाक्तनक वैधता को बरकरार
रखा। ईच्चतम न्यायािय ने यह क्तनणणय ददया है दक ये पुरस्कार ऄनुच्छेद 18 के प्रावधानों के तहत ईपाक्तधयां नहीं हैं। ऄनुच्छेद
18 के वि वंिानुगत कु िीन ईपाक्तधयों पर प्रक्ततबंध अरोक्तपत करता है। आसक्तिए ये पुरस्कार ऄनुच्छेद 18 का ईल्िंघन नहीं करते
हैं क्योंदक समानता के क्तसिांत यह ऄक्तधदेक्तित नहीं करते हैं दक योग्यता को मान्यता नहीं दी जानी चाक्तहए।
 न्यायािय ने यह भी क्तनणणय ददया दक आन पुरस्कारों का ईपयोग नाम के प्रत्यय या ईपसगण के रूप में नहीं दकया जाना चाक्तहए,
ऄन्यथा, ईन्हें पुरस्कारों को त्यागना होगा।

Q 84. C
 क्तपट्स आं क्तडया एक्ि, 1784 ने कं पनी के वाक्तणक्तज्यक और राजनैक्ततक प्रकायों के मध्य पृथक्करण दकया था। आसने क्तनदेिक मंडि
(Court of Directors) को व्यापाररक मामिों का प्रबंधन करने की ऄनुमक्तत दी, ककतु राजनीक्ततक मामिों के प्रबंधन के क्तिए
क्तनयंत्रण बोडण (Board of Control) नामक एक नए क्तनकाय का गठन दकया। आस प्रकार, द्वैध िासन (Dual government)
की एक प्रणािी स्थाक्तपत की गइ। ऄथाणत, इस्ि आं क्तडया कं पनी के वाक्तणक्तज्यक और प्रिासक्तनक कायों के क्तिए पृथक क्तनकायों का
गठन दकया गया। आसक्तिए कथन 2 सही नहीं है।
 1833 के चािणर ऄक्तधक्तनयम की क्तविेषताएं:
o बंगाि के गवनणर-जनरि को भारत का गवनणर-जनरि बना ददया गया। आसमें सभी नागररक और सैन्य िक्तियां क्तनक्तहत थी।
िॉडण क्तवक्तियम बैंरिक भारत के प्रथम गवनणर-जनरि बने।
o आसने बॉम्बे (वतणमान मुब ं इ) और मद्रास के गवनणरों को क्तवधाक्तयका संबध
ं ी िक्तियों से वंक्तचत कर ददया। भारत के गवनणर-
जनरि को सम्पूणण क्तब्ररिि भारत में क्तवधाक्तयका के ऄसीक्तमत ऄक्तधकार दे ददए गए। आसक्तिए कथन 1 सही है।
o आसने एक व्यापाररक क्तनकाय के रूप में इस्ि आं क्तडया कं पनी की गक्ततक्तवक्तधयों को समाप्त कर ददया। यह ऄब क्तविुि रूप से
एक प्रिासक्तनक क्तनकाय बन गइ। आसके ऄंतगणत कं पनी के ऄक्तधकार वािे क्षेत्र को क्तब्ररिि राजिाही और ईसके
ईत्तराक्तधकाररयों के ऄधीन कर ददया गया।
o 1833 के चािणर ऄक्तधक्तनयम ने क्तसक्तवि सेवकों के चयन के क्तिए खुिी प्रक्ततयोक्तगता का एक तंत्र स्थाक्तपत करने का प्रयास
दकया। हािांदक, क्तनदेिक मंडि के क्तवरोध के बाद आस प्रावधान को समाप्त कर ददया गया था। ऄंत में, 1853 के चािणर एक्ि
ने क्तसक्तवि सेवकों के चयन और भती की एक खुिी प्रक्ततयोक्तगता प्रणािी िुरू की। आसक्तिए कथन 3 सही नहीं है।

Q 85. D
 मेनका गांधी मामिे (1978) में, ईच्चतम न्यायािय ने ऄनुच्छेद 21 की व्यापक व्याख्या की और माना दक ऄनुच्छेद 21 में 'प्राण
के ऄक्तधकार' (Right to Live) को के वि जीव के ऄक्तस्तत्व या जीक्तवत रहने तक ही सीक्तमत नहीं रखा गया है, बक्तल्क आसमें
मानवीय गररमा के साथ जीना और वे सब पहिू जो जीवन को ऄथणपूण,ण पूणण तथा जीने योग्य बनाते हैं, अदद िाक्तमि हैं।
 आसमें यह भी कहा गया दक ऄनुच्छेद 21 के ऄंतगणत 'व्यक्तिगत स्वतंत्रता' का अयाम व्यापक है और आसमें क्तवक्तभन्न ऐसे ऄक्तधकार
िाक्तमि हैं जो दकसी व्यक्ति की व्यक्तिगत स्वतंत्रता का क्तनमाणण करते हैं।
 सतवंत लसह साहनी बनाम डी. रामारत्नम मामिे में, क्तवदेि यात्रा करने के ऄक्तधकार के मुद्दे के बारे में, ईच्चतम न्यायािय ने
स्थाक्तपत दकया है दक “व्यक्तिगत स्वतंत्रता” की ऄक्तभव्यक्ति में घूमने और क्तवदेि यात्रा करने का ऄक्तधकार िाक्तमि है।
 सतीि चंद्र वमाण बनाम भारत संघ 2019 के मामिे में ईच्चतम न्यायािय ने दोहराया दक क्तववाह और पररवार की तरह क्तवदेि
यात्रा करने का ऄक्तधकार एक वास्तक्तवक और बुक्तनयादी मानव ऄक्तधकार है।
o क्तवदेि यात्रा करने का ऄक्तधकार, दकसी व्यक्ति के क्तिए एक महत्वपूणण बुक्तनयादी मानवीय ऄक्तधकार है, क्योंदक यह न के वि
ईसके दक्रयाकिापों की स्वतंत्रता को बढ़ाकर, बक्तल्क ईसके ऄनुभव के दायरे को बढ़ाकर भी व्यक्ति के स्वतंत्र और अत्म
क्तनधाणरण करने के रचनात्मक चररत्र को पोक्तषत करता है।
 प्रक्ततष्ठा का ऄक्तधकार (Right to Reputation) संक्तवधान के ऄनुच्छेद 21 का एक ऄक्तभन्न ऄंग रहा है। प्रत्येक व्यक्ति को
गररमापूणण जीवन जीने का ऄक्तधकार है। प्रक्ततष्ठा और सम्मान, गररमा के साथ जुडे हुए हैं और आस प्रकार, मानव जीवन का एक
ऄिू ि क्तहस्सा है।
 सुब्रमण्यम स्वामी बनाम भारत संघ के हाक्तिया फै सिे में, ईच्चतम न्यायािय ने माना दक दकसी व्यक्ति की प्रक्ततष्ठा, संक्तवधान के
ऄनुच्छेद 21 के ऄंतगणत एक अधारभूत तत्व है।

35 www.visionias.in ©Vision IAS


 भारत के महान्यायवादी बनाम िछमा देवी ( Lachma Devi) मामिे में ईच्चतम न्यायािय ने कहा दक राजस्थान ईच्च
न्यायािय द्वारा ददया गया क्तनदेि ऄथाणत, मौत की सजा का सावणजक्तनक क्तनष्पादन, ऄसंवध
ै ाक्तनक और ऄनुच्छेद 21 का ईल्िंघन
है।
 आसके ऄक्ततररि यह स्पष्ट दकया गया दक सावणजक्तनक फांसी से मृत्यु दंड देना बबणरता होगी। भिे ही क्तजस ऄपराध के क्तिए
अरोपी को दोषी पाया गया है, वह बबणर हो, दफर भी यह सभ्य समाज के क्तिए िमणनाक होगा दक वह वैसे ही कृ त्य को दोहराया
जाए।
 प्रेम िंकर बनाम ददल्िी प्रिासन 1980 के मामिे में, ईच्चतम न्यायािय ने ईन क्तनयमों को हिा ददया, क्तजनमें कहा गया था दक
तीन साि से ऄक्तधक कारावास की सजा वािे गैर-ज़मानती ऄपराध के क्तिए क्तवचाराधीन प्रत्येक व्यक्ति को क्तनयक्तमत रूप से
हथकडी में रहना पडता है।
 आसक्तिए क्तवकल्प (d) सही ईत्तर है।

Q 86.C
 ऄक्तधकार पृच्छा का ऄथण दकसी ‘प्राक्तधकरण या वारं ि के द्वारा’ है। आसे न्यायािय द्वारा दकसी व्यक्ति द्वारा सावणजक्तनक कायाणिय में
दायर ऄपने दावे की वैधता की जांच करने के क्तिए जारी दकया जाता है। ऄतः, यह दकसी व्यक्ति द्वारा सावणजक्तनक कायाणिय के
ऄवैध ऄनक्तधकार ग्रहण करने से रोकता है।
 ररि को एक क्तवक्तध या संक्तवधान द्वारा क्तनर्ममत स्थायी प्रकृ क्तत के वास्तक्तवक सावणजक्तनक कायाणिय के मामिे में ही जारी दकया जा
सकता है। आसे मंत्रािय के ऄधीन कायाणिय या क्तनजी कायाणिय के मामिों में जारी नहीं दकया जा सकता है।
 कोइ भी आच्छु क व्यक्ति ऄक्तधकार पृच्छा की मांग कर सकता है, यह अवश्यक नहीं दक वह पीक्तडत व्यक्ति ही हो। आसक्तिए कथन
1 सही नहीं है।
 परमादेि का िाक्तब्दक ऄथण है 'हम अदेि देते हैं'। यह एक अदेि है क्तजसे न्यायािय द्वारा एक िोक सेवक को ईसके अक्तधकाररक
कतणव्यों का पािन करने के क्तिए जारी दकया जाता है क्तजसे करने में वह या तो क्तवफि रहा है या करने से आनकार कर ददया है।
 यह दकसी भी सावणजक्तनक क्तनकाय, एक क्तनगम, एक ऄधीनस्थ न्यायािय, एक ऄक्तधकरण या सरकार के क्तवरुि समान ईद्देश्य के
क्तिए जारी दकया जा सकता है।
 परमादेि ररि को जारी नहीं दकया जा सकता है
o एक क्तनजी व्यक्ति या क्तनकाय के क्तवरुि। आसक्तिए कथन 2 सही है।
o ऐसे क्तवभागीय क्तनदेि को िागू करने के क्तिए क्तजनका कोइ सांक्तवक्तधक अधार नहीं है।
o जब कतणव्य क्तववेकाधीन हो तथा ऄक्तनवायण नहीं हो।
o एक संक्तवदात्मक दाक्तयत्व को िागू करने के क्तिए।
o भारत के राष्ट्रपक्तत या राज्य के राज्यपािों के क्तवरुि।
o ईच्च न्यायािय के मुख्य न्यायाधीि के क्तवरुि जो ऄपनी न्याक्तयक क्षमता में कायणरत हो।
 ईत्प्रेषण का ऄथण है 'प्रमाक्तणत होना' या 'सूक्तचत दकया जाना'। आसे एक ईच्च न्यायािय द्वारा ऄधीनस्थ न्यायाियों या ऄक्तधकरणों
को जारी दकया जाता है, या तो ऄधीनस्थ न्यायाियों में िंक्तबत मामिे को ईसे स्थानांतररत करने के क्तिए या दकसी मामिे में
ऄधीनस्थ न्यायाियों के अदेि को समाप्त (squash) करने के क्तिए।
 यह ऄक्तधकार क्षेत्र के ऄक्ततक्रमण या ऄक्तधकार क्षेत्र के ऄभाव या कानून की त्रुरि के अधार पर जारी दकया जाता है। आस प्रकार
प्रक्ततषेध, जोदक के वि क्तनवारक है, के क्तवपरीत ईत्प्रेषण क्तनवारक के साथ-साथ ईपचारात्मक भी है।
 पहिे, ईत्प्रेषण ररि के वि न्याक्तयक और ऄधण-न्याक्तयक ऄक्तधकाररयों के क्तवरुि ही जारी की जा सकती थी और प्रिासक्तनक
ऄक्तधकाररयों के क्तवरुि नहीं। दकन्तु 1991 में, ईच्चतम न्यायािय ने यह क्तनणणय ददया दक व्यक्तियों के ऄक्तधकारों को प्रभाक्तवत
करने वािे प्रिासक्तनक ऄक्तधकाररयों के क्तवरुि भी ईत्प्रेषण ररि जारी दकया जा सकता है। आसक्तिए कथन 3 सही है।
Q 87.C
 ऄनुच्छेद 3 के ऄंतगणत नाम पररवतणन से संबंक्तधत क्तवचाराधीन क्तवधेयक को दो ितों को पूरा करना होता है:
o ऐसा क्तवधेयक संसद में राष्ट्रपक्तत की पूवण ऄनुिस
ं ा के साथ ही प्रस्तुत दकया जा सकता है;
o क्तवधेयक की ऄनुिंसा करने से पहिे, राष्ट्रपक्तत क्तवधेयक को संबंक्तधत राज्य क्तवधानमंडि को एक क्तनर्ददष्ट ऄवक्तध के भीतर
ईसके मत व्यि करने हेतु संदर्मभत करता है।
36 www.visionias.in ©Vision IAS
 राष्ट्रपक्तत राज्य क्तवधानमंडि के मत को मानने के क्तिए बाध्य नहीं है और समय पर मत प्राप्त होने पर भी वह ईसे स्वीकार या
ऄस्वीकार कर सकता है।
 आसके ऄक्ततररि, संक्तवधान (ऄनुच्छेद 4) स्वयं यह घोषणा करता है दक नए राज्यों के प्रवेि या स्थापना (ऄनुच्छेद 2 के तहत)
और नए राज्यों के गठन और क्षेत्रों, सीमाओं या क्तवद्यमान राज्यों के नामों में पररवतणन (ऄनुच्छेद 3 के तहत) हेतु क्तनर्ममत
क्तवक्तधयों को ऄनुच्छेद 368 के तहत संक्तवधान संिोधनों के रूप में नहीं माना जाता है। आसका तात्पयण यह है दक आस प्रकार के
क्तवक्तधयों को एक साधारण बहुमत और साधारण क्तवधायी प्रदक्रया द्वारा पाररत दकया जा सकता है।
 आसक्तिए क्तवकल्प (c) सही ईत्तर है।

Q 88.D
 िोकपाि, िोकपाि और िोकायुि ऄक्तधक्तनयम, 2013 के तहत स्थाक्तपत एक सांक्तवक्तधक क्तनकाय है। यह एक "ओम्बड्समैन
(ombudsman)" के रूप में कायण करता है और कु छ सावणजक्तनक ऄक्तधकाररयों के क्तवरुि और संबंक्तधत मामिों के क्तिए भ्रष्टाचार
के अरोपों की जांच करता है। आसके क्षेत्राक्तधकार में प्रधानमंत्री, मंत्री, संसद के सदस्य, समूह A, B, C और D के ऄक्तधकारी और
कें द्र सरकार के ऄक्तधकारी सक्तम्मक्तित हैं।
 क्तवकल्प 1 सही नहीं है: यद्यक्तप िोकपाि के क्षेत्राक्तधकार का क्तवस्तार मंक्तत्रयों तक है, दफर भी यह मंक्तत्रयों और संसद सदस्यों
द्वारा संसद में कही गइ दकसी बात या संसद में दकए गए मतदान के मामिे में दकसी भी त्य की जांच नहीं कर सकता है।
 क्तवकल्प 2 सही नहीं है: सरकार द्वारा सहायता प्राप्त संस्थानों को िोकपाि के दायरे (क्षेत्राक्तधकार) से बाहर रखा गया है।
हािांदक, सरकार द्वारा पूणण एवं अंक्तिक रूप से क्तवत्त पोक्तषत संस्थान िोकपाि के क्षेत्राक्तधकार में अते हैं।
 क्तवकल्प 3 सही नहीं है: िोकपाि और िोकायुि ऄक्तधक्तनयम, 2013 के ऄनुसार, क्तवदेिी ऄक्तभदाय (क्तवक्तनयमन) ऄक्तधक्तनयम
(FCRA) के संदभण में क्तवदेिी स्रोतों से प्रक्तत वषण 10 िाख रुपये से ऄक्तधक ऄक्तभदाय प्राप्त करने वािी संस्थाओं को (और न दक
सभी संस्थाओं को) िोकपाि के क्षेत्राक्तधकार में रखा गया है।

Q 89.C
 1976 में, मूि कतणव्यों के क्तवषय में ऄनुिस
ं ा करने हेतु सरदार स्वणण लसह सक्तमक्तत का गठन दकया गया था। सक्तमक्तत ने संक्तवधान
में अठ मूि कतणव्यों को समाक्तवष्ट करने का सुझाव ददया था दकन्तु 42 वें संक्तवधान संिोधन ऄक्तधक्तनयम, 1976 द्वारा आसमें दस
मूि कतणव्यों को िाक्तमि दकया गया था।
 स्वणण लसह सक्तमक्तत की कु छ ऄनुिस
ं ाओं को सरकार द्वारा स्वीकार नहीं दकया गया था। आसमें क्तनम्नक्तिक्तखत िाक्तमि है:
 संसद दकसी भी मूि कतणव्य के ऄनुपािन नहीं करने पर जुमाणना/दंड िगाने का प्रावधान कर सकती है। आसक्तिए कथन 2 सही है।
 आस तरह का जुमाणना िगाने वािा दकसी भी कानून को मूि ऄक्तधकारों के ईल्िंघन या संक्तवधान के दकसी ऄन्य प्रावधान के साथ
ऄसंगतता के दकसी भी अधार पर न्यायािय में चुनौती नहीं दी जाएगी।
 नागररकों के क्तिए कर भुगतान करने का कतणव्य भी एक मूि कतणव्य होना चाक्तहए। आसक्तिए कथन 1 सही है।

Q 90.B
 मूि भारतीय संक्तवधान में अठ ऄनुसूक्तचयां थीं। वषण 1951 में संक्तवधान में प्रथम बार संिोधन दकया गया था आसी के साथ नौवीं
ऄनुसूची संक्तवधान में िाक्तमि की गयी।
 आसे प्रथम संक्तवधान (संिोधन) ऄक्तधक्तनयम, 1951 द्वारा सृक्तजत दकया गया था क्तजसके ऄंतगणत कृ क्तष सुधार से संबक्तं धत कानूनों का
संरक्षण करने और जमींदारी प्रथा का ईन्मूिन करने के क्तिए 31A के साथ एक नया ऄनुच्छेद 31B जोडा गया था। आसक्तिए
कथन 1 और 2 सही हैं।
 यद्यक्तप प्रथम संिोधन (1951) मूि ऄक्तधकारों के ईल्िंघन के अधार पर नौंवी ऄनुसच
ू ी में िाक्तमि कानूनों की न्याक्तयक जांच से
रक्षा करने के क्तिए दकया गया था।
 हािांदक, जनवरी 2007 में ईच्चतम न्यायािय ने क्तनणणय ददया दक नौवीं ऄनुसच
ू ी में सक्तम्मक्तित कानूनों की न्याक्तयक पुनर्मविोकन
से कोइ क्तनबंध ईन्मुक्ति (blanket immunity) नहीं हो सकती। न्यायािय द्वारा कहा गया दक न्याक्तयक पुनर्मविोकन संक्तवधान
की एक 'मूि क्तविेषता' है और आसे नौवीं ऄनुसच
ू ी के तहत एक कानून बनाकर समाप्त नहीं दकया जा सकता है। न्यायािय द्वारा
यह कहा गया दक 24 ऄप्रैि, 1973 के बाद नौवीं ऄनुसच
ू ी में सक्तम्मक्तित कानूनों को न्यायािय में चुनौती दी जा सकती हैं, यदद

37 www.visionias.in ©Vision IAS


ये संक्तवधान के ऄनुच्छेद 14, 15, 19 और 21 के तहत गारं िीकृ त मूि ऄक्तधकारों या 'मूि ढांचा' का ईल्िंघन करते हैं। (24
ऄप्रैि, 1973 को ईच्चतम न्यायािय द्वारा के िवानंद भारती मामिे में ऄपने ऐक्ततहाक्तसक क्तनणणय में संक्तवधान के मूि ढांचा ’या
मूि क्तविेषताओं’ के क्तसिांत को प्रथम बार प्रक्ततपाददत दकया गया था)। आसक्तिए कथन 3 सही नहीं है।

Q 91.C
 मूि कतणव्य मूि संक्तवधान का भाग नहीं थे। आसे 42 वें संिोधन ऄक्तधक्तनयम के माध्यम से 1976 में सक्तम्मक्तित दकया गया था।
मूि रूप से दस कतणव्यों को मूि कतणव्यों के रूप में संक्तवधान के भाग IV-A के रूप में जोडा गया था, क्तजसमें ऄनुच्छेद 51-A
सक्तम्मक्तित है। हािांदक, 2002 में 86 वें संक्तवधान संिोधन ऄक्तधक्तनयम के माध्यम से एक और मूि कतणव्य जोडा गया था।
 सरदार स्वणण लसह सक्तमक्तत की क्तसफाररिों के अधार पर संक्तवधान में मूि कतणव्यों को जोडा गया था। ये भूतपूवण सोक्तवयत संघ के
संक्तवधान से प्रेररत हैं। आसक्तिए कथन 1 सही है।
 स्वणण लसह सक्तमक्तत ने कतणव्यों का पािन न करने पर दंड के प्रावधानों की भी क्तसफाररि की थी। हािांदक आसे संक्तवधान में नहीं
जोडा गया था। संक्तवधान आन कतणव्यों के प्रवतणन के बारे में कु छ नहीं कहता है। आसक्तिए कथन 2 सही है।
 न्यायाियों द्वारा मूि कतणव्यों का ईपयोग दकसी क्तवक्तध की संवध
ै ाक्तनक वैधता की जांच में दकया जा सकता है। संसद क्तवक्तध के
जररए भी आसे िागू भी कर सकती है।
 संक्तवधान कतणव्यों की पूर्मत करने के अधार या ितण पर ऄक्तधकारों का ईपभोग करने का प्रावधान नहीं करता है। आसक्तिए कथन
3 सही नहीं है। यह दिाणता है दक मूि कतणव्यों के समावेिन ने हमारे मूि ऄक्तधकारों की क्तस्थक्तत में कोइ पररवतणन नहीं दकया है।

Q 92.C
 ऄंिाकण रिका के '्वाआट्स ग्िेक्तियर (Thwaites Glacier)’ को "डू म्सडे ग्िेक्तियर (Doomsday Glacier)" कहा जाता है, जो
तीव्र संचिन एवं वषों से तीव्रता से क्तपघिने के कारण 120 दकिोमीिर (स्वयं का सवाणक्तधक) चौडा हो गया है। ऄपने अकार
(1.9 िाख वगण दकिोमीिर) के कारण आसमें वैक्तश्वक जि स्तर को अधे मीिर से ऄक्तधक बढ़ा देने के क्तिए पयाणप्त जि है।
 जिवायु पररवतणन के कारण वैक्तश्वक समुद्र ति के स्तर में वृक्ति में तीव्रता िाने की आसकी ईच्च क्षमता के कारण यह िंबे समय से
लचता का कारण बना हुअ है। ऄध्ययन में पाया गया है दक क्तपछिे 30 वषों में आससे प्रवाक्तहत होने वािी क्तहम की मात्रा िगभग
दोगुनी हो गइ है।
 वतणमान में, ्वाआट्स ग्िेक्तियर के क्तपघिने से वैक्तश्वक समुद्र ति के स्तर में प्रत्येक वषण 4% की वृक्ति हो रही है। ऄनुमान है दक
यह 200-900 वषों में समुद्र में समा जाएगा। ऄंिाकण रिका के क्तिए ्वाआट्स महत्वपूणण है क्योंदक यह ऄपने पीछे क्तस्थत बफण को
समुद्र में स्वतंत्र रूप से प्रवाक्तहत होने से रोकता है। आसके समक्ष ईपक्तस्थत जोक्तखम एवं ईस जोक्तखम को रोकने की क्षमता के
कारण ्वाआट्स को प्रायः डू म्सडे ग्िेक्तियर कहा जाता है।
 आसक्तिए क्तवकल्प (c) सही ईत्तर है।

Q 93.A
 राष्ट्रीय अपात की ईद्घोषणा पूरे देि या ईसके दकसी एक भाग के संबंध में की जा सकती है। 1976 के 42वें संिोधन ऄक्तधक्तनयम
ने राष्ट्रपक्तत को राष्ट्रीय अपात के संचािन को भारत के एक क्तनर्ददष्ट भाग में सीक्तमत करने हेतु सक्षम बनाया है। आसक्तिए कथन 1
सही है।
 हािांदक, राष्ट्रपक्तत मंक्तत्रमंडि से क्तिक्तखत ऄनुिस
ं ा प्राप्त करने के पिात् ही राष्ट्रीय अपात की ईद्घोषणा कर सकता है। आसका
तात्पयण यह है दक अपात की ईद्घोषणा के वि मंक्तत्रमंडि की सहमक्तत से ही हो सकती है न दक मात्र प्रधानमंत्री की सिाह से।
आसक्तिए कथन 2 सही नहीं है।
 1975 में, तत्कािीन प्रधानमंत्री आं ददरा गांधी ने मंक्तत्रमंडि से परामिण दकए क्तबना राष्ट्रपक्तत को अपातकाि की ईद्घोषणा करने
की सिाह दी थी। अपातकाि िागू करने के पिात् मंक्तत्रमंडि को आस ईद्घोषणा के बारे में सूक्तचत दकया गया था। 1978 के 44
वें संिोधन ऄक्तधक्तनयम ने प्रधानमंत्री द्वारा आस संदभण में ऄके िे क्तनणणय िेने की संभावना को समाप्त करने हेतु आस सुरक्षा प्रावधान
को अरं भ दकया।

38 www.visionias.in ©Vision IAS


Q 94.D
 संक्तवधान का ऄनुच्छेद 280 भारत में राजकोषीय संघवाद के संति
ु न चक्र के रूप में क्तवत्त अयोग का प्रावधान करता है। यह एक
ऄधण-न्याक्तयक क्तनकाय है, क्तजसका गठन राष्ट्रपक्तत द्वारा प्रत्येक पांचवें वषण या अवश्यकतानुसार ईससे पहिे दकया जाता है।
 कथन 1 सही है: संक्तवधान का ऄनुच्छेद 280 अयोग की संरचना को क्तनधाणररत करता है। यह ईपबंध करता है दक क्तवत्त अयोग
में एक ऄध्यक्ष और चार ऄन्य सदस्य होंगे, क्तजन्हें राष्ट्रपक्तत द्वारा क्तनयुि दकया जाएगा। ईनका कायणकाि राष्ट्रपक्तत के अदेि के
तहत क्तनधाणररत होगा और वे पुनर्मनयुक्ति के क्तिए पात्र होंगे।
 कथन 2 सही है: क्तवत्त अयोग (प्रकीणण ईपबंध) ऄक्तधक्तनयम, 1951 के तहत संसद द्वारा अयोग के ऄध्यक्ष और सदस्यों की
ऄहणताओं को क्तनर्ददष्ट दकया जाता है। ऄक्तधक्तनयम के ऄनुसार, ऄध्यक्ष को िोक कायों में ऄनुभव प्राप्त होना चाक्तहए। ऄतः राज्य के
तीनों ऄंगों- कायणपाक्तिका, क्तवधाक्तयका और न्यायपाक्तिका के ऄंतगणत अने वािे व्यक्ति क्तनयुक्ति के क्तिए पात्र हैं। पी. वी.
राजमन्नार जो मद्रास ईच्च न्यायािय के मुख्य न्यायाधीि थे, चौथे क्तवत्त अयोग के ऄध्यक्ष थे।
 कथन 3 सही है: हािांदक क्तवत्त अयोग एक ऄधण-न्याक्तयक क्तनकाय है, िेदकन आसकी क्तसफाररिें सरकार के क्तिए बाध्यकारी नहीं
हैं। क्तसफाररिें प्रकृ क्तत में के वि सिाहकारी हैं और यह सरकार पर क्तनभणर करता है दक वह आसकी क्तसफाररिों को कायाणक्तन्वत करे
या नहीं।

Q 95.C
 हाक्तिया संदभण: सरकार ने समुद्रपारीय भारतीय नागररक (OCI) काडण को बनाए रखने की प्रदक्रया को सरि बनाया है। ऄब,
OCI काडणधारकों को वतणमान में अवश्यक कइ बार के बजाय 20 वषण की अयु में के वि एक बार ऄपने पुनः जारी दस्तावेज
प्राप्त करने की अवश्यकता होगी। आसक्तिए कथन 3 सही है।
 समुद्रपारीय भारतीय नागररक (OCI) काडण भारत में दीघाणवक्तध वीजा-मुि यात्रा और प्रवास की सुक्तवधा प्रदान करते हैं और
क्तविेष रूप से काडणधारकों को प्राप्त होने वािे क्तविेषाक्तधकारों का प्रक्ततक्तनक्तधत्व करते हैं, जोदक क्तवदेिी नागररक को प्राप्त नहीं हैं।
आसक्तिए कथन 1 सही है।
 नइ ऄक्तधसूचना के ऄनुसार ऄब एक OCI काडणधारक को ऄनुसंधान कायण करने के क्तिए क्तविेष ऄनुमक्तत िेनी होगी। ऄक्तधसूचना
में OCI काडणधारकों को ऄक्तखि भारतीय प्रवेि परीक्षा जैसे दक मेक्तडकि पाठ्यक्रमों के क्तिए NEET, आंजीक्तनयररग के क्तिए
JEE- मेन और एडवांस- या दकसी ऄन्य परीक्षण के क्तिए अवेदन करने की पात्रता है। हािांदक, यह के वि समय-समय पर
ईपयुि प्राक्तधकारी और सरकार द्वारा ऄनुमोददत स्वीकृ त ग्राह्यता से ऄक्तधक होने और दकसी भी NRI या ऄक्ततररि सीि के
क्तवरुि होगा। आसक्तिए कथन 2 सही है

Q 96.C
 हाक्तिया संदभण: संयुि राज्य ऄमेररका के राष्ट्रपक्तत जो बाआडेन ने ओिोमन तुकों द्वारा अमेक्तनयाइ िोगों की सामूक्तहक हत्या को
अक्तधकाररक तौर पर “नरसंहार के कृ त्य” (act of genocide) की संज्ञा दी है।
 पृष्ठभूक्तम: प्रथम क्तवश्व युि के िुरुअती चरण में ओिोमन साम्राज्य के क्षेत्रों के भीतर िगभग 1.5 क्तमक्तियन अमेक्तनयाइ िोगों की
हत्या की गइ थी। 2019 में, ऄमेररकी कांग्रेस (संयुि राज्य ऄमेररका की संसद) ने हत्याओं को नरसंहार की संज्ञा देते हुए
प्रस्ताव पाररत दकया था, परन्तु डोनाल्ड ट्रम्प प्रिासन ने अक्तधकाररक तौर पर ऐसा होने से रोक ददया था।
 यूरोप और एक्तिया के मध्य क्तस्थत कािा सागर, ऄििांरिक महासागर का एक सीमांत सागर है;
 कािा सागर बुल्गाररया, जॉर्मजया, रोमाक्तनया, रूस, तुकी और यूक्रेन से क्तघरा हुअ है।

39 www.visionias.in ©Vision IAS


Q 97.A
 हाि ही में RBI ने घोषणा की है दक खुदरा क्तनवेिक सीधे सरकारी प्रक्ततभूक्ततयों (G-Sec) में क्तनवेि कर सकते है।
 ऄब तक, सरकारी प्रक्ततभूक्ततयों की पहुुँच प्रत्यक्ष रूप से के वि संस्थागत प्रक्ततभाक्तगयों जैसे दक बैंक, प्राथक्तमक डीिसण, बीमा
कं पक्तनयों, म्यूचऄ
ु ि फं ड्स, क्तवदेिी पोिणफोक्तियो क्तनवेिक अदद तक सीक्तमत थी। आसक्तिए RBI द्वारा सरकारी प्रक्ततभूक्तत बाजार में
खुदरा क्तनवेिकों को सीधे पहुंच प्रदान करने का यह क्तनणणय एक प्रमुख संरचनात्मक सुधार है।
 RBI, खुदरा क्तनवेिकों को सीधे RBI से सरकारी प्रक्ततभूक्ततयां खरीदने में सक्षम बनाने हेतु "ररिेि डायरे क्ि" नामक एक नया
प्िेिफॉमण िॉन्च करे गा। आसक्तिए क्तवकल्प (a) सही ईत्तर है।
 सरकारी प्रक्ततभूक्ततयां खरीदने के क्तिए, खुदरा क्तनवेिकों को RBI के साथ ऄपने क्तगल्ि खाते (िेयर/बांड अदद खरीदने हेतु डीमैि
खाते के समान) खोिने की अवश्यकता होगी।

Q 98.A
 ऄनुच्छेद 355, कें द्र पर यह कतणव्य ऄक्तधरोक्तपत करता है दक वह सुक्तनक्तित करे दक प्रत्येक राज्य सरकार संक्तवधान के प्रावधानों के
ऄनुसार ही संचाक्तित हो। आस कतणव्य के ऄनुपािन के क्तिए कें द्र, ऄनुच्छेद 356 के तहत राज्य का संवैधाक्तनक तंत्र क्तवफि होने की
दिा में राज्य सरकार को ऄपने क्तनयंत्रण में िे सकता है। क्तजसे िोकक्तप्रय रूप से ‘राष्ट्रपक्तत िासन’ के रूप में जाना जाता है। आसे
'राज्य अपात' या 'संवध
ै ाक्तनक अपातकाि' भी कहा जाता है।
 राष्ट्रपक्तत िासन ऄनुच्छेद 356 के तहत दो अधारों पर ईद्घोक्तषत दकया जा सकता है- पहिा स्वयं ऄनुच्छेद 356 के ऄनुसार एवं
दूसरा ऄनुच्छेद 365 के ऄनुसार।
o ऄनुच्छेद 356, राष्ट्रपक्तत को यह घोषणा जारी करने का ऄक्तधकार देता है, यदद वह अश्वस्त है दक ऐसी क्तस्थक्तत ईत्पन्न हो
गइ है, क्तजसमें राज्य सरकार संक्तवधान के प्रावधानों के ऄनुसार नहीं चि सकती है। क्तविेष रूप से, राष्ट्रपक्तत या तो राज्य के
राज्यपाि से प्रक्ततवेदन क्तमिने पर या ऄन्यथा (ऄथाणत, राज्यपाि की ररपोिण के क्तबना भी) यह कायण कर सकता है।
o ऄनुच्छेद 365, के ऄनुसार जब भी कोइ राज्य, कें द्र से प्राप्त क्तनदेिों का ऄनुपािन करने या ईनको प्रभावी करने में ऄसफि
रहता है, तो वहां राष्ट्रपक्तत के क्तिए यह क्तवक्तधसंगत होगा दक वह यह क्तनधाणररत करे दक ऐसी क्तस्थक्तत ईत्पन्न हो गयी है,
क्तजसमें राज्य सरकार संक्तवधान के प्रावधानों के ऄनुरूप नहीं चि सकती है।
 राष्ट्रपक्तत िासन के संचािन के दौरान, राज्य की कायणपाक्तिका बखाणस्त कर दी जाती है और राज्य की क्तवधाक्तयका को क्तनिंक्तबत
या भंग कर ददया जाता है। राष्ट्रपक्तत राज्यपाि के माध्यम से राज्य का प्रिासन चिाता है। आसक्तिए कथन 1 सही है।
40 www.visionias.in ©Vision IAS
 जब दकसी राज्य में राष्ट्रपक्तत िासन ऄक्तधरोक्तपत दकया जाता है, तो संसद को ईस राज्य से संबक्तं धत राज्य सूची के दकसी भी
क्तवषय पर क्तवक्तध बनाने की िक्ति प्राप्त हो जाती है। संसद द्वारा बनाइ गइ क्तवक्तध राष्ट्रपक्तत िासन के बाद भी संचाक्तित रहती है।
आसक्तिए कथन 2 सही नहीं है।

Q 99.A
 अंचक्तिक/क्षेत्रीय पररषदें (Zonal Councils) संसद के एक ऄक्तधक्तनयम द्वारा गरठत वैधाक्तनक क्तनकाय हैं। प्रत्येक क्षेत्रीय पररषद
का नेतृत्व कें द्रीय गृह मंत्री करते हैं, आसमें क्षेत्र के राज्यों के मुख्यमंत्री, क्षेत्र के संघ राज्य क्षेत्रों के प्रिासक और क्षेत्र के प्रत्येक
राज्य के दो ऄन्य मंत्री िाक्तमि होते हैं।
 कथन 1 सही है: ऄनुच्छेद 263 के तहत राष्ट्रपक्तत संघ और राज्यों के बीच सहज समन्वय के क्तिए ऄंतरराज्यीय पररषद की
स्थापना कर सकते हैं। ईपयुि
ण क्षेत्रीय पररषदें संसद के ऄक्तधक्तनयम द्वारा क्तनर्ममत वैधाक्तनक क्तनकाय हैं।
 कथन 2 सही नहीं है: राज्य पुनगणठन ऄक्तधक्तनयम, 1956 देि को पांच क्षेत्रों में क्तवभाक्तजत करता है- यह पांच क्षेत्र ईत्तरी,
पक्तिमी, कें द्रीय, पूवी और दक्तक्षणी क्षेत्र है। पूवोत्तर पररषद ऄक्तधक्तनयम, 1971/ईत्तर-पूवी पररषद ऄक्तधक्तनयम, 1971 से एक
पृथक पूवोत्तर क्षेत्रीय पररषद का क्तनमाणण दकया गया है। ऄक्तधक्तनयम के ऄनुसार, संघ िाक्तसत प्रदेिों के क्तिए ऄिग क्षेत्रीय
पररषद का प्रावधान नहीं है। ईदाहरण के क्तिए, चंडीगढ़, ददल्िी जैसे संघ राज्य क्षेत्र हररयाणा, पंजाब जैसे राज्यों के साथ
ईत्तरी क्षेत्रीय पररषद में समाक्तवष्ट हैं।

Q 100.C
 बेरूबारी यूक्तनयन मामिे (1960) में, ईच्चतम न्यायािय ने प्रस्तावना के महत्व को मान्यता दी है। ईच्चतम न्यायािय ने ईल्िेख
दकया दक प्रस्तावना से ईन ऄनुच्छेदों की व्याख्या में सहायता िी जा सकती है जो ऄस्पष्ट हैं या क्तजनके एक से ऄक्तधक ऄथण ईत्पन्न
होते हैं। आसक्तिए कथन 1 सही है।
 यह भी ईल्िेख दकया दक प्रस्तावना संक्तवधान में क्तनक्तहत प्रयोजनों के सामान्य ईद्देश्यों को दिाणती है और आस प्रकार यह संक्तवधान
क्तनमाणताओं के मक्तस्तष्क की एक कुं जी है। हािांदक, ईच्चतम न्यायािय ने क्तविेष रूप से ईल्िेख दकया दक प्रस्तावना संक्तवधान का
भाग नहीं है।
 के िवानंद भारती मामिे (1973) में, ईच्चतम न्यायािय ने पूवण की व्याख्या को ऄस्वीकार कर ददया और यह व्यवस्था दी दक
प्रस्तावना संक्तवधान का एक ऄंग है। साथ ही यह माना दक प्रस्तावना में ईक्तल्िक्तखत महान क्तवचारों को ध्यान में रखकर संक्तवधान
का ऄध्ययन दकया जाना चाक्तहए। ईच्चतम न्यायािय ने यह भी कहा दक प्रस्तावना में तब तक संिोधन दकया जा सकता है जब
तक दक प्रस्तावना में ईक्तल्िक्तखत संक्तवधान की मूि क्तविेषताओं में संिोधन न हो।
 एि.अइ.सी ऑफ आं क्तडया मामिे (1995) में भी ईच्चतम न्यायािय ने पुन: व्यवस्था दी दक प्रस्तावना संक्तवधान का ऄक्तभन्न ऄंग
है। आसक्तिए कथन 2 सही है।

Copyright © by Vision IAS


All rights are reserved. No part of this document may be reproduced, stored in a retrieval system or transmitted
in any form or by any means, electronic, mechanical, photocopying, recording or otherwise, without prior
permission of Vision IAS.

41 www.visionias.in ©Vision IAS

You might also like